LEWIS

Ace your homework & exams now with Quizwiz!

At the end of the surgical procedure, the perioperative nurse evaluates the patient's response to the nursing care delivered during the perioperative period. Which of the following criteria reflects an outcome related to the patient's physical status? a. the patient's right to privacy is maintained b. the patient's care is consistent with the perioperative plan of care c. the patient receives consistent and comparable care regardless of the setting d. the patient's respiratory function is consistent with or improved from baseline levels established preoperatively.

D (The Perioperative Nursing Data Set includes outcome statements that reflect standards and recommended practices or perioperative nursing. Outcomes related to physiologic responses include those of physiologic function, such as respiratory function; perioperative safety includes the patient's freedom from any type of injury; and behavioral responses include knowledge and actions of the patient and family, including the consistency of the patient's care with the perioperative plan and the patient's right to privacy.)

Which of the following instructions given to a patient who is about to undergo Holter monitoring is most appropriate? A) "You may remove the monitor only to shower or bathe." B) "You should connect the monitor whenever you feel symptoms." C) "You should refrain from exercising while wearing this monitor." D) "You will need to keep a diary of all your activities and symptoms."

D) "You will need to keep a diary of all your activities and symptoms." A Holter monitor is worn for at least 24 hours while a patient continues with usual activity and keeps a diary of activities and symptoms. The patient should not take a bath or shower while wearing this monitor.

The blood pressure of a 71-year-old patient admitted with pneumonia is 160/70 mm Hg. Which of the following is an age-related change that contributes to this finding? A) Stenosis of the heart valves B) Decreased adrenergic sensitivity C) Increased parasympathetic activity D) Loss of elasticity in arterial vessels

D) Loss of elasticity in arterial vessels An age-related change that increases the risk of systolic hypertension is a loss of elasticity in the arterial walls. Because of the increasing resistance to flow, pressure is increased within the blood vessel and hypertension results.

A 55-year-old female patient develops acute pericarditis after a myocardial infarction. It is most important for the nurse to assess for which clinical manifestation of a possible complication? A. Presence of a pericardial friction rub B. Distant and muffled apical heart sounds C. Increased chest pain with deep breathing D. Decreased blood pressure with tachycardia

D. Decreased blood pressure with tachycardia Cardiac tamponade is a serious complication of acute pericarditis. Signs and symptoms indicating cardiac tamponade include narrowed pulse pressure, tachypnea, tachycardia, a decreased cardiac output, and decreased blood pressure. The other symptoms are consistent with acute pericarditis.

An 80-year-old patient with uncontrolled type 1 diabetes mellitus is diagnosed with aortic stenosis. When conservative therapy is no longer effective, the nurse knows that the patient will need to do or have what done? A. Aortic valve replacement B. Take nitroglycerin for chest pain. C. Open commissurotomy (valvulotomy) procedure D. Percutaneous transluminal balloon valvuloplasty (PTBV) procedure

D. Percutaneous transluminal balloon valvuloplasty (PTBV) procedure

Which of the following information is accurate about amlodipine? a. Reduce the "fight or flight" response b. Decrease spasm of the coronary arteries c. Increase the force of myocardial contraction d. Help prevent clotting in the coronary arteries

Decrease spasm of the coronary arteries

Dark Blue

Depth markings

The nurse is assessing an underweight older-adult client and observes pulsation of the abdominal aorta in the epigastric area. Which of the following actions should the nurse take? a. Notify the hospital rapid response team. b. Instruct the client to remain on bed rest. c. Teach the client about aortic aneurysms. d. Document the finding in the client chart.

Document the finding in the client chart.

. The nurse is monitoring a client who is undergoing exercise (stress) testing on a treadmill. Which of the following assessment findings requires the most rapid action by the nurse? a. Client complaint of feeling tired b. Pulse change from 80 to 96 beats/minute c. BP increase from 134/68 to 150/80 mm Hg d. Electrocardiographic changes indicating coronary ischemia

Electrocardiographic changes indicating coronary ischemia

The nurse is assessing a client who has just arrived in the emergency department and notes a pulse deficit. Which of the following actions should the nurse anticipate for the client? a. A 2-D echocardiogram b. A cardiac catheterization c. Hourly blood pressure checks d. Electrocardiographic monitoring

Electrocardiographic monitoring

. The nurse is developing a health teaching plan for a 60-year-old man with the following risk factors for coronary artery disease (CAD). Which of the following risk factors should the nurse focus on when teaching the client? a. Family history of coronary artery disease b. Increased risk associated with the client's gender c. High incidence of cardiovascular disease in older people d. Elevation of the client's serum low density lipoprotein (LDL) level

Elevation of the client's serum low density lipoprotein (LDL) level

The nurse is caring for a client who has recently started taking rosuvastatin and niacin who reports all of these symptoms to the nurse. Which of the following finding is most important to communicate to the health care provider? a. Generalized muscle aches and weakness b. Skin flushing after taking the medications c. Dizziness when changing positions quickly d. Nausea when taking the drugs before eating

Generalized muscle aches and weakness

Which of the following nursing actions should the nurse take first in order to assist a client with newly diagnosed stage 1 hypertension in making needed dietary changes? a. Have the client record dietary intake for 3 days. b. Give the client a detailed list of low-sodium foods. c. Teach the client about foods that are high in sodium. d. Help the client make an appointment with a dietitian

Have the client record dietary intake for 3 days.

Which of the following actions should the nurse in the hypertension clinic take in order to obtain an accurate baseline blood pressure (BP) for a new client? a. Obtain a BP reading in each arm and average the results. b. Deflate the BP cuff at a rate of 5-10 mm Hg/second. c. Have the client sit in a chair. d. Assist the client to the supine position for BP measurements.

Have the client sit in a chair.

The nurse is caring for a client with a non-ST-segment-elevation myocardial infarction (NSTEMI) who is receiving heparin. Which of the following information explains the purpose of the heparin? a. Platelet aggregation is enhanced by IV heparin infusion. b. Heparin will dissolve the clot that is blocking blood flow to the heart. c. Coronary artery plaque size and adherence are decreased with heparin. d. Heparin will prevent the development of new clots in the coronary arteries.

Heparin will prevent the development of new clots in the coronary arteries.

Which of the following information should the nurse include when teaching a client with newly diagnosed hypertension? a. Dietary sodium restriction will control BP for most clients. b. Most clients are able to control BP through lifestyle changes. c. Hypertension is usually asymptomatic until significant organ damage occurs. d. Annual BP checks are needed to monitor treatment effectiveness.

Hypertension is usually asymptomatic until significant organ damage occurs.

The charge nurse observes a new RN doing discharge teaching for a client who is hypertensive and has a new prescription for enalapril. Which of the following actions by the new RN should cause the charge nurse to intervene in the client's care? a. Check the BP with a home BP monitor every day. b. Move slowly when moving from lying to standing. c. Increase the dietary intake of high-potassium foods. d. Make an appointment with the dietitian for teaching.

Increase the dietary intake of high-potassium foods.

Purple

Inflated cuff

Green

Inflating Tube

Red #2

Irrigation port for saline lavage

The nurse is providing teaching to a client being evaluated for rhythm disturbances with a Holter monitor. Which of the following information should the nurse include in the teaching plan? a. Exercise more than usual while the monitor is in place. b. Remove the electrodes when taking a shower or tub bath. c. Keep a diary of daily activities while the monitor is worn. d. Connect the recorder to a telephone transmitter once daily

Keep a diary of daily activities while the monitor is worn.

Three major mechanisms the body employs to control acid-base balance are cellular buffers, the lungs and the _____________.

Kidneys

Light Blue #2

Modified T piece for ventilator circuit

Red

Murphy eye

The nurse obtains this information from a client with prehypertension. Which of the following findings is most important to address with the client? a. Low dietary fibre intake b. No regular aerobic exercise c. BMI of 23 kg/m2 d. Drinks wine with dinner once a week

No regular aerobic exercise

The nurse is assessing a client who was admitted with heart failure and notes that the client has jugular venous distension (JVD) when lying flat in bed. Which of the following actions should the nurse take next? a. Use a ruler to measure the level of the JVD. b. Document this finding in the client's record. c. Observe for JVD with the head at 45 degrees. d. Have the client perform the Valsalva manoeuvre.

Observe for JVD with the head at 45 degrees.

Orange

One-way valve

The RN is observing a student nurse who is doing a physical assessment on a client. The RN will need to intervene immediately if the student nurse implements which of the following interventions? a. Places the client in the left lateral position to check for the point of maximal impulse (PMI). b. Presses on the skin over the tibia for 10 seconds to check for edema. c. Palpates both carotid arteries simultaneously to compare pulse quality. d. Documents a murmur heard along the left sternal border as an aortic murmur.

Palpates both carotid arteries simultaneously to compare pulse quality.

Yellow

Pilot balloon

Brown

Radiopaque line

Orange #2

Removable plug

The nurse is caring for a client who has had an acute myocardial infarction and the client asks the nurse about when sexual intercourse can be resumed. Which of the following responses by the nurse is best? a. "Most clients are able to enjoy intercourse without any complications." b. "Sexual activity uses about as much energy as climbing two flights of stairs." c. "The doctor will discuss sexual intercourse when your heart is strong enough." d. "Holding and cuddling are good ways to maintain intimacy after a heart attack."

Sexual activity uses about as much energy as climbing two flights of stairs."

The nurse is admitting a client to the emergency department with severe chest pain and gives the following list of medications taken at home to the nurse. Which of the following medications has the most immediate implications for the client's care? a. Sildenafil b. Furosemide c. Diazepam d. Captopril

Sildenafil

22. A 26-yr-old female with type 1 diabetes develops a sore throat and runny nose after caring for her sick toddler. The patient calls the clinic for advice about her symptoms and a blood glucose level of 210 mg/dL despite taking her usual glargine (Lantus) and lispro (Humalog) insulin. The nurse advises the patient to a. use only the lispro insulin until the symptoms are resolved. b. limit intake of calories until the glucose is less than 120 mg/dL. c. monitor blood glucose every 4 hours and notify the clinic if it continues to rise. d. decrease intake of carbohydrates until glycosylated hemoglobin is less than 7%.

c. monitor blood glucose every 4 hours and notify the clinic if it continues to rise. - Infection and other stressors increase blood glucose levels and the patient will need to test blood glucose frequently, treat elevations appropriately with lispro insulin, and call the health care provider if glucose levels continue to be elevated. Discontinuing the glargine will contribute to hyperglycemia and may lead to diabetic ketoacidosis (DKA). Decreasing carbohydrate or caloric intake is not appropriate because the patient will need more calories when ill. Glycosylated hemoglobin testing is not used to evaluate short-term alterations in blood glucose.

A patient is admitted to the PACU after major abdominal surgery. During the initial assessment the patient tells the nurse he thinks he is going to "throw up." A priority nursing intervention would be to: a. increase the rate of IV fluids b. obtain vital signs, including O2 saturation c. position patient in lateral recovery position d. administer antiemetic medication as ordered

c. position patient in lateral recovery position Rationale: If the patient is nauseated and may vomit, place the patient in a lateral recovery position to keep the airway open and reduce the risk of aspiration if vomiting occurs.

17. When a patient who takes metformin (Glucophage) to manage type 2 diabetes develops an allergic rash from an unknown cause, the health care provider prescribes prednisone. The nurse will anticipate that the patient may a. need a diet higher in calories while receiving prednisone. b. develop acute hypoglycemia while taking the prednisone. c. require administration of insulin while taking prednisone. d. have rashes caused by metformin-prednisone interactions.

c. require administration of insulin while taking prednisone. - Glucose levels increase when patients are taking corticosteroids, and insulin may be required to control blood glucose. Hypoglycemia is not a side effect of prednisone. Rashes are not an adverse effect caused by taking metformin and prednisone simultaneously. The patient may have an increased appetite when taking prednisone but will not need a diet that is higher in calories.

The nurse is assessing a client who is being investigated for possible white coat hypertension. Which of the following actions should the nurse implement first? a. Schedule the client for frequent BP checks in the clinic. b. Instruct the client about the need to decrease stress levels. c. Tell the client how to self-monitor and record BPs at home. d. Teach the client about ambulatory blood pressure monitoring

Tell the client how to self-monitor and record BPs at home.

The nurse will suspect that the client with stable angina is experiencing an adverse effect of the prescribed metoprolol if which of the following findings are assessed? a. The client is restless and agitated. b. The blood pressure is 190/110 mm Hg. c. The client complains about feeling anxious. d. The cardiac monitor shows a heart rate of 45.

The cardiac monitor shows a heart rate of 45.

The nurse is teaching a client with stage 1 hypertension about diet modifications that should be implemented. Which of the following diet choices indicates that the teaching has been effective? a. The client avoids eating nuts or nut butters. b. The client restricts intake of dietary protein. c. The client has only one cup of coffee in the morning. d. The client has a glass of low-fat milk with each meal

The client has a glass of low-fat milk with each meal

Which of the following information obtained by the nurse who is admitting the client for magnetic resonance imaging (MRI) will be most important to report to the health care provider before the MRI? a. The client has an allergy to shellfish and iodine. b. The client has a history of coronary artery disease. c. The client has a permanent ventricular pacemaker in place. d. The client took all the prescribed cardiac medications today.

The client has a permanent ventricular pacemaker in place.

During change-of-shift report, the nurse obtains this information about a client who is hypertensive and received the first dose of propranolol during the previous shift. Which of the following information indicates that the client needs immediate intervention? a. The client's most recent BP reading is 156/94 mm Hg. b. The client's pulse has dropped from 64 to 58 beats/minute. c. The client has developed wheezes throughout the lung fields. d. The client complains that the fingers and toes feel quite cold.

The client has developed wheezes throughout the lung fields.

The nurse is admitting a client for a coronary arteriogram and angiogram. Which of the following information about the client is most important for the nurse to communicate to the health care provider? a. The client's pedal pulses are +1. b. The client is allergic to iodine. c. The client has not eaten anything today. d. The client had an arteriogram a year ago.

The client is allergic to iodine.

Which of the following information given by a client admitted with chronic stable angina will help the nurse confirm this diagnosis? a. The client rates the pain at a level 3-5 (0-10 scale). b. The client states that the pain "wakes me up at night." c. The client says that the frequency of the pain has increased over the last few weeks. d. The client states that the pain is resolved after taking one sublingual nitroglycerin tablet.

The client states that the pain is resolved after taking one sublingual nitroglycerin tablet.

The nurse is reviewing the 12-lead electrocardiograph (ECG) of a healthy older-adult client who is having an annual physical examination. Which of the following findings should be of most concern to the nurse? a. The heart rate is 43 beats/minute. b. The PR interval is 0.21 seconds. c. There is a right bundle-branch block. d. The QRS duration is 0.13 seconds.

The heart rate is 43 beats/minute.

The nurse is providing teaching to a client about use of sublingual nitroglycerin. Which of the following client statements indicates that the teaching has been effective? a. "I can expect indigestion as an adverse effect of nitroglycerin." b. "I can only take the nitroglycerin if I start to have chest pain." c. "I will call an ambulance if I still have pain 5 minutes after taking the nitroglycerin." d. "I will help slow down the progress of the plaque formation by taking nitroglycerin."

"I will call an ambulance if I still have pain 5 minutes after taking the nitroglycerin."

The nurse is providing teaching to a client with chronic stable angina about how to use the prescribed short-acting and long-acting nitrates. Which of the following client statements indicates that the teaching has been effective? a. "I will put on the nitroglycerin patch as soon as I develop any chest pain." b. "I will check the pulse rate in my wrist just before I take any nitroglycerin." c. "I will be sure to remove the nitroglycerin patch before using any sublingual nitroglycerin." d. "I will stop what I am doing and sit down before I put the nitroglycerin under my tongue."

"I will stop what I am doing and sit down before I put the nitroglycerin under my tongue."

The nurse has just finished teaching a client who is hypertensive about the newly prescribed quinapril. Which of the following client statements indicates that more teaching is needed? a. "The medication may not work as well if I take any Aspirin." b. "My health care provider may order a blood potassium level occasionally." c. "I will call my health care provider if I notice that I have a frequent cough." d. "I won't worry if I have a little swelling around my lips and face."

"I won't worry if I have a little swelling around my lips and face."

The nurse is providing teaching to a client about the use of atenolol in preventing anginal episodes. Which of the following client statements indicate that the teaching has been effective? a. "It is important not to suddenly stop taking the atenolol." b. "Atenolol will increase the strength of my heart muscle." c. "I can expect to feel short of breath when taking atenolol." d. "Atenolol will improve the blood flow to my coronary arteries."

"It is important not to suddenly stop taking the atenolol."

Which of the following information collected by the nurse who is admitting a client with chest pain suggests that the pain is caused by an acute myocardial infarction? a. The pain increases with deep breathing. b. The pain has persisted longer than 30 minutes. c. The pain worsens when the client raises the arms. d. The pain is relieved after the client takes nitroglycerin

The pain has persisted longer than 30 minutes

Green #2

Thumb control for suction

The nurse hears a murmur between the S1 and S2 heart sounds at the client's left 5th intercostal space and midclavicular line. How should the nurse record this information? a. "Systolic murmur heard at mitral area." b. "Diastolic murmur heard at aortic area." c. "Systolic murmur heard at Erb's point." d. "Diastolic murmur heard at tricuspid area."

"Systolic murmur heard at mitral area."

The nurse is caring for a client who is 3 days post myocardial infarction and the client states, "I just had a little chest pain. As soon as I get out of here, I'm going for my vacation as planned." Which of the following responses should the nurse make? a. "Where are you planning to go for your vacation?" b. "What do you think caused your chest pain episode?" c. "Sometimes plans need to change after a heart attack." d. "Recovery from a heart attack takes at least a few weeks."

"What do you think caused your chest pain episode?"

Which of the following actions should the nurse implement for a client who arrives for a calcium-scoring CT scan? a. Administer oral sedative medications. b. Teach the client about the procedure. c. Ask whether the client has eaten today. d. Insert a large gauge intravenous catheter.

. Teach the client about the procedure.

for the patient and in counseling other family members. The patient should be counseled against the use of stimulant drugs, but the limited past history indicates that the patient is not at current risk for cocaine use. Viral infections and CAD are risk factors for dilated cardiomyopathy, but not for HC.

...

. The nurse is assessing the laboratory results for a client who developed chest pain 4 hours ago and may be having a myocardial infarction. Which of the following laboratory results is most important for the nurse to review? a. LDL cholesterol b. Troponins T and I c. C-reactive protein d. Creatine kinase-MB (CK-MB)

Troponins T and I

What is an adequate intake of sodium for an adult aged 71 or older? 1. 1.2g 2. 1.3g 3. 1.5g 4. 2.3g

1

Which of the following actions should the nurse include in the plan of care for a client who is receiving sodium nitroprusside to treat a hypertensive emergency? a. Organize nursing activities so that the client has undisturbed sleep for 6 to 8 hours at night. b. Assist the client up in the chair for meals to avoid complications associated with immobility. c. Use an automated noninvasive blood pressure machine to obtain frequent BP measurements. d. Place the client on NPO status to prevent aspiration caused by nausea and the associated vomiting.

Use an automated noninvasive blood pressure machine to obtain frequent BP measurements.

To auscultate for S3 or S4 gallops in the mitral area, which of the following should the nurse implement? a. Use the bell of the stethoscope with the client in the left lateral position. b. Use the bell of the stethoscope with the client sitting and leaning forward. c. Use the diaphragm of the stethoscope with the client in a reclining position. d. Use the diaphragm of the stethoscope with the client lying flat on the left side.

Use the bell of the stethoscope with the client in the left lateral position.

Pink #2

Ventilator circuit

The patient donated a kidney, and early ambulation is included in her plan of care. But the patient refuses to get up and walk. What rationale should the nurse explain to the patient for early ambulation? a. "Early walking keeps your legs limber and strong." b. "Early ambulation will help you be ready to go home." c. "Early ambulation will help you get rid of your syncope and pain." d. "Early walking is the best way to prevent postoperative complications."

d. "Early walking is the best way to prevent postoperative complications." The best rationale is that early ambulation will prevent postoperative complications that can then be discussed. Ambulating increases muscle tone, stimulates circulation that prevents venous stasis and VTE, speeds wound healing, and increases vital capacity and maintains normal respiratory function. These things help the patient be ready for discharge, but early ambulation does not eliminate syncope and pain. Pain management should always occur before walking.

16. The nurse has been teaching a patient with type 2 diabetes about managing blood glucose levels and taking glipizide (Glucotrol). Which patient statement indicates a need for additional teaching? a. "If I overeat at a meal, I will still take the usual dose of medication." b. "Other medications besides the Glucotrol may affect my blood sugar." c. "When I am ill, I may have to take insulin to control my blood sugar." d. "My diabetes won't cause complications because I don't need insulin."

d. "My diabetes won't cause complications because I don't need insulin." - The patient should understand that type 2 diabetes places the patient at risk for many complications and that good glucose control is as important when taking oral agents as when using insulin. The other statements are accurate and indicate good understanding of the use of glipizide.

When planning the care of a patient with dehydration, what would the nurse instruct the unlicensed assistive personnel (UAP) to report? a. 60 mL urine output in 90 minutes b. 1200 mL urine output in 24 hours c. 300 mL urine output per 8-hour shift d. 20 mL urine output for 2 consecutive hours

d. 20 mL urine output for 2 consecutive hours The minimal urine output necessary to maintain kidney function is 30 mL/hr. If the output is less than this for 2 consecutive hours, the nurse should be notified so that additional fluid volume replacement therapy can be instituted.

An older patient who had surgery is displaying manifestations of delirium. What should the nurse do first to provide the best care for this new patient? a. Check his chart for intraoperative complications. b. Check which medications were used for anesthesia. c. Check the effectiveness of the analgesics he has received. d. Check his preoperative assessment for previous delirium or dementia.

d. Check his preoperative assessment for previous delirium or dementia. If the patient's ABCs are okay, it is important to first know if the patient was mentally alert without cognitive impairments before surgery. Then intraoperative complications, anesthesia medications, and pain will be assessed as these can all contribute to delirium.

The nurse working on a medical unit recognizes that which of the following individuals are a risk for hyponatremia? Select all that apply. 1. A 19-year-old drowning victim rescued from a nearby lake. 2. A 52-year-old with congestive heart failure taking diuretics who is NPO for a cardiac catheterization. 3. A 68-year-old with bowel obstruction receiving nasogastric suction 4. A 92-year-old who is receiving total parenteral nutrition 5. A 55-year-old who takes calcium supplements for osteoporosis 6. A 42-year-old with chronic renal failure.

1, 2, 3, 4

The nurse working on a medical unit recognizes that which of the following individuals are a risk for hyponatremia? Select all that apply. 1. A 19-year-old drowning victim rescued from a nearby lake. 2. A 52-year-old with congestive heart failure taking diuretics who is NPO for a cardiac catheterization. 3. A 68-year-old with bowel obstruction receiving nasogastric suction 4. A 92-year-old who is receiving total parenteral nutrition 5. A 55-year-old who takes calcium supplements for osteoporosis 6. A 42-year-old with chronic renal failure.

1, 2, 3, 4 In drowning victims, large amounts of water are often taken in (swallowed and/or aspirated), which may result in hyponatremia. Patients who are NPO, receiving gastric suctioning, or on TPN are at risk for hyponatremia.

The nurse is caring for a patient with suspected hypokalemia. Which of the following are signs and/or symptoms for which the nurse should be vigilant? Select all that apply. 1. Weak thready pulse 2. Shallow breathing 3. Increased gastrointestinal motility 4. Muscle weakness 5. Nausea 6. Pinpoint pupils

1, 2, 4, 5

The nurse is caring for a patient with suspected hypokalemia. Which of the following are signs and/or symptoms for which the nurse should be vigilant? Select all that apply. 1. Weak thready pulse 2. Shallow breathing 3. Increased gastrointestinal motility 4. Muscle weakness 5. Nausea 6. Pinpoint pupils

1, 2, 4, 5 A weak, thready pulse may be caused by cardiac dysrhythmias, a common symptom of hypokalemia. Shallow breathing, muscle weakness, and nausea are also symptoms of hypokalemia. Increased GI motility (abdominal cramping, diarrhea, vomiting) is caused by hyperkalemia. Pinpoint pupils are a sign of depressant toxicity, not electrolyte imbalance.

The most reliable indicator of fluid loss or gain is body __________.

Weight

The patient had surgery at an ambulatory surgery center. Which criteria support that this patient is ready for discharge (select all that apply)? a. Vital signs baseline or stable b. Minimal nausea and vomiting c. Wants to go to the bathroom at home d. Responsible adult taking patient home e. Comfortable after IV opioid 15 minutes ago

a, b, & d Ambulatory surgery discharge criteria includes meeting Phase I PACU discharge criteria that includes vital signs baseline or stable and minimal nausea and vomiting. Phase II criteria includes a responsible adult driving patient, no IV opioid drugs for last 30 minutes, able to void, able to ambulate if not contraindicated, and received written discharge instruction with patient understanding confirmed.

An older woman was admitted to the medical unit with dehydration. Clinical indications of this problem are (select all that apply): a. weight loss b. dry oral mucosa c. full bounding pulse d. engorged neck veins e. decreased central venous pressure

a, b, & e Rationale: Body weight loss, especially sudden change, is an excellent indicator of overall fluid volume loss. Other clinical manifestations of dehydration include dry mucous membranes and a decreased central venous pressure, which reflect fluid volume loss.

The nurse is caring for a patient who is very anxious and hyperventilating. Which of the following are signs and symptoms of respiratory alkalosis for which to observe? Select all that apply. 1. Increased heart rate 2. Increased PaCO2 level 3. Slow deeps respirations 4. Rapid shallow respirations 5. Decreased heart rate 6. Lightheadedness

1, 4, 6

Which of the following intravenous solutions is isotonic? (select all that apply) 1. 0.9% saline 2. 5% dextrose in water (D5W) 3. 0.45% NaCl 4. Dextrose in 0.255% NaCl 5. Lactated Ringers (LR)

1, 5 normal saline (0.9% saline) and lactated ringers (LR) are isotonic solutions. D5W is isotonic at first, but becomes hypotonic when the dextrose is metabolized 1/2 NS (0.45% NaCl) and Dextrose in 0.255% NaCl (D1/2NS) are both hypotonic.

Which of the floowing intravenous solutions is isotonic? (select all that apply) 1. 0.9% saline 2. 5% dextrose in water (D5W) 3. 0.45% NaCl 4. Dextrose in 0.255% NaCl

1. 0.9% saline 4. Dextrose in 0.225% Salien

What is an adequate intake of sodium for an adult aged 71 or older? 1. 1.2g 2. 1.3g 3. 1.5g 4. 2.3g

1. 1.2g

Light Blue

15-mm adapter

The dehydrated patient is receiving a hypertonic solution. What assessments must be done to avoid risk factors of these solutions (select all that apply)? a. Lung sounds b. Bowel sounds c. Blood pressure d. Serum sodium level e. Serum potassium level

a, c, & d BP, lung sounds, and serum sodium levels must be monitored frequently because of the risk for excess intravascular volume with hypertonic solutions.

The nurse has been invited to discuss "the importance of promoting a good fluid and electrolyte balance in children" for a group of parents at the local school parents club meeting. Of the following actions, which is NOT representative of this topic? a. Recognizing possible risk factors for fluid and electrolyte balance, such as prolonged or repeated vomiting, frequent watery stools, or inability to consume fluids b. Increasing fluid intake before, during, and after strenuous exercise, particularly when the environmental temperature is high, and replacing lost electrolytes from excessive perspiration as needed with commercial electrolyte solutions c. Consuming six to eight glasses of water daily d. Encouraging excessive amounts of foods or fluids high in salt or caffeine

d. Encouraging excessive amounts of foods or fluids high in salt or caffeine Rationale: Salt causes the body to retain fluids due to an increase in the concentration of sodium and the release of ADH. Caffeine acts as a diuretic in individuals and may lead to loss of excess fluids in the body. The remaining identified measures are all appropriate.

When planning care for a patient with dehydration related to nausea and vomiting, the nurse would anticipate which fluid shift to occur because of the fluid volume deficit? a. Fluid movement from the blood vessels into the cells b. Fluid movement from the interstitial spaces into the cells c. Fluid movement from the blood vessels into interstitial spaces d. Fluid movement from the interstitial space into the blood vessels

d. Fluid movement from the interstitial space into the blood vessels In dehydration, fluid is lost first from the blood vessels. To compensate, fluid moves out of the interstitial spaces into the blood vessels to restore circulating volume in that compartment. As the interstitial spaces then become volume depleted, fluid moves out of the cells into the interstitial spaces.

When assessing a patient admitted with nausea and vomiting, which finding supports the nursing diagnosis of deficient fluid volume? a. Polyuria b. Decreased pulse c. Difficulty breathing d. General restlessness

d. General restlessness Restlessness is an early cerebral sign that dehydration has progressed to the point where an intracellular fluid shift is occurring. If the dehydration is left untreated, cerebral signs could progress to confusion and later coma.

A patient is being treated for hypokalemia. When evaluating his response to potassium replacement therpy, which of the following changes in his assessment should the nurse observe for? a. Improving visual acuity b. Worsening constipation c. Decreasing serum glucose d. Increasing muscle strength

d. Hypokalemia is associated with muscle weakness.

The nurse is caring for an older adult patient who presents to the emergency room complaining of severe vomiting and diarrhea, sweating, and rapid heartbeat. The body temperature is normal. Which of the following assessments should the nurse complete next? 1. Evaluate the presence of leg edema 2. Check skin turgor 3. Listen for crackles 4. Assess capillary refill

2

What percentage of an older adult's body weight is water? 1. 30% 2. 50% 3. 60% 4. 70%

2 Older adults have a body weight that is 45-55% water Normal adults: 50-60% Infants: 70%

You are caring for an older patient who is receiving IV fluids postoperatively. During the 8:00 AM assessment of this patient, you note that the IV solution, which was ordered to infuse at 125 mL/hr, has infused 950 mL since it was hung at 4:00 AM. What is the priority nursing intervention? a. Notify the physician and complete an incident report. b. Slow the rate to keep vein open until next bag is due at noon. c. Obtain a new bag of IV solution to maintain patency of the site. d. Listen to the patient's lung sounds and assess respiratory status.

d. Listen to the patient's lung sounds and assess respiratory status. After 4 hours of infusion time, 500 mL of IV solution should have infused, not 950 mL. This patient is at risk for fluid volume excess, and you should assess the patient's respiratory status and lung sounds as the priority action and then notify the physician for further orders.

You are caring for a patient admitted with diabetes mellitus, malnutrition, and massive GI bleed. In analyzing the morning lab results, the nurse understands that a potassium level of 5.5 mEq/L could be caused by which factors in this patient (select all that apply)? a. The potassium level may be increased if the patient has renal nephropathy. b. The patient may be excreting extra sodium and retaining potassium because of malnutrition. c. The potassium level may be increased as a result of dehydration that accompanies high blood glucose levels. d. There may be excess potassium being released into the blood as a result of massive transfusion of stored hemolyzed blood. e. The patient has been overeating raisins, baked beans, and salt substitute that increase the potassium level.

a, c, & d Hyperkalemia may result from hyperglycemia, renal insufficiency, and/or cell death. Diabetes mellitus, along with the stress of hospitalization and illness, can lead to hyperglycemia. Renal insufficiency is a complication of diabetes. Malnutrition does not cause sodium excretion accompanied by potassium retention. Thus it is not a contributing factor to this patient's potassium level. Stored hemolyzed blood can cause hyperkalemia when large amounts are transfused rapidly. The patient with a massive GI bleed would have an NG tube and not be eating.

It is especially important for the nurse to assess for which clinical manifestation(s) in a patient who has just undergone a total thyroidectomy (select all that apply)? a. Confusion b, Weight gain c. Depressed reflexes d. Circumoral numbness e. Positive Chxostek's sign

a, d, & e Rationale: Inadvertent removal of a portion of or injury to the parathyroid glands during thyroid or neck surgery can result in a lack of parathyroid hormone, leading to hypocalcemia. A positive Chvostek sign, confusion, and circumoral numbness are manifestations of low serum calcium levels.

Which of the following are functions of sodium in the body? Select all that apply. a. Maintenance of serum osmolarity b. Formation of bones and teeth c. Control of bronchodilation d. Control of serum glucose e. Maintenance of cellular function

a, e Sodium is the major cation in the blood and helps maintain serum osmolarity. Sodium is also important for cell function, especially in the central nervous system.

25. Which question during the assessment of a patient who has diabetes will help the nurse identify autonomic neuropathy? a. "Do you feel bloated after eating?" b. "Have you seen any skin changes?" c. "Do you need to increase your insulin dosage when you are stressed?" d. "Have you noticed any painful new ulcerations or sores on your feet?"

a. "Do you feel bloated after eating?" - Autonomic neuropathy can cause delayed gastric emptying, which results in a bloated feeling for the patient. The other questions are also appropriate to ask but would not help in identifying autonomic neuropathy

10. Which statement by the patient indicates a need for additional instruction in administering insulin? a. "I need to rotate injection sites among my arms, legs, and abdomen each day." b. "I can buy the 0.5-mL syringes because the line markings will be easier to see." c. "I do not need to aspirate the plunger to check for blood before injecting insulin." d. "I should draw up the regular insulin first, after injecting air into the NPH bottle."

a. "I need to rotate injection sites among my arms, legs, and abdomen each day." - Rotating sites is no longer recommended because there is more consistent insulin absorption when the same site is used consistently. The other patient statements are accurate and indicate that no additional instruction is needed.

12. A patient receives aspart (NovoLog) insulin at 8:00 AM. At which time would the nurse anticipate the highest risk for hypoglycemia? a. 10:00 AM b. 12:00 AM c. 2:00 PM d. 4:0 PM

a. 10:00 AM - The rapid-acting insulins peak in 1 to 3 hours. The patient is not at a high risk for hypoglycemia at the other listed times, although hypoglycemia may occur.

Which serum potassium result best supports the rationale for administering a stat dose of potassium chloride 20 mEq in 250 mL of normal saline over 2 hours? a. 3.1 mEq/L b. 3.9 mEq/L c. 4.6 mEq/L d. 5.3 mEq/L

a. 3.1 mEq/L The normal range for serum potassium is 3.5 to 5.0 mEq/L. This IV order provides a substantial amount of potassium. Thus the patient's potassium level must be low. The only low value shown is 3.1 mEq/L.

Which pH value respresents acidosis? a. 7.26 b. 7.35 c. 7.4 d. 7.49

a. 7.26 is below 7.35 and is acidotic.

When assessing the patient with a multi-lumen central line, the nurse notices that the cap is off one of the lines. On assessment, the patient is in respiratory distress, and the vital signs show hypotension and tachycardia. What is the nurse's priority action? a. Administer oxygen. b. Notify the physician. c. Rapidly administer more IV fluid. d. Reposition the patient to the right side.

a. Administer oxygen. The cap off the central line could allow entry of air into the circulation. For an air emboli, oxygen is administered; the catheter is clamped; the patient is positioned on the left side with the head down. Then the physician is notified.

In planning postoperative interventions to promote repositioning, ambulation, coughing, and deep breathing, which action should the nurse recognize will best enable the patient to achieve the desired outcomes? a. Administering adequate analgesics to promote relief or control of pain b. Asking the patient to demonstrate the postoperative exercises every 1 hour c. Giving the patient positive feedback when the activities are performed correctly d. Warning the patient about possible complications if the activities are not performed

a. Administering adequate analgesics to promote relief or control of pain Even when a patient understands the importance of postoperative activities and demonstrates them correctly, it is unlikely that the best outcome will occur unless the patient has sufficient pain relief to cooperate with the activities.

The nurse is caring for a client with hypertension and has a prescription for nadolol. Which of the following assessment findings should the nurse report to the health care provider before administering this medication? a. Asthma b. Peptic ulcer disease c. Alcohol dependency d. Myocardial infarction

a. Asthma

Bronchial obstruction by retained secretions has contributed to a postoperative patient's recent pulse oximetry reading of 87%. Which health problem is the patient probably experiencing? a. Atelectasis b. Bronchospasm c. Hypoventilation d. Pulmonary embolism

a. Atelectasis The most common cause of postoperative hypoxemia is atelectasis, which may be the result of bronchial obstruction caused by retained secretions or decreased respiratory excursion. Bronchospasm involves the closure of small airways by increased muscle tone, whereas hypoventilation is marked by an inadequate respiratory rate or depth. Pulmonary emboli do not involve blockage by retained secretions.

26. Which information will the nurse include in teaching a female patient who has peripheral arterial disease, type 2 diabetes, and sensory neuropathy of the feet and legs? a. Choose flat-soled leather shoes. b. Set heating pads on a low temperature. c. Use callus remover for corns or calluses. d. Soak feet in warm water for an hour each day.

a. Choose flat-soled leather shoes. - The patient is taught to avoid high heels and that leather shoes are preferred. The feet should be washed, but not soaked, in warm water daily. Heating pad use should be avoided. Commercial callus and corn removers should be avoided. The patient should see a specialist to treat these problems.

A patient is being discharged following hospitalization for fluid imbalance. Which instruction by the nurse should take priority? a. "Weigh yourself at the same time every day and report changes." b. "Call your doctor immediately if you feel weak or fatigued." c. "Drink eight glasses of water a day." d. "Measure everything you drink, and measure how much you urinate each day."

a. Daily or every-other-day weights are easy to keep track of at home.

Measurements related to fluid balance of clients that a nurse can initiate without a physician's order include: a. Daily weights, vital signs, and fluid intake and output b. Daily weights, diuretics, and waist measurement c. Monitoring temperature, fluid intake and output, and calorie count d. Auscultating lung sounds, monitoring color of urine, and placing a Foley catheter into the client

a. Daily weights, vital signs, and fluid intake and output Rationale: Daily weights, checking vital signs, and monitoring fluid I&O all fall within the realm of nursing interventions. The remaining interventions either have the nurse perform a task requiring an MD order, such as giving diuretics or placing a Foley catheter, or have an action unrelated to this problem, such as the calorie count.

9. To assist an older patient with diabetes to engage in moderate daily exercise, which action is most important for the nurse to take? a. Determine what types of activities the patient enjoys. b. Remind the patient that exercise improves self-esteem. c. Teach the patient about the effects of exercise on glucose level. d. Give the patient a list of activities that are moderate in intensity

a. Determine what types of activities the patient enjoys.

When caring for a patient with fluid excess, which of the following interventions will best help relieve respiratory distress? a. Elevate the head of the bed b. Encourage the patient to cough and deep breathe. c. Increase fluids to promote urine output. d. Perform percussion and postural drainage.

a. Elevating the head of the bed will provide more room for lung expansion and provide the quickest relief for shortness of breath.

44. The nurse has administered 4 oz of orange juice to an alert patient whose blood glucose was 62 mg/dL. Fifteen minutes later, the blood glucose is 67 mg/dL. Which action should the nurse take next? a. Give the patient 4 to 6 oz more orange juice. b. Administer the PRN glucagon (Glucagon) 1 mg IM. c. Have the patient eat some peanut butter with crackers. d. Notify the health care provider about the hypoglycemia.

a. Give the patient 4 to 6 oz more orange juice. - The "rule of 15" indicates that administration of quickly acting carbohydrates should be done two or three times for a conscious patient whose glucose remains less than 70 mg/dL before notifying the health care provider. More complex carbohydrates and fats may be used after the glucose has stabilized. Glucagon should be used if the patient's level of consciousness decreases so that oral carbohydrates can no longer be given.

35. A patient with diabetic ketoacidosis is brought to the emergency department. Which prescribed action should the nurse implement first? a. Infuse 1 L of normal saline per hour. b. Give sodium bicarbonate 50 mEq IV push. c. Administer regular insulin 10 U by IV push. d. Start a regular insulin infusion at 0.1 units/kg/hr.

a. Infuse 1 L of normal saline per hour. - The most urgent patient problem is the hypovolemia associated with diabetic ketoacidosis (DKA), and the priority is to infuse IV fluids. The other actions can be done after the infusion of normal saline is initiated.

14. A patient with diabetes is starting on intensive insulin therapy. Which type of insulin will the nurse discuss using for mealtime coverage? a. Lispro (Humalog) b. Glargine (Lantus) c. Detemir (Levemir) d. NPH (Humulin N)

a. Lispro (Humalog) - Rapid- or short-acting insulin is used for mealtime coverage for patients receiving intensive insulin therapy. NPH, glargine, or detemir will be used as the basal insulin.

45. Which nursing action can the nurse delegate to experienced unlicensed assistive personnel (UAP) who are working in the diabetic clinic? a. Measure the ankle-brachial index. b. Check for changes in skin pigmentation. c. Assess for unilateral or bilateral foot drop. d. Ask the patient about symptoms of depression.

a. Measure the ankle-brachial index. - Checking systolic pressure at the ankle and brachial areas and calculating the ankle-brachial index is a procedure that can be done by UAP who have been trained in the procedure. The other assessments require more education and critical thinking and should be done by the registered nurse (RN).

A patient is having elective cosmetic surgery performed on her face. The surgeon will keep her at the surgery center for 24 hours after surgery. What is the nurse's postoperative priority for this patient? a. Manage patient pain. b. Control the bleeding. c. Maintain fluid balance. d. Manage oxygenation status.

d. Manage oxygenation status. The nurse's priority is to manage the patient's oxygenation status by maintaining an airway and ventilation. With surgery on the face, there may be swelling that could compromise her ability to breathe. Pain, bleeding, and fluid imbalance from the surgery may increase her risk for upper airway edema causing airway obstruction and respiratory suppression, which also indicate managing oxygenation status as the priority.

After admission of the postoperative patient to the clinical unit, which assessment data require the most immediate attention? a. Oxygen saturation of 85% b. Respiratory rate of 13/min c. Temperature of 100.4F d. Blood pressure of 90/60 mmHg

a. Oxygen saturation of 85% Rationale: During the initial assessment, identify signs of inadequate oxygenation and ventilation. Pulse oximetry monitoring is initiated because it provides a noninvasive means of assessing the adequacy of oxygenation. Pulse oximetry may indicate low oxygen saturation (<90% to 92%) with respiratory compromise. This necessitates prompt intervention.

You are caring for a patient receiving calcium carbonate for the treatment of osteopenia. Which serum laboratory result would you identify as an adverse effect related to this therapy? a. Sodium falling to 138 mEq/L b. Potassium rising to 4.1 mEq/L c. Magnesium rising to 2.9 mg/dL d. Phosphorus falling to 2.1 mg/dL

d. Phosphorus falling to 2.1 mg/dL Calcium has an inverse relationship with phosphorus in the body. When phosphorus levels fall, calcium rises, and vice versa. Since hypercalcemia rarely occurs as a result of calcium intake, the patient's phosphorus falling to 2.1 mg/dL (normal 2.4-4.4 mg/dL) may be a result of the phosphate-binding effect of calcium carbonate.

34. A 27-yr-old patient admitted with diabetic ketoacidosis (DKA) has a serum glucose level of 732 mg/dL and serum potassium level of 3.1 mEq/L. Which action prescribed by the health care provider should the nurse take first? a. Place the patient on a cardiac monitor. b. Administer IV potassium supplements. c. Ask the patient about home insulin doses. d. Start an insulin infusion at 0.1 units/kg/hr.

a. Place the patient on a cardiac monitor. - Hypokalemia can lead to potentially fatal dysrhythmias such as ventricular tachycardia and ventricular fibrillation, which would be detected with electrocardiogram (ECG) monitoring. Because potassium must be infused over at least 1 hour, the nurse should initiate cardiac monitoring before infusion of potassium. Insulin should not be administered without cardiac monitoring because insulin infusion will further decrease potassium levels. Discussion of home insulin and possible causes can wait until the patient is stabilized.

Edema that forms in clients with kidney disease is due to: a. Reduced plasma oncotic pressure, so that fluid is not drawn into the capillaries from interstitial tissues b. Decreased capillary hydrostatic pressures pushing fluid into the interstitial tissues c. Capillaries becoming less permeable, allowing fluid to escape into interstitial tissues d. Obstructed lymph flow that assists the movement of fluid from the interstitial tissues back into the vascular compartment

a. Reduced plasma oncotic pressure, so that fluid is not drawn into the capillaries from interstitial tissues Rationale: The edema is due to low levels of plasma proteins that exist with this disease, altering the oncotic pressure that helps regulate fluid movement in the vascular space moving into interstitial area. Increased capillary hydrostatic pressure is the cause. Capillaries have increased permeability when edema formation is possible. Obstructed lymph flow impairs the movement of fluid from interstitial tissues back into the vascular compartment, resulting in edema.

The patient has chronic kidney disease and ate a lot of nuts, bananas, peanut butter, and chocolate. The patient is admitted with loss of deep tendon reflexes, somnolence, and altered respiratory status. What treatment should the nurse expect for this patient? a. Renal dialysis b. IV potassium chloride c. IV furosemide (Lasix) d. IV normal saline at 250 mL per hour

a. Renal dialysis Renal dialysis will need to be administered to remove the excess magnesium that is in the blood from the increased intake of foods high in magnesium. If renal function was adequate, IV potassium chloride would oppose the effects of magnesium on the cardiac muscle. IV furosemide and increased fluid would increase urinary output which is the major route of excretion for magnesium.

A client suffering from a narcotic overdose is seen in the Emergency Department. The client is confused, with warm, flushed skin, headache, and weakness. Vital signs of noted are T 102.6, HR 128, R 14, and BP 130/86. A blood gas analysis sample was drawn on room air, and the results are as follows: pH 7.35, PaCO2 53, PaO2 72, HCO3 24. This client is at risk for: a. Respiratory acidosis b. Respiratory alkalosis c. Metabolic acidosis d. Metabolic alkalosis

a. Respiratory acidosis Rationale: Narcotic overdose causes more carbonic acid levels to rise because of hypoventilation and carbon dioxode retention.

You receive a physician's order to change a patient's IV from D5½ NS with 40 mEq KCl/L to D5NS with 20 mEq KCl/L. Which serum laboratory values on this same patient best support the rationale for this IV order change? a. Sodium 136 mEq/L, potassium 4.5 mEq/L b. Sodium 145 mEq/L, potassium 4.8 mEq/L c. Sodium 135 mEq/L, potassium 3.6 mEq/L d. Sodium 144 mEq/L, potassium 3.7 mEq/L

a. Sodium 136 mEq/L, potassium 4.5 mEq/L The normal range for serum sodium is 135 to 145 mEq/L, and the normal range for potassium is 3.5 to 5.0 mEq/L. The change in the IV order decreases the amount of potassium and increases the amount of sodium. Therefore for this order to be appropriate, the potassium level must be near the high end and the sodium level near the low end of their respective ranges.

The nurse is caring for a client who is experiencing a hypertensive crisis and is receiving sodium nitroprusside. Which of the following time frequencies should the nurse assess the clients' blood pressure and pulse during the initial administration of this medication? a. 2-3 minutes b. 5-10 minutes c. 15-30 minutes d. Hourly

2-3 minutes

What percentage of an older adult's body weight is water? 1. 30% 2. 50% 3. 60% 4. 70%

2. 50%

The nurse is caring for an older adult patient who presents to the emergency room complaining of severe vomiting and diarrhea, sweating, and rapid heartbeat. The body temperature is normal. Which of the following assessments should the nurse complete next? 1. Evaluate the presence of leg edema 2. Check skin turgor 3. Listen for crackles 4. Assess capillary refill

2. Check skin turgor

What term is used to refer to the relationship between a 0.9% saline solution and the blood? 1. Hypotonic 2. Isotonic 3. Hypertonic

2. Isotonic

A positive Chvostek's sign and a positive Trousseau's sign are classic signs of hypocalcemia and of what other electrolyte imbalance? 1. Hypermagnesemia 2. Hyponatremia 3. Hypomagnesemia 4. Hypokalemia

3 Hypocalcemia, hypomagnesemia, and hyperphosphatemia all have similar symptoms: CATS (Cramping/Chvostek's, Arrhythmias, Tremors/Trousseau's, Stridor/Spasm)

13. Which patient action indicates a good understanding of the nurse's teaching about the use of an insulin pump? a. The patient programs the pump for an insulin bolus after eating. b. The patient changes the location of the insertion site every week. c. The patient takes the pump off at bedtime and starts it again each morning. d. The patient plans a diet with more calories than usual when using the pump.

a. The patient programs the pump for an insulin bolus after eating. - In addition to the basal rate of insulin infusion, the patient will adjust the pump to administer a bolus after each meal, with the dosage depending on the oral intake. The insertion site should be changed every 2 or 3 days. There is more flexibility in diet and exercise when an insulin pump is used, but it does not provide for consuming a higher calorie diet. The pump will deliver a basal insulin rate 24 hours a day.

37. A female patient is scheduled for an oral glucose tolerance test. Which information from the patient's health history is important for the nurse to communicate to the health care provider regarding this test? a. The patient uses oral contraceptives. b. The patient runs several days a week. c. The patient has been pregnant three times. d. The patient has a family history of diabetes.

a. The patient uses oral contraceptives. - Oral contraceptive use may falsely elevate oral glucose tolerance test (OGTT) values. Exercise and a family history of diabetes both can affect blood glucose but will not lead to misleading information from the OGTT. History of previous pregnancies may provide informational about gestational glucose tolerance but will not lead to misleading information from the OGTT.

6. The nurse is assessing a 55-yr-old female patient with type 2 diabetes who has a body mass index (BMI) of 31 kg/m2 .Which goal in the plan of care is most important for this patient? a. The patient will reach a glycosylated hemoglobin level of less than 7%. b. The patient will follow a diet and exercise plan that results in weight loss. c. The patient will choose a diet that distributes calories throughout the day. d. The patient will state the reasons for eliminating simple sugars in the diet.

a. The patient will reach a glycosylated hemoglobin level of less than 7%. - The complications of diabetes are related to elevated blood glucose and the most important patient outcome is the reduction of glucose to near-normal levels. A BMI of 30?9?kg/m2 or above is considered obese, so the other outcomes are appropriate but are not as high in priority.

While caring for a patient with metastatic bone cancer, which clinical manifestations would alert the nurse to the possibility of hypercalcemia in this patient? a. Weakness b. Paresthesia c. Facial spasms d. Muscle tremors

a. Weakness Signs of hypercalcemia are lethargy, headache, weakness, muscle flaccidity, heart block, anorexia, nausea, and vomiting. Paresthesia, facial spasms, and muscle tremors are symptoms of hypocalcemia.

Which patient is most at risk for fluid excess? a. An infant with pneumonia b. A teen with multiple injuries following and automobile accident. c. A middle-aged man who has just had surgery d. An elderly patient receiving IV therapy

d. The elderly have reduced kidney function and may not be able to handle excess fluids

7. A patient who has type 1 diabetes plans to swim laps for an hour daily at 1:00 PM. The clinic nurse will plan to teach the patient to a. check glucose level before, during, and after swimming. b. delay eating the noon meal until after the swimming class. c. increase the morning dose of neutral protamine Hagedorn (NPH) insulin. d. time the morning insulin injection so that the peak occurs while swimming.

a. check glucose level before, during, and after swimming. - The change in exercise will affect blood glucose, and the patient will need to monitor glucose carefully to determine the need for changes in diet and insulin administration. Because exercise tends to decrease blood glucose, patients are advised to eat before exercising. Increasing the morning NPH or timing the insulin to peak during exercise may lead to hypoglycemia, especially with the increased exercise.

The nurse is preparing to administer the discharge teaching instructions to a client with heart failure. Which measure will be included that the client will need to monitor on their own related to their recent hospitalization: _________________ . a. daily weight b. weekly electrolyte level c. daily fasting blood sugar d. daily urine dipstick for protein

a. daily weight Daily weights are one of the most important determinations of fluid balance. Weighing at home at the same time with the same amount of clothing on will track overall fluid changes as they occur. Acute weight gain or loss represents fluid gain or loss. The other measures would not be the responsibility of the client. Blood sugar measurement does not directly apply in this situation. .

The nursing care for a patient with hyponatremia includes: a. fluid restriction b. administration of hypotonic IV fluids c. administration of a cation-exchance resin d. increased water intake for patients on nasogastric suction

a. fluid restriction Rationale: In hyponatremia that is caused by water excess, fluid restriction often is all that is needed to treat the problem.

The lungs act as an acid-base buffer by: a. increasing respiratory rate and depth when CO2 levels in the blood are high, reducing acid load. b. increasing respiratory rate and depth when CO2 levels in the blood are low, reducing base load c. decreasing respiratory rate and depth when CO2 levels in the blood are high, reducing acid load. d. decreasing respiratory rate and depth when CO2 levels in the blood are low, increasing acid load.

a. increasing respiratory rate and depth when CO2 levels in the blood are high, reducing acid load. Rationale: As a compensatory mechanism, the respiratory system acts on the CO2 + H2O side of the reaction by altering the rate and depth of breathing to "blow off" (through hyperventilation) or "retain" (through hypoventilation) CO2.

During administration of a hypertonic IV solution, the mechanism involved in equalizing the fluid concentration between ECF and the cells is: a. osmosis b. diffusion c. active transport d. facilitated diffusion

a. osmosis Rationale: Osmosis is the movement of water between two compartments separated by a semipermeable membrane. Water moves through the membrane from an area of low solute concentration to an area of high solute concentration.

You are caring for a patient admitted with an exacerbation of asthma. After several treatments, the ABG results are pH 7.40, PaCO2 40 mm Hg, HCO3 24 mEq/L, PaO2 92 mm Hg, and O2 saturation of 99%. You interpret these results as a. within normal limits. b. slight metabolic acidosis. c. slight respiratory acidosis. d. slight respiratory alkalosis.

a. within normal limits. The normal pH is 7.35 to 7.45. Normal PaCO2 levels are 35 to 45 mm Hg, and HCO3 is 22 to 26 mEq/L. Normal PaO2 is >80 mm Hg. Normal oxygen saturation is >95%. Since the patient's results all fall within these normal ranges, the nurse can conclude that the patient's blood gas results are within normal limits.

32. The nurse is interviewing a new patient with diabetes who takes rosiglitazone (Avandia). Which information would the nurse anticipate resulting in the health care provider discontinuing the medication? a. The patient's blood pressure is 154/92. b. The patient's blood glucose is 86 mg/dL. c. The patient reports a history of emphysema. d. The patient has chest pressure when walking.

d. The patient has chest pressure when walking. - Rosiglitazone can cause myocardial ischemia. The nurse should immediately notify the health care provider and expect orders to discontinue the medication. A blood glucose level of 86 mg/dL indicates a positive effect from the medication. Hypertension and a history of emphysema do not contraindicate this medication.

Which patient is at risk for respiratory acidosis? a. The patient with uncontrolled diabetes mellitus b. The patient with chronic pulmonary disease c. The patient who is very anxious d. The patient who overuses antacids

b

A 93-year-old patient with diarrhea and dehydration is admitted to the hospital from an extended care facility. For which of the following symptoms of dehydration should the nurse assess? a. Pale-colored urine, bradycardia b. Disorientation, poor skin turgor c. Decreased hematocrit, hypothermia d. Lung congestion, abdominal discomfort

b Dehydration is associated with poor skin turgor because of loss of water in the tissues, and with disorientation because of loss of blood volume in the brain.

While performing patient teaching regarding hypercalcemia, which statements are appropriate (select all that apply)? a. Have patient restrict fluid intake to less than 2000 mL/day. b. Renal calculi may occur as a complication of hypercalcemia. c. Weight-bearing exercises can help keep calcium in the bones. d. The patient should increase daily fluid intake to 3000 to 4000 mL. e. Treatment of heartburn can best be managed with Tums as needed.

b, c, & d A daily fluid intake of 3000 to 4000 mL is necessary to enhance calcium excretion and prevent the formation of renal calculi, a potential complication of hypercalcemia. Tums are a calcium-based antacid that should not be used in patients with hypercalcemia. Weight-bearing exercise does enhance bone mineralization.

A patient is being placed on a potassium-losing diuretic. Which foods are high in potassium and should be recommended to the patient by the nurse? Select all that apply. a. Bread b. Potato c. Tomato juice d. Banana e. Gelatin

b, c, d Potatoes, tomatoes and bananas are highest in potassium.

3. A 28-yr-old male patient with type 1 diabetes reports how he manages his exercise and glucose control. Which behavior indicates that the nurse should implement additional teaching? a. The patient always carries hard candies when engaging in exercise. b. The patient goes for a vigorous walk when his glucose is 200 mg/dL. c. The patient has a peanut butter sandwich before going for a bicycle ride. d. The patient increases daily exercise when ketones are present in the urine.

d. The patient increases daily exercise when ketones are present in the urine. - When the patient is ketotic, exercise may result in an increase in blood glucose level. Patients with type 1 diabetes should be taught to avoid exercise when ketosis is present. The other statements are correct

8. The nurse determines a need for additional instruction when the patient with newly diagnosed type 1 diabetes says which of the following? a. "I will need a bedtime snack because I take an evening dose of NPH insulin." b. "I can choose any foods, as long as I use enough insulin to cover the calories." c. "I can have an occasional beverage with alcohol if I include it in my meal plan." d. "I will eat something at meal times to prevent hypoglycemia, even if I am not hungry."

b. "I can choose any foods, as long as I use enough insulin to cover the calories." - Most patients with type 1 diabetes need to plan diet choices very carefully. Patients who are using intensified insulin therapy have considerable flexibility in diet choices but still should restrict dietary intake of items such as fat, protein, and alcohol. The other patient statements are correct and indicate good understanding of the diet instruction.

Which question by a client hospitalized for a fluid volume deficit would require the nurse to investigate in more detail the probable cause of the dehydration? a. "Do I have to drink everything that is sent on my meal trays? I do not drink that much at home." b. "I have not had a bowel movement for 2 days. Can I get a laxative?" c. "I have tried to limit my sodium intake at home, but I am not very successful. Will I have to continue with this?" d. "I try to use the bathroom many times a day so that I will not have to get up through the night. Do you think this caused me to lose so much water?"

b. "I have not had a bowel movement for 2 days. Can I get a laxative?" Chronic abuse of laxatives and/or enemas may lead to fluid loss in a client. Elderly clients are most at risk for this as their overall fluid composition to total body weight has decreased. The frequent or dependent use of laxatives or enemas may not be readily reported on admission.

A client is brought to the emergency room following a motor vehicle accident. There is a moderate amount of blood loss. The physician has ordered 1000 ml of intravenous fluids to infuse over 4 hours. Which type of solution would be appropriate? Select all that apply. a. 5% dextrose with 0.45% sodium chloride b. 0.9% sodium chloride c. 5% dextrose in water d. Lactated Ringers

b. 0.9% sodium chloride d. Lactated Ringers Isotonic solutions (similar to normal body fluid) are used to expand plasma volume in hypotensive clients or to replace abnormal losses. Fluid loss due to injury will usually cause isotonic fluid loss. Expanding plasma volume will increase the overall circulating body volume. Lactated Ringers is an isotonic solution and is used when electrolyte replacement is also needed.

The patient was admitted for a paracentesis to remove ascites fluid. Five liters of fluid was removed. What IV solution may be used to pull fluid into the intravascular space after the paracentesis? a. 0.9% sodium chloride b. 25% albumin solution c. Lactated Ringer's solution d. 5% dextrose in 0.45% saline

b. 25% albumin solution After a paracentesis of 5 L or greater of ascites fluid, 25% albumin solution may be used as a volume expander. Normal saline, lactated Ringer's, and 5% dextrose in 0.45% saline will not be effective for this action.

47. After change-of-shift report, which patient should the nurse assess first? a. A 19-yr-old patient with type 1 diabetes who has a hemoglobin A1C of 12% b. A 23-yr-old patient with type 1 diabetes who has a blood glucose of 40 mg/dL c. A 40-yr-old patient who is pregnant and whose oral glucose tolerance test is 202 mg/dL d. A 50-yr-old patient who uses exenatide (Byetta) and is complaining of acute abdominal pain

b. A 23-yr-old patient with type 1 diabetes who has a blood glucose of 40 mg/dL - Because the brain requires glucose to function, untreated hypoglycemia can cause unconsciousness, seizures, and death. The nurse will rapidly assess and treat the patient with low blood glucose. The other patients also have symptoms that require assessments or interventions, but they are not at immediate risk for life-threatening complications.

Which of the following hospitalized clients would the nurse be most concerned is at risk for developing an imbalance related to water loss? a. A 50-year old undernourished female b. A 75-year old female of average body weight c. A 60-year old male of average body weight d. A 45-year old obese male

b. A 75-year old female of average body weight On average, females have a lower proportion of water to total body weight composition. After age 65, total body water may decrease 45-50% of the total body weight.

28. A patient who has diabetes and reported burning foot pain at night receives a new prescription. Which information should the nurse teach the patient about amitriptyline ? a. Amitriptyline decreases the depression caused by your foot pain. b. Amitriptyline helps prevent transmission of pain impulses to the brain. c. Amitriptyline corrects some of the blood vessel changes that cause pain. d. Amitriptyline improves sleep and makes you less aware of nighttime pain.

b. Amitriptyline helps prevent transmission of pain impulses to the brain. - Tricyclic antidepressants (TCAs) decrease the transmission of pain impulses to the spinal cord and brain. TCAs also improve sleep quality and are used for depression, but that is not the major purpose for their use in diabetic neuropathy. The blood vessel changes that contribute to neuropathy are not affected by TCAs.

A patient who weighs 250 pounds drinks eight 8-ounce glasses of water daily. Based on the 30 ml per kg rule, how many milliliters should the patient drink per day? (Round to the nearest whole number.) __________mL

3,409

A positive Chvostek's sign and a positive Trousseau's sign are classic signs of hypocalcemia and of what other electrolyte imbalance? 1. Hypermagnesemia 2. Hyponatremia 3. Hypomagnesemia 4. Hypokalemia

3. Hypomagnesemia

27. Which finding indicates a need to contact the health care provider before the nurse administers metformin (Glucophage)? a. The patient's blood glucose level is 174 mg/dL. b. The patient is scheduled for a chest x-ray in an hour. c. The patient has gained 2 lb (0.9 kg) in the past 24 hours. d. The patient's blood urea nitrogen (BUN) level is 52 mg/dL

d. The patient's blood urea nitrogen (BUN) level is 52 mg/dL - The BUN indicates possible renal failure, and metformin should not be used in patients with renal failure. The other findings are not contraindications to the use of metformin.

The nurse is evaluating the medication list of a newly-admitted client with hypokalemia. The client has been experiencing dysrhythmias and is on a cardiac monitor. Which medication from the list would alert the nurse to pursue a discussion with the physician? a. an oral antidiabetic agent b. a cardiac rhythm drug c. an over the counter antacid d. a diuretic

d. a diuretic Excess potassium loss through the kidneys is often secondary to drugs such as potassium-wasting diuretics. Hypokalemia may lead to cardiac dysrhythmias. Potassium-wasting diuretic therapy may require supplemental oral potassium.

10. A nurse believes that patients with the same type of tissue injury should have the same amount of pain. This statement reflects a. a belief that will contribute to appropriate pain management. b. an accurate statement about pain mechanisms and an expected goal of pain therapy. c. a belief that will have no effect on the type of care provided to people in pain. d. a lack of knowledge about pain mechanisms, which is likely to contribute to poor pain management.

d. a lack of knowledge about pain mechanisms, which is likely to contribute to poor pain management.

While assisting a new client from a wheelchair to a bed in the emergency room, the client complains of being dizzy. Which intervention by the nurse would be the best in this situation: ___________________ . a. assess peripheral pulses b. take an apical pulse c. assess for diuretic use d. assess blood pressure

d. assess blood pressure Postural or orthostatic hypotension is a sign of hypovolemia. A drop of more than 15 mmHg in systolic blood pressure when changing positions (lying to sitting, sitting to standing) often indicates fluid depletion. Assessing for diuretic use may yield information that contributes to the problem, but directly assessing the blood pressure is an immediate response to an exhibited symptom.

A 70-kg postoperative patient has an average urine output of 25 mL/hr during the first 8 hours. The priority nursing intervention(s) given this assessment would be to: a. perform a straight catheterization to measure the amount of urine in the bladder b. notify the physician and anticipate obtaining blood work to evaluate renal function c. continue to monitor the patient because this is a normal finding during this time period d. evaluate the patient's fluid volume status since surgery and obtain a bladder ultrasound

d. evaluate the patient's fluid volume status since surgery and obtain a bladder ultrasound Rationale: Because of the possibility of infection associated with catheterization, the nurse should first try to validate that the bladder is full. The nurse should consider fluid intake during and after surgery and should determine bladder fullness by percussion, by palpation, or by a portable bladder ultrasound study to assess the volume of urine in the bladder and avoid unnecessary catheterization.

The nurse is unable to flush a central venous access device and suspects occlusion. The best nursing intervention would be to: a. apply warm moist compresses to the insertion site b. attempt to force 10mL of normal saline into the device c. place the patient on the left side with head-down position d. instruct the patient to change positions, raise arm, and cough.

d. instruct the patient to change positions, raise arm, and cough. Rationale: Interventions for catheter occlusion include instructing the patient to change position, raise an arm, and cough; assessing for and alleviating clamping or kinking of the tube; flushing the catheter with normal saline through a 10-mL syringe (do not force flush); using fluoroscopy to determine cause and site of occlusion; and instilling anticoagulant or thrombolytic agents.

A patient has the following arterial blood gas results: pH 7.52; PaCO2 30 mmHg; HCO3- 24 mEq/L. The nurse determines that these results indicate: a. metabolic acidosis b. metabolic alkalosis c. respiratory acidosis d. respiratory alkalosis

d. respiratory alkalosis Rationale: Respiratory alkalosis (carbonic acid deficit) occurs with hyperventilation. The primary cause of respiratory alkalosis is hypoxemia from acute pulmonary disorders. Anxiety, central nervous system (CNS) disorders, and mechanical overventilation also increase ventilation rate and decrease the partial pressure of arterial carbon dioxide (PaCO2). This leads to a decrease in carbonic acid level and to alkalosis.

During the postoperative care of a 76-year-old patient, the nurse monitors the patient's intake and output carefully, knowing that the patient is at risk for fluid and electrolyte imbalances primarily because: a. older adults have an impaired thirst mechanism and need reminding to drink fluids b. water accounts for a greater percentage of body weight in the older adult than in younger adults c. older adults are more likely than younger adults to lose extracellular fluid during surgical procedures d. small losses of fluid are more significant because body fluids account for only about 50% of body weight in older adults

d. small losses of fluid are more significant because body fluids account for only about 50% of body weight in older adults Rationale: In the older adult, body water content averages 45% to 55% of body weight.

Which organ(s) is/are most at risk for dysfunction in a patient with a potassium level of 6.2 mEq/L? 1. Lungs 2. Kidneys 3. Liver 4. Heart

4 The most serious complications of potassium imbalance are cardiac dysrhythmias.

Which individual would least likely suffer from a disturbance in fluid volume, electrolyte, or acid-base balance? a. An infant suffering from gastroenteritis for three days b. An elderly client suffering from a type I decubitus ulcer c. Adults with impaired cardiac function d. Clients who are confused

b. An elderly client suffering from a stage I decubitus Rationale: The proportion of body water decreases with aging. Tissue trauma, such as burns, causes fluids and electrolytes to be lost from the damaged cells, and the breakdown in the continuity of the tissue. In a stage I Decubitus ulcer, the skin remains intact, and any shifting of fluids is due to the inflammatory process and internally maintained within the body. Vomiting and diarrhea can cause significant fluid loses. Age, sex, and body fat affect total body water. Infants have the delete spaces highest proportion of water; it accounts for 70-80% of their body weight. Decreased blood flow to the kidneys as caused by impaired cardiac function stimulates the renin-angiotensin-aldosterone system, causing sodium and water retention. Clients who are confused or unable to communicate are at risk for inadequate fluid intake. Age does not play a significant factor here.

20. The nurse is preparing to teach a 43-yr-old man who is newly diagnosed with type 2 diabetes about home management of the disease. Which action should the nurse take first? a. Ask the patient's family to participate in the diabetes education program. b. Assess the patient's perception of what it means to have diabetes mellitus. c. Demonstrate how to check glucose using capillary blood glucose monitoring. d. Discuss the need for the patient to actively participate in diabetes management.

b. Assess the patient's perception of what it means to have diabetes mellitus. - Before planning teaching, the nurse should assess the patient's interest in and ability to self-manage the diabetes. After assessing the patient, the other nursing actions may be appropriate, but planning needs to be individualized to each patient.

The nurse is working on a surgical floor and is preparing to receive a postoperative patient from the postanesthesia care unit (PACU). What should the nurse's initial action be upon the patient's arrival? a. Assess the patient's pain. b. Assess the patient's vital signs. c. Check the rate of the IV infusion. d. Check the physician's postoperative orders.

b. Assess the patient's vital signs. The highest priority action by the nurse is to assess the physiologic stability of the patient. This is accomplished in part by taking the patient's vital signs. The other actions can then take place in rapid sequence.

40. An active 32-yr-old male who has type 1 diabetes is being seen in the endocrine clinic. Which finding indicates a need for the nurse to discuss a possible a change in therapy with the health care provider? a. Hemoglobin A1C level of 6.2% b. Blood pressure of 140/88 mmHg c. Heart rate at rest of 58 beats/minute d. High density lipoprotein (HDL) level of 65 mg/dL

b. Blood pressure of 140/88 mmHg - To decrease the incidence of macrovascular and microvascular problems in patients with diabetes, the goal blood pressure is usually 130/80 mm Hg. An A1C less than 6.5%, a low resting heart rate (consistent with regular aerobic exercise in a young adult), and an HDL level of 65 mg/dL all indicate that the patient's diabetes and risk factors for vascular disease are well controlled.

While orienting a new nurse to work in the surgical intensive care unit, the charge nurse quizzes the new nurse as to which of the following postoperative clients would be at highest risk for problems related to excess fluids? Select all that apply. a. Client with diabetes insipidus b. Client with heart failure c. Client with systemic lupus erythematous d. Client with liver cirrhosis

b. Client with heart failure d. Client with liver cirrhosis With fluid volume excess, water and sodium are gained together. This creates a total water increase in the body. A client with a heart that is not able to pump this excess fluid and a liver that is not functioning appropriately will have potential problems. Clients with diabetes insipidus and systemic lupus erythematous are not considered to be at general risk for fluid volume excess. .

24. Which action should the nurse take after a patient treated with intramuscular glucagon for hypoglycemia regains consciousness? a. Assess the patient for symptoms of hyperglycemia. b. Give the patient a snack of peanut butter and crackers. c. Have the patient drink a glass of orange juice or nonfat milk. d. Administer a continuous infusion of 5% dextrose for 24 hours.

b. Give the patient a snack of peanut butter and crackers. - Rebound hypoglycemia can occur after glucagon administration, but having a meal containing complex carbohydrates plus protein and fat will help prevent hypoglycemia. Orange juice and nonfat milk will elevate blood glucose rapidly, but the cheese and crackers will stabilize blood glucose. Administration of IV glucose might be used in patients who were unable to take in nutrition orally. The patient should be assessed for symptoms of hypoglycemia after glucagon administration.

15. Which information will the nurse include when teaching a patient who has type 2 diabetes about glyburide ? a. Glyburide decreases glucagon secretion from the pancreas. b. Glyburide stimulates insulin production and release from the pancreas. c. Glyburide should be taken even if the morning blood glucose level is low. d. Glyburide should not be used for 48 hours after receiving IV contrast media.

b. Glyburide stimulates insulin production and release from the pancreas.

An elderly patient was hydrated with lactated Ringer's solution in the emergency room for the last hour. During the most recent evaluation of the patient by the nurse, a finding of a rapid bounding pulse and shortness of breath were noted. Reporting this episode to the physician, the nurse suspects that the patient now shows signs of: a. Hypovolemia, and needs more fluids b. Hypervolemia, and needs the fluids adjusted c. An acid-base disturbance d. Needing no adjustment in fluid administration

b. Hypervolemia, and needs the fluids adjusted Rationale: Isotonic solutions has the same osmolality as body fluids. Isotonic solutions, such as Normal Saline and Ringer's Lactate, initially remain in the vascular compartment, expanding vascular volume. Isotonic imbalances occur when water and electrolytes are lost or gained in equal proportions, and serum osmolality remains constant.

The patient is admitted with metabolic acidosis. Which system is not functioning normally? a. Buffer system b. Kidney system c. Hormone system d. Respiratory system

b. Kidney system When the patient has metabolic acidosis, the kidneys are not combining H+ with ammonia to form ammonium or eliminating acid with secretion of free hydrogen into the renal tubule. The buffer system neutralizes hydrochloric acid by forming a weak acid. The hormone system is not directly related to acid-base balance. The respiratory system releases CO2 that combines with water to form hydrogen ions and bicarbonate. The hydrogen is then buffered by the hemoglobin.

Unless contraindicated by the surgical procedure, which position is preferred for the unconscious patient immediately postoperative? a. Supine b. Lateral c. Semi-Fowler's d. High-Fowler's

b. Lateral Unless contraindicated by the surgical procedure, the unconscious patient is positioned in a lateral "recovery" position. This recovery position keeps the airway open and reduces the risk of aspiration if the patient vomits. Once conscious, the patient is usually returned to a supine position with the head of the bed elevated.

You are caring for a patient admitted with heart failure. The morning laboratory results reveal a serum potassium level of 2.9 mEq/L. Which classification of medications should you withhold until consulting with the physician? a. Antibiotics b. Loop diuretics c. Bronchodilators d. Antihypertensives

b. Loop diuretics Loop diuretics are contraindicated during episodes of hypokalemia because these medications cause the kidneys to excrete sodium and potassium. Thus administration of this type of medication at this time would worsen the hypokalemia, putting the patient at risk for dysrhythmias. The prescribing physician should be consulted for potassium replacement therapy, and the drug should be withheld until the potassium has returned to normal range.

The ambulance arrives with a client who presents with Kussmaul's respirations. The client has a history of diabetes. The nurse does a fingerstick blood sugar test immediately. The nurse is anticipating results that will lead to the diagnosis of which acid-base imbalance: a. Respiratory acidosis. b. Metabolic acidosis. c. Respiratory alkalosis. d. Metabolic alkalosis.

b. Metabolic acidosis. Kussmaul's respirations are deep and rapid respirations that are a compensatory mechanism by the respiratory system to return the blood pH to normal by eliminating carbon dioxide. This occurs when the body is in a metabolic acidotic state. The nurse expects the blood sugar to be very high. When the blood sugar of a diabetic client is very high, diabetic ketoacidosis may result. This is a type of metabolic acidosis. Clients with a history of diabetes are not prone to respiratory imbalances.

You are admitting a patient with complaints of abdominal pain, nausea, and vomiting. A bowel obstruction is suspected. You assess this patient for which anticipated primary acid-base imbalance if the obstruction is high in the intestine? a. Metabolic acidosis b. Metabolic alkalosis c. Respiratory acidosis d. Respiratory alkalosis

b. Metabolic alkalosis Because gastric secretions are rich in hydrochloric acid, the patient who is vomiting will lose a significant amount of gastric acid and be at an increased risk for metabolic alkalosis.

Which nursing intervention is most appropriate when caring for a patient with dehydration? a. Auscultate lung sounds every 2 hours. b. Monitor daily weight and intake and output. c. Monitor diastolic blood pressure for increases. d. Encourage the patient to reduce sodium intake.

b. Monitor daily weight and intake and output. Measuring weight is the most reliable means of detecting changes in fluid balance. Weight loss would indicate the dehydration is worsening, whereas weight gain would indicate restoration of fluid volume. Recall that a 1-kg weight gain indicates a gain of approximately 1000 mL of body water.

38. Which laboratory value reported to the nurse by the unlicensed assistive personnel (UAP) indicates an urgent need for the nurse's assessment of the patient? a. Bedtime glucose of 140 mg/dL b. Noon blood glucose of 52 mg/dL c. Fasting blood glucose of 130 mg/dL d. 2-hr postprandial glucose of 220 mg/dL

b. Noon blood glucose of 52 mg/dL - The nurse should assess the patient with a blood glucose level of 52 mg/dL for symptoms of hypoglycemia and give the patient a carbohydrate-containing beverage such as orange juice. The other values are within an acceptable range or not immediately dangerous for a patient with diabetes

You are caring for a patient admitted with a diagnosis of chronic obstructive pulmonary disease (COPD) who has the following arterial blood gas results: pH 7.33, PaO2 47 mm Hg, PaCO2 60 mm Hg, HCO3 32 mEq/L, and O2 saturation of 92%. What is the correct interpretation of these results? a. Fully compensated respiratory alkalosis b. Partially compensated respiratory acidosis c. Normal acid-base balance with hypoxemia d. Normal acid-base balance with hypercapnia

b. Partially compensated respiratory acidosis A low pH (normal 7.35-7.45) indicates acidosis. In the patient with respiratory disease such as COPD, the patient retains carbon dioxide (normal 35-45 mm Hg), which acts as an acid in the body. For this reason, the patient has respiratory acidosis. The elevated HCO3 indicates a partial compensation for the elevated CO2.

The patient had abdominal surgery. The estimated blood loss was 400 mL. The patient received 300 mL of 0.9% saline during surgery. Postoperatively, the patient is hypotensive. What should the nurse anticipate for this patient? a. Blood administration b. Restoring circulating volume c. An ECG to check circulatory status d. Return to surgery to check for internal bleeding

b. Restoring circulating volume The nurse should anticipate restoring circulating volume with IV infusion. Although blood could be used to restore circulating volume, there are no manifestations in this patient indicating a need for blood administration. An ECG may be done if there is no response to the fluid administration, or there is a past history of cardiac disease, or cardiac problems were noted during surgery. Returning to surgery to check for internal bleeding would only be done if patient's level of consciousness changes or the abdomen becomes firm and distended.

33. The nurse is taking a health history from a 29-yr-old pregnant patient at the first prenatal visit. The patient reports that she has no personal history of diabetes, but her mother has diabetes. Which action will the nurse plan to take? a. Teach the patient about administering regular insulin. b. Schedule the patient for a fasting blood glucose level. c. Teach about an increased risk for fetal problems with gestational diabetes. d. Schedule an oral glucose tolerance test for the twenty-fourth week of pregnancy.

b. Schedule the patient for a fasting blood glucose level.

The client with a history of chronic obstructive pulmonary disease has 10 liters of oxygen per nasal cannula applied by the student nurse for complaints of shortness of breath. The nursing instructor reduces this to 2 liters after reading the student's documentation. What is the rationale for this intervention? a. The student did not evaluate the client with the nursing instructor. b. The client's history prohibits the use of high levels of supplemental oxygen. c. The physician did not order the oxygen. d. The client needs to be evaluated by respiratory therapy before supplemental oxygen can be applied.

b. The client's history prohibits the use of high levels of supplemental oxygen. Clients with chronic lung disease may have consistently high carbon dioxide levels in their blood. Administering a high level of oxygen to these clients could inhibit their drive to breath as their carbon dioxide levels primarily determine respiratory response. ."

11. Which patient action indicates good understanding of the nurse's teaching about administration of aspart (NovoLog) insulin? a. The patient avoids injecting the insulin into the upper abdominal area. b. The patient cleans the skin with soap and water before insulin administration. c. The patient stores the insulin in the freezer after administering the prescribed dose. d. The patient pushes the plunger down while removing the syringe from the injection site.

b. The patient cleans the skin with soap and water before insulin administration. - Cleaning the skin with soap and water is acceptable. Insulin should not be frozen. The patient should leave the syringe in place for about 5 seconds after injection to be sure that all the insulin has been injected. The upper abdominal area is one of the preferred areas for insulin injection.

1. Pain is best described as a. a creation of a person's imagination. b. an unpleasant, subjective experience. c. a maladaptive response to a stimulus. d. a neurologic event resulting from activation of nociceptors.

b. an unpleasant, subjective experience.

41. A 30-yr-old patient has a new diagnosis of type 2 diabetes. The nurse will discuss the need to schedule a dilated eye examination a. every 2 years. b. as soon as possible. c. when the patient is 39 years old. d. within the first year after diagnosis.

b. as soon as possible. - Because many patients have some diabetic retinopathy when they are first diagnosed with type 2 diabetes, a dilated eye examination is recommended at the time of diagnosis and annually thereafter. Patients with type 1 diabetes should have dilated eye examinations starting 5 years after they are diagnosed and then annually.

The nurse anticipates treatment of the patient with hyperphosphatemia secondary to renal failure will include: a. fluid restriction b. calcium supplements c. loop diuretic therapy d. magnesium supplements

b. calcium supplements Rationale: The major conditions that can lead to hyperphosphatemia are acute kidney injury and chronic kidney disease that alter the ability of the kidneys to excrete phosphate. For the patient with renal failure, measures to reduce serum phosphate levels include calcium supplements, phosphate-binding agents or gels, fluid replacement therapy, and dietary phosphate restrictions.

19. The nurse identifies a need for additional teaching when the patient who is self-monitoring blood glucose a. washes the puncture site using warm water and soap. b. chooses a puncture site in the center of the finger pad. c. hangs the arm down for a minute before puncturing the site. d. says the result of 120 mg indicates good blood sugar control.

b. chooses a puncture site in the center of the finger pad. - The patient is taught to choose a puncture site at the side of the finger pad because there are fewer nerve endings along the side of the finger pad. The other patient actions indicate that teaching has been effective.

7. Appropriate nonopioid analgesics for mild pain include (select all that apply) a. oxycodone. b. ibuprofen (Advil). c. lorazepam (Ativan). d. acetaminophen (Tylenol). e. codeine with acetaminophen (Tylenol #3).

b. ibuprofen (Advil). d. acetaminophen (Tylenol).

21. An unresponsive patient with type 2 diabetes is brought to the emergency department and diagnosed with hyperosmolar hyperglycemic syndrome (HHS). The nurse will anticipate the need to a. give 50% dextrose. b. insert an IV catheter. c. initiate O2 by nasal cannula. d. administer glargine (Lantus) insulin.

b. insert an IV catheter. - HHS is initially treated with large volumes of IV fluids to correct hypovolemia. Regular insulin is administered, not a long-acting insulin. There is no indication that the patient requires O2. Dextrose solutions will increase the patient's blood glucose and would be contraindicated.

A client is admitted to the hospital for dehydration related to a gastrointestinal viral illness. The client is also on an alcohol withdrawal protocol. They complain of their lips and mouth feeling numb and tingling. When the nurse observes the lab results from the morning, which result would provide an insight into this client's complaint: a. low sodium level. b. low calcium level. c. high magnesium level. d. high potassium level.

b. low calcium level. Clients with a history of alcoholism are prone to hypocalcemia. The numbness and tingling of the lips and mouth are symptomatic of this and are referred to as "tetany."

6. An example of distraction to provide pain relief is a. TENS. b. music. c. exercise. d. biofeedback.

b. music.

Discharge criteria for the Phsae II patient include (select all that apply): a. no nausea or vomiting b. ability to drive self home c. no respiratory depression d. written discharge instructions understood e. opioid pain medication given 45 minutes ago

c, d, & e Rationale: Phase II discharge criteria that must be met include the following: all PACU discharge criteria (Phase I) met; no intravenous opioid drugs administered for the past 30 minutes; patient's ability to void (if appropriate with regard to surgical procedure or orders); patient's ability to ambulate if it is not contraindicated; presence of a responsible adult to accompany or drive patient home; and written discharge instructions given and understood.

4. The nurse is assessing a 22-yr-old patient experiencing the onset of symptoms of type 1 diabetes. To which question would the nurse anticipate a positive response? a. "Are you anorexic?" b. "Is your urine dark colored?" c. "Have you lost weight lately?" d. "Do you crave sugary drinks?"

c. "Have you lost weight lately?" - Weight loss occurs because the body is no longer able to absorb glucose and starts to break down protein and fat for energy. The patient is thirsty but does not necessarily crave sugar-containing fluids. Increased appetite is a classic symptom of type 1 diabetes. With the classic symptom of polyuria, urine will be very dilute.

42. After the nurse has finished teaching a patient who has a new prescription for exenatide (Byetta), which patient statement indicates that the teaching has been effective? a. "I may feel hungrier than usual when I take this medicine." b. "I will not need to worry about hypoglycemia with the Byetta." c. "I should take my daily aspirin at least an hour before the Byetta." d. "I will take the pill at the same time I eat breakfast in the morning."

c. "I should take my daily aspirin at least an hour before the Byetta." - Because exenatide slows gastric emptying, oral medications should be taken at least 1 hour before the exenatide to avoid slowing absorption. Exenatide is injected and increases feelings of satiety. Hypoglycemia can occur with this medication.

Which organ(s) is/are most at risk for dysfunction in a patient with a potassium level of 6.2 mEq/L? 1. Lungs 2. Kidneys 3. Liver 4. Heart

4. Heart

The nurse in the emergency department received change-of-shift report on these four clients with hypertension. Which of the following clients should the nurse assess first? a. 52-year-old with a BP of 212/90 who has intermittent claudication b. 43-year-old with a BP of 190/102 who is complaining of chest pain c. 50-year-old with a BP of 210/110 who has a creatinine of 133 mcmol/L d. 48-year-old with a BP of 200/98 whose urine shows microalbuminuria

43-year-old with a BP of 190/102 who is complaining of chest pain

Which pH value respresents acidosis? a. 7.26 b. 7.35 c. 7.4 d. 7.49

A normal serum pH is 7.35-7.45

The primary advantage of the use of midazolam (Versed) as an adjunct to general anesthesia is its a. amnestic effect b. analgesic effect c. antiemetic effect d. prolonged action

A (Amnestic effect Midazolam (Versed) is a rapid, short acting, sedative-hypnotic benzodiazepine that is used to prevent recall of events under anesthesia because of its amnestic properties.)

A break in sterile technique during surgery would occur when the scrub nurse touches a. the mask with gloved hands b. gloves hands to the gown at chest level c. the drape at the incision site with gloved hands d. the lower arms to the instruments on the instrument tray

A (The mask covering the face is not considered sterile, and if in contact with sterile gloved hands, contaminates the gloves. The gown at chest level and to 2 inches above elbows is considered sterile, as is the drape placed at the surgical area.)

The nurse is administering a fibrinolytic agent to a client with an acute myocardial infarction. Which of the following assessments should cause the nurse to stop the drug infusion? a. Bleeding from the gums b. Surface bleeding from the IV site c. A decrease in level of consciousness d. A nonsustained episode of ventricular tachycardia

A decrease in level of consciousness

The nurse is caring for a client who is scheduled for a cardiac catheterization with coronary angiography. Which of the following information should the nurse provide to the client before the test? a. Electrocardiographic (ECG) monitoring will be required for 24 hours after the test. b. It will be important to lie completely still during the procedure. c. A warm feeling may be noted when the contrast dye is injected. d. Monitored anaesthesia care will be provided during the procedure.

A warm feeling may be noted when the contrast dye is injected.

Which of the following cardiovascular effects of aging should the nurse anticipate when providing care for older adults (select all that apply)? A) Arterial stiffening B) Increased blood pressure C) Increased maximal heart rate D) Decreased maximal heart rate E) Increased recovery time from activity

A) Arterial stiffening B) Increased blood pressure D) Decreased maximal heart rate E) Increased recovery time from activity Well-documented cardiovascular effects of the aging process include arterial stiffening, possible increased blood pressure, and an increased amount of time that is required for recovery from activity. Maximal heart rate tends to decrease rather than increase with age.

A nurse is caring for a patient immediately following a transesophageal echocardiogram (TEE). Which of the following assessments are appropriate for this patient (select all that apply)? A) Assess for return of gag reflex. B) Assess groin for hematoma or bleeding. C) Monitor vital signs and oxygen saturation. D) Position patient supine with head of bed flat. E) Assess lower extremities for circulatory compromise.

A) Assess for return of gag reflex. C) Monitor vital signs and oxygen saturation. The patient undergoing a TEE has been given conscious sedation and has had the throat numbed with a local anesthetic spray, thus eliminating the gag reflex until the effects wear off. Therefore it is imperative that the nurse assess for gag reflex return before allowing the patient to eat or drink. Vital signs and oxygen saturation are also important assessment parameters resulting from the use of sedation. A TEE does not involve invasive procedures of the circulatory blood vessels. Therefore it is not necessary to monitor the patient's groin or lower extremities in relation to this procedure.

The nurse is providing care for a patient who has decreased cardiac output related to heart failure. The nurse recognizes that cardiac output is A) Calculated by multiplying the patient's stroke volume by the heart rate. B) The average amount of blood ejected during one complete cardiac cycle. C) Determined by measuring the electrical activity of the heart and the patient's heart rate. D) The patient's average resting heart rate multiplied by the patient's mean arterial blood pressure.

A) Calculated by multiplying the patient's stroke volume by the heart rate. Cardiac output is determined by multiplying the patient's stroke volume by heart rate, thus identifying how much blood is pumped by the heart over a 1-minute period. Electrical activity of the heart and blood pressure are not direct components of cardiac output.

1. In which order will the nurse take these steps to prepare NPH 20 units and regular insulin 2 units using the same syringe? (Put a comma and a space between each answer choice [A, B, C, D, E]). a. Rotate NPH vial. b. Withdraw regular insulin. c. Withdraw 20 units of NPH. d. Inject 20 units of air into NPH vial. e. Inject 2 units of air into regular insulin vial.

A, D, E, B, C

Which of the following are functions of sodium in the body? Select all that apply. a. Maintenance of serum osmolarity b. Formation of bones and teeth c. Control of bronchodilation d. Control of serum glucose e. Maintenance of cellular function

A,E Sodium is the major cation in the blood and helps maintain serum osmolarity. Sodium is also important for cell function, especially in the central nervous system.

When planning care for adult patients, which oral intake is adequate to meet daily fluid needs of a stable patient? a. 500 to 1500 mL b. 1200 to 2200 mL c. 2000 to 3000 mL d. 3000 to 4000 mL

c. 2000 to 3000 mL Daily fluid intake and output is usually 2000 to 3000 mL. This is sufficient to meet the needs of the body and replace both sensible and insensible fluid losses. These would include urine output and fluids lost through the respiratory system, skin, and GI tract.

Which patient would be at greatest risk for the potential development of hypermagnesemia? a. 83-year-old man with lung cancer and hypertension b. 65-year-old woman with hypertension taking Beta-adrenergic blockers c. 42-year-old woman with systemic lupus erythematosus and renal failure d. 50-year-old man with benign prostatic hyperplasia and a urinary tract infection

c. 42-year-old woman with systemic lupus erythematosus and renal failure Rationale: Causes of hypermagnesemia include renal failure (especially if the patient is given magnesium products), excessive administration of magnesium for treatment of eclampsia, and adrenal insufficiency.

A patient's temperature has been 101° F (38.3° C) for several days. The patient's normal caloric intake to meet nutritional needs is 2000 calories per day. Knowing that the metabolic rate increases 7% for each Fahrenheit degree above 100° in body temperature, how many total calories should the patient receive each day?

ANS: 2140 calories DIF: Cognitive Level: Apply (application)

The nurse notes new onset confusion in an 89-year-old patient in a long-term care facility. The patient is normally alert and oriented. In which order should the nurse take the following actions? Put a comma and space between each answer choice (a, b, c, d, etc.) ____________________ a. Obtain the oxygen saturation. b. Check the patient's pulse rate. c. Document the change in status. d. Notify the health care provider.

ANS: A, B, D, C Assessment for physiologic causes of new onset confusion such as pneumonia, infection, or perfusion problems should be the first action by the nurse. Airway and oxygenation should be assessed first, then circulation. After assessing the patient, the nurse should notify the health care provider. Finally, documentation of the assessments and care should be done. DIF: Cognitive Level: Analysis REF: 549 | 551 OBJ: Special Questions: Alternate Item Format, Prioritization TOP: Nursing Process: Implementation MSC: NCLEX: Physiolog

46. After change-of-shift report, which patient will the nurse assess first? a. A 19-yr-old patient with type 1 diabetes who was admitted with possible dawn phenomenon b. A 35-yr-old patient with type 1 diabetes whose most recent blood glucose reading was 230 mg/dL c. A 60-yr-old patient with hyperosmolar hyperglycemic syndrome who has poor skin turgor and dry oral mucosa d. A 68-yr-old patient with type 2 diabetes who has severe peripheral neuropathy and complains of burning foot pain

c. A 60-yr-old patient with hyperosmolar hyperglycemic syndrome who has poor skin turgor and dry oral mucosa - The patient's diagnosis of HHS and signs of dehydration indicate that the nurse should rapidly assess for signs of shock and determine whether increased fluid infusion is needed. The other patients also need assessment and intervention but do not have life-threatening complications.

A patient who has an infected abdominal wound develops a temperature of 104° F (40° C). All the following interventions are included in the patient's plan of care. In which order should the nurse perform the following actions? (Put a comma and a space between each answer choice [A, B, C, D]). a. Administer IV antibiotics. b. Sponge patient with cool water. c. Perform wet-to-dry dressing change. d. Administer acetaminophen (Tylenol).

ANS: A, D, B, C The first action should be to administer the antibiotic because treating the infection that has caused the fever is the most important aspect of fever management. The next priority is to lower the high fever, so the nurse should administer acetaminophen to lower the temperature set point. A cool sponge bath should be done after the acetaminophen is given to lower the temperature further. The wet-to-dry dressing change will not have an immediate impact on the infection or fever and should be done last. DIF: Cognitive Level: Analyze (analysis)

The nurse enters the room of a patient who has just returned from surgery for a total laryngectomy and radical neck dissection and notes the following problems. In which order should the nurse address the problems? Put a comma and space between each answer choice (a, b, c, d, etc.) ____________________ a. The nasogastric (NG) tube is disconnected from suction and clamped off. b. The patient is in a side-lying position with the head of the bed flat. c. The Hemovac in the neck incision contains 200 mL of bloody drainage. d. The patient is coughing blood-tinged secretions from the tracheostomy

ANS: B, D, C, A The patient should first be placed in a semi-Fowler's position to maintain the airway and reduce incisional swelling. The blood-tinged secretions may obstruct the airway, so suctioning is the next appropriate action. Then the Hemovac should be drained because the 200 mL of drainage will decrease the amount of suction in the Hemovac and could lead to incisional swelling and poor healing. Finally, the NG tube should be reconnected to suction to prevent gastric dilation, nausea, and vomiting. DIF: Cognitive Level: Analysis REF: 532-534 | 538-539 OBJ: Special Questions: Alternate Item Format, Prioritization TOP: Nursing Process: Implementation MSC: NCLEX: Physiologi

When caring for a patient who has just arrived on the telemetry unit after having cardiac catheterization, which nursing intervention should the nurse delegate to a licensed practical/vocational nurse (LPN/LVN)? a. Give the scheduled aspirin and lipid-lowering medication. b. Perform the initial assessment of the catheter insertion site. c. Teach the patient about the usual postprocedure plan of care. d. Titrate the heparin infusion according to the agency protocol.

ANS: A Administration of oral medications is within the scope of practice for LPNs/LVNs. The initial assessment of the patient, patient teaching, and titration of IV anticoagulant medications should be done by the registered nurse (RN). DIF: Cognitive Level: Apply (application)

When admitting a patient with a non-ST-segment-elevation myocardial infarction (NSTEMI) to the intensive care unit, which action should the nurse perform first? a. Attach the heart monitor. c. Assess the peripheral pulses. b. Obtain the blood pressure. d. Auscultate the breath sounds.

ANS: A Because dysrhythmias are the most common complication of myocardial infarction (MI), the first action should be to place the patient on a heart monitor. The other actions are also important and should be accomplished as quickly as possible. DIF: Cognitive Level: Analyze (analysis)

19. After being hit by a baseball, a patient arrives in the emergency department with a possible nasal fracture. Which finding by the nurse is most important to report to the health care provider? a. Clear nasal drainage b. Complaint of nasal pain c. Bilateral nose swelling and bruising d. Inability to breathe through the nose

ANS: A Clear nasal drainage may indicate a meningeal tear with leakage of cerebrospinal fluid. This would place the patient at risk for complications such as meningitis. The other findings are typical with a nasal fracture and do not indicate any complications.

A patient with bacterial pneumonia has rhonchi and thick sputum. Which action will the nurse use to promote airway clearance? a. Assist the patient to splint the chest when coughing. b. Educate the patient about the need for fluid restrictions. c. Encourage the patient to wear the nasal oxygen cannula. d. Instruct the patient on the pursed lip breathing technique.

ANS: A Coughing is less painful and more likely to be effective when the patient splints the chest during coughing. Fluids should be encouraged to help liquefy secretions. Nasal oxygen will improve gas exchange, but will not improve airway clearance. Pursed lip breathing is used to improve gas exchange in patients with COPD, but will not improve airway clearance. DIF: Cognitive Level: Application REF: 552-553 TOP: Nursing Process: Implementation MSC: NCLEX: Physiological Integrity

The nurse is caring for a patient who was admitted to the coronary care unit following an acute myocardial infarction (AMI) and percutaneous coronary intervention the previous day. Teaching for this patient would include a. when cardiac rehabilitation will begin. b. the typical emotional responses to AMI. c. information regarding discharge medications. d. the pathophysiology of coronary artery disease.

ANS: A Early after an AMI, the patient will want to know when resumption of usual activities can be expected. At this time, the patient's anxiety level or denial will interfere with good understanding of complex information such as the pathophysiology of coronary artery disease. Teaching about discharge medications should be done closer to discharge. The nurse should support the patient by decreasing anxiety rather than discussing the typical emotional responses to myocardial infarction. DIF: Cognitive Level: Apply (application)

39. When a patient with type 2 diabetes is admitted for a cholecystectomy, which nursing action can the nurse delegate to a licensed practical/vocational nurse (LPN/LVN)? a. Communicate the blood glucose level and insulin dose to the circulating nurse in surgery. b. Discuss the reason for the use of insulin therapy during the immediate postoperative period. c. Administer the prescribed lispro (Humalog) insulin before transporting the patient to surgery. d. Plan strategies to minimize the risk for hypoglycemia or hyperglycemia during the postoperative period.

c. Administer the prescribed lispro (Humalog) insulin before transporting the patient to surgery. - LPN/LVN education and scope of practice includes administration of insulin. Communication about patient status with other departments, planning, and patient teaching are skills that require RN education and scope of practice.

A patient who has diabetes is admitted for an exploratory laparotomy for abdominal pain. When planning interventions to promote wound healing, what is the nurse's highest priority? a. Maintaining the patient's blood glucose within a normal range b. Ensuring that the patient has an adequate dietary protein intake c. Giving antipyretics to keep the temperature less than 102° F (38.9° C) d. Redressing the surgical incision with a dry, sterile dressing twice daily

ANS: A Elevated blood glucose will have an impact on multiple factors involved in wound healing. Ensuring adequate nutrition is also important for the postoperative patient, but a higher priority is blood glucose control. A temperature of 102° F will not impact adversely on wound healing, although the nurse may administer antipyretics if the patient is uncomfortable. Application of a dry, sterile dressing daily may be ordered, but frequent dressing changes for a wound healing by primary intention is not necessary to promote wound healing. DIF: Cognitive Level: Analyze (analysis)

A patient who has been receiving diuretic therapy is admitted to the emergency department with a serum potassium level of 3.0 mEq/L. The nurse should alert the health care provider immediately that the patient is on which medication? a. Oral digoxin (Lanoxin) 0.25 mg daily b. Ibuprofen (Motrin) 400 mg every 6 hours c. Metoprolol (Lopressor) 12.5 mg orally daily d. Lantus insulin 24 U subcutaneously every evening

ANS: A Hypokalemia increases the risk for digoxin toxicity, which can cause serious dysrhythmias. The nurse will also need to do more assessment regarding the other medications, but they are not of as much concern with the potassium level

6. The nurse is caring for a mechanically ventilated patient with a cuffed tracheostomy tube. Which action by the nurse would best determine if the cuff has been properly inflated? a. Use a manometer to ensure cuff pressure is at an appropriate level. b. Check the amount of cuff pressure ordered by the health care provider. c. Suction the patient first with a fenestrated inner cannula to clear secretions. d. Insert the decannulation plug before the nonfenestrated inner cannula is removed.

ANS: A Measurement of cuff pressure using a manometer to ensure that cuff pressure is 20 mm Hg or lower will avoid compression of the tracheal wall and capillaries. Never insert the decannulation plug in a tracheostomy tube until the cuff is deflated and the nonfenestrated inner cannula is removed. Otherwise, the patient's airway is occluded. A health care provider's order is not required to determine safe cuff pressure. A nonfenestrated inner cannula must be used to suction a patient to prevent tracheal damage occurring from the suction catheter passing through the fenestrated openings.

The nurse will perform which action when doing a wet-to-dry dressing change on a patient's stage III sacral pressure ulcer? a. Administer prescribed PRN hydrocodone 30 minutes before the change. b. Pour sterile saline onto the new dry dressings after the wound has been packed. c. Apply antimicrobial ointment before repacking the wound with moist dressings. d. Soak the old dressings with sterile saline 30 minutes before the dressing change

ANS: A Mechanical debridement with wet-to-dry dressings is painful, and patients should receive pain medications before the dressing change begins. The new dressings are moistened with saline before being applied to the wound but not soaked after packing. Soaking the old dressings before removing them will eliminate the wound debridement that is the purpose of this type of dressing. Application of antimicrobial ointments is not indicated for a wet-to-dry dressing. DIF: Cognitive Level: Apply (application)

A patient who has recently started taking pravastatin (Pravachol) and niacin reports several symptoms to the nurse. Which information is most important to communicate to the health care provider? a. Generalized muscle aches and pains b. Dizziness with rapid position changes c. Nausea when taking the drugs before meals d. Flushing and pruritus after taking the drugs

ANS: A Muscle aches and pains may indicate myopathy and rhabdomyolysis, which have caused acute kidney injury and death in some patients who have taken the statin medications. These symptoms indicate that the pravastatin may need to be discontinued. The other symptoms are common side effects when taking niacin, and although the nurse should follow-up with the health care provider, they do not indicate that a change in medication is needed. DIF: Cognitive Level: Analyze (analysis)

When teaching the patient who is receiving standard multidrug therapy for tuberculosis (TB) about possible toxic effects of the antitubercular medications, the nurse will give instructions to notify the health care provider if the patient develops a. yellow-tinged skin. b. changes in hearing. c. orange-colored sputum. d. thickening of the fingernails.

ANS: A Noninfectious hepatitis is a toxic effect of isoniazid (INH), rifampin, and pyrazinamide, and patients who develop hepatotoxicity will need to use other medications. Changes in hearing and nail thickening are not expected with the four medications used for initial TB drug therapy. Orange discoloration of body fluids is an expected side effect of rifampin and not an indication to call the health care provider. DIF: Cognitive Level: Application REF: 555 | 556 TOP: Nursing Process: Implementation MSC: NCLEX: Physiological Integrity

The nurse is caring for a hospitalized 82-year-old patient who has nasal packing in place to treat a nosebleed. Which of the following assessment findings will require the most immediate action by the nurse? a. The oxygen saturation is 89%. b. The nose appears red and swollen. c. The patient's temperature is 100.1° F (37.8° C). d. The patient complains of level 7 (0 to 10 scale) pain.

ANS: A Older patients with nasal packing are at risk of aspiration or airway obstruction. An O2 saturation of 89% should alert the nurse to assess further for these complications. The other assessment data also indicate a need for nursing action but not as immediately as the fall in O2 saturation. DIF: Cognitive Level: Application REF: 520 OBJ: Special Questions: Prioritization TOP: Nursing Process: Assessment MSC: NCLEX: Physiological Integrity

18. The nurse is caring for a hospitalized older patient who has nasal packing in place to treat a nosebleed. Which assessment finding will require the most immediate action by the nurse? a. The oxygen saturation is 89%. b. The nose appears red and swollen. c. The patient's temperature is 100.1° F (37.8° C). d. The patient complains of level 8 (0 to 10 scale) pain.

ANS: A Older patients with nasal packing are at risk of aspiration or airway obstruction. An O2 saturation of 89% should alert the nurse to further assess for these complications. The other assessment data also indicate a need for nursing action but not as immediately as the low O2 saturation.

Which information will the nurse include in the patient teaching plan for a patient who is receiving rifampin (Rifadin) for treatment of tuberculosis? a. "Your urine, sweat, and tears will be orange colored." b. "Read a newspaper daily to check for changes in vision." c. "Take vitamin B6 daily to prevent peripheral nerve damage." d. "Call the health care provider if you notice any hearing loss."

ANS: A Orange-colored body secretions are a side effect of rifampin. The other adverse effects are associated with other antituberculosis medications. DIF: Cognitive Level: Application REF: 555 TOP: Nursing Process: Planning MSC: NCLEX: Physiological Integrity

A patient experiences a steering wheel injury as a result of an automobile accident. During the initial assessment, the emergency department nurse would be most concerned about a. paradoxic chest movement. b. the complaint of chest wall pain. c. a heart rate of 110 beats/minute. d. a large bruised area on the chest.

ANS: A Paradoxic chest movement indicates that the patient may have flail chest, which can severely compromise gas exchange and can rapidly lead to hypoxemia. Chest wall pain, a slightly elevated pulse rate, and chest bruising all require further assessment or intervention, but the priority concern is poor gas exchange. DIF: Cognitive Level: Application REF: 567 | 569 TOP: Nursing Process: Assessment MSC: NCLEX: Physiological Integrity

The nurse observes nursing assistive personnel (NAP) doing all the following activities when caring for a patient with right lower lobe pneumonia. The nurse will need to intervene when NAP a. lower the head of the patient's bed to 10 degrees. b. splint the patient's chest during coughing. c. help the patient to ambulate to the bathroom. d. assist the patient to a bedside chair for meals.

ANS: A Positioning the patient with the head of the bed lowered will decrease ventilation. The other actions are appropriate for a patient with pneumonia. DIF: Cognitive Level: Application REF: 552-553 OBJ: Special Questions: Delegation TOP: Nursing Process: Implementation MSC: NCLEX: Safe and Effective Care Environment

A new nurse performs a dressing change on a stage II left heel pressure ulcer. Which action by the new nurse indicates a need for further teaching about pressure ulcer care? a. The new nurse cleans the ulcer with half-strength peroxide. b. The new nurse uses a hydrocolloid dressing (DuoDerm)on the ulcer. c. The new nurse irrigates the pressure ulcer with saline using a 30-mL syringe. d. The new nurse inserts a sterile cotton-tipped applicator into the pressure ulcer.

ANS: A Pressure ulcers should not be cleaned with solutions that are cytotoxic, such as hydrogen peroxide. The other actions by the new nurse are appropriate. DIF: Cognitive Level: Apply (application)

The nurse is obtaining a health history from a 67-year-old patient with a 40 pack-year smoking history, complaints of hoarseness and tightness in the throat, and difficulty swallowing. Which question is most important for the nurse to ask? a. "How much alcohol do you drink in an average week?" b. "Do you have a family history of head or neck cancer?" c. "Have you had frequent streptococcal throat infections?" d. "Do you use antihistamines for upper airway congestion?"

ANS: A Prolonged alcohol use and smoking are associated with the development of laryngeal cancer, which the patient's symptoms and history suggest. Family history is not a risk factor for head or neck cancer. Frequent antihistamine use would be asked about if the nurse suspected allergic rhinitis, but the patient's symptoms are not suggestive of this diagnosis. Streptococcal throat infections also may cause these clinical manifestations, but patients also will complain of pain and fever. DIF: Cognitive Level: Application REF: 535 | 538 TOP: Nursing Process: Assessment MSC: NCLEX: Health Promotion and Maintenance

8. A nurse obtains a health history from a patient who has a 35 pack-year smoking history. The patient complains of hoarseness and tightness in the throat and difficulty swallowing. Which question is most important for the nurse to ask? a. "How much alcohol do you drink in an average week?" b. "Do you have a family history of head or neck cancer?" c. "Have you had frequent streptococcal throat infections?" d. "Do you use antihistamines for upper airway congestion?"

ANS: A Prolonged alcohol use and smoking are associated with the development of laryngeal cancer, which the patient's symptoms and history suggest. Family history is not a risk factor for head or neck cancer. Frequent antihistamine use would be asked about if the nurse suspected allergic rhinitis, but the patient's symptoms are not suggestive of this diagnosis. Streptococcal throat infections also may cause these clinical manifestations, but patients with this type of infection will also have pain and a fever.

A patient arrives in the emergency department with a swollen ankle after a soccer injury. Which action by the nurse is appropriate? a. Elevate the ankle above heart level. b. Apply a warm moist pack to the ankle. c. Ask the patient to try bearing weight on the ankle. d. Assess the ankle's passive range of motion (ROM).

ANS: A Soft tissue injuries are treated with rest, ice, compression, and elevation (RICE). Elevation of the ankle will decrease tissue swelling. Moving the ankle through the ROM will increase swelling and risk further injury. Cold packs should be applied the first 24 hours to reduce swelling. The nurse should not ask the patient to move or bear weight on the swollen ankle because immobilization of the inflamed or injured area promotes healing by decreasing metabolic needs of the tissues. DIF: Cognitive Level: Apply (application)

A patient with an uncuffed tracheostomy tube coughs violently during suctioning and dislodges the tracheostomy tube. Which action should the nurse take first? a. Insert the obturator and attempt to reinsert the tracheostomy tube. b. Position the patient in an upright position with the neck extended. c. Assess the patient's oxygen saturation and notify the health care provider. d. Ventilate the patient with a manual bag until the health care provider arrives.

ANS: A The first action should be to attempt to reinsert the tracheostomy tube to maintain the patient's airway. Assessing the patient's oxygenation is an important action, but it is not the most appropriate first action in this situation. Covering the stoma with a dressing and manually ventilating the patient may be an appropriate action if the nurse is unable to reinsert the tracheostomy tube. The patient should be placed in a semi-Fowler's position if reinsertion of the tracheostomy tube is not successful. DIF: Cognitive Level: Application REF: 531 OBJ: Special Questions: Prioritization TOP: Nursing Process: Implementation MSC: NCLEX: Physiological Integrity

After 2 months of tuberculosis (TB) treatment with a standard four-drug regimen, a patient continues to have positive sputum smears for acid-fast bacilli (AFB). Which action should the nurse take next? a. Ask the patient whether medications have been taken as directed. b. Discuss the need to use some different medications to treat the TB. c. Schedule the patient for directly observed therapy three times weekly. d. Educate about using a 2-drug regimen for the last 4 months of treatment.

ANS: A The first action should be to determine whether the patient has been compliant with drug therapy because negative sputum smears would be expected if the TB bacillus is susceptible to the medications and if the medications have been taken correctly. Depending on whether the patient has been compliant or not, different medications or directly observed therapy may be indicated. A two-drug regimen will be used only if the sputum smears are negative for AFB. DIF: Cognitive Level: Application REF: 556-557 TOP: Nursing Process: Implementation MSC: NCLEX: Physiological Integrity

12. Which action should the nurse take first when a patient develops a nosebleed? a. Pinch the lower portion of the nose for 10 minutes. b. Pack the affected nare tightly with an epistaxis balloon. c. Obtain silver nitrate that will be needed for cauterization. d. Apply ice compresses over the patient's nose and cheeks.

ANS: A The first nursing action for epistaxis is to apply direct pressure by pinching the nostrils. Application of cold packs may decrease blood flow to the area, but will not be sufficient to stop bleeding. Cauterization and nasal packing are medical interventions that may be needed if pressure to the nares does not stop the bleeding, but these are not the first actions to take for a nosebleed.

A patient with renal failure has been taking aluminum hydroxide/magnesium hydroxide suspension (Maalox) at home for indigestion. The patient arrives for outpatient hemodialysis and is unresponsive to questions and has decreased deep tendon reflexes. Which action should the dialysis nurse take first? a. Notify the patient's health care provider. b. Obtain an order to draw a potassium level. c. Review the magnesium level on the patient's chart. d. Teach the patient about the risk of magnesium-containing antacids

ANS: A The health care provider should be notified immediately. The patient has a history and manifestations consistent with hypermagnesemia. The nurse should check the chart for a recent serum magnesium level and make sure that blood is sent to the laboratory for immediate electrolyte and chemistry determinations. Dialysis should correct the high magnesium levels. The patient needs teaching about the risks of taking magnesium-containing antacids. Monitoring of potassium levels also is important for patients with renal failure, but the patient's current symptoms are not consistent with hyperkalemia

A patient with an open leg lesion has a white blood cell (WBC) count of 13,500/µL and a band count of 11%. What prescribed action should the nurse take first? a. Obtain cultures of the wound. b. Begin antibiotic administration. c. Continue to monitor the wound for drainage. d. Redress the wound with wet-to-dry dressings.

ANS: A The increase in WBC count with the increased bands (shift to the left) indicates that the patient probably has a bacterial infection, and the nurse should obtain wound cultures. Antibiotic therapy and/or dressing changes may be started, but cultures should be done first. The nurse will continue to monitor the wound, but additional actions are needed as well. DIF: Cognitive Level: Analyze (analysis)

A staff nurse has a tuberculosis (TB) skin test of 16-mm induration. A chest radiograph is negative, and the nurse has no symptoms of TB. The occupational health nurse will plan on teaching the staff nurse about the a. use and side effects of isoniazid (INH). b. standard four-drug therapy for TB. c. need for annual repeat TB skin testing. d. bacille Calmette-Guérin (BCG) vaccine.

ANS: A The nurse is considered to have a latent TB infection and should be treated with INH daily for 6 to 9 months. The four-drug therapy would be appropriate if the nurse had active TB. TB skin testing is not done for individuals who have already had a positive skin test. BCG vaccine is not used in the United States and would not be helpful for this individual, who already has a TB infection. DIF: Cognitive Level: Application REF: 556-557 TOP: Nursing Process: Planning MSC: NCLEX: Health Promotion and Maintenance

A patient who is lethargic and exhibits deep, rapid respirations has the following arterial blood gas (ABG) results: pH 7.32, PaO2 88 mm Hg, PaCO2 37 mm Hg, and HCO3 16 mEq/L. How should the nurse interpret these results? a. Metabolic acidosis b. Metabolic alkalosis c. Respiratory acidosis d. Respiratory alkalosis

ANS: A The pH and HCO3 indicate that the patient has a metabolic acidosis. The ABGs are inconsistent with the other responses

A patient with a possible pulmonary embolism complains of chest pain and difficulty breathing. The nurse finds a heart rate of 142, BP reading of 100/60, and respirations of 42. The nurse's first action should be to a. elevate the head of the bed to 45 to 60 degrees. b. administer the ordered pain medication. c. notify the patient's health care provider. d. offer emotional support and reassurance.

ANS: A The patient has symptoms consistent with a pulmonary embolism. Elevating the head of the bed will improve ventilation and gas exchange. The other actions can be accomplished after the head is elevated (and oxygen is started). DIF: Cognitive Level: Application REF: 580 OBJ: Special Questions: Prioritization TOP: Nursing Process: Implementation MSC: NCLEX: Physiological Integrity

An hour after a thoracotomy, a patient complains of incisional pain at a level 7 out of 10 and has decreased left-sided breath sounds. The pleural drainage system has 100 mL of bloody drainage and a large air leak. Which action is best for the nurse to take next? a. Administer the prescribed PRN morphine. b. Assist the patient to deep breathe and cough. c. Milk the chest tube gently to remove any clots. d. Tape the area around the insertion site of the chest tube.

ANS: A The patient is unlikely to take deep breaths or cough until the pain level is lower. A chest tube output of 100 mL is not unusual in the first hour after thoracotomy and would not require milking of the chest tube. An air leak is expected in the initial postoperative period after thoracotomy. DIF: Cognitive Level: Application REF: 573-574 TOP: Nursing Process: Implementation MSC: NCLEX: Physiological Integrity

A newly admitted patient is diagnosed with hyponatremia. When making room assignments, the charge nurse should take which action? a. Assign the patient to a room near the nurse's station. b. Place the patient in a room nearest to the water fountain. c. Place the patient on telemetry to monitor for peaked T waves. d. Assign the patient to a semi-private room and place an order for a low-salt diet.

ANS: A The patient should be placed near the nurse's station if confused in order for the staff to closely monitor the patient. To help improve serum sodium levels, water intake is restricted. Therefore a confused patient should not be placed near a water fountain. Peaked T waves are a sign of hyperkalemia, not hyponatremia. A confused patient could be distracting and disruptive for another patient in a semiprivate room. This patient needs sodium replacement, not restriction

Which of these patients in the respiratory disease clinic should the nurse assess first? a. A 23-year-old, complaining of a sore throat, who has a "hot potato" voice b. A 34-year-old who has a "scratchy throat" and a positive rapid strep antigen test c. A 55-year-old who is receiving radiation for throat cancer and has severe fatigue d. A 72-year-old with a history of a total laryngectomy whose stoma is red and inflamed

ANS: A The patient's clinical manifestation of a "hot potato" voice suggests a possible peritonsillar abscess that could lead to an airway obstruction requiring rapid assessment and potential treatment. The other patients do not have diagnoses or symptoms that indicate any life-threatening problems. DIF: Cognitive Level: Analysis REF: 528 OBJ: Special Questions: Prioritization TOP: Nursing Process: Assessment MSC: NCLEX: Physiological Integrity

15. Which patient in the ear, nose, and throat (ENT) clinic should the nurse assess first? a. A 23-year-old who is complaining of a sore throat and has a muffled voice b. A 34-year-old who has a "scratchy throat" and a positive rapid strep antigen test c. A 55-year-old who is receiving radiation for throat cancer and has severe fatigue d. A 72-year-old with a history of a total laryngectomy whose stoma is red and inflamed

ANS: A The patient's clinical manifestation of a muffled voice suggests a possible peritonsillar abscess that could lead to an airway obstruction requiring rapid assessment and potential treatment. The other patients do not have diagnoses or symptoms that indicate any life-threatening problems.

Two days after an acute myocardial infarction (MI), a patient complains of stabbing chest pain that increases with a deep breath. Which action will the nurse take first? a. Auscultate the heart sounds. b. Check the patient's temperature. c. Notify the patient's health care provider. d. Give the PRN acetaminophen (Tylenol).

ANS: A The patient's clinical manifestations and history are consistent with pericarditis, and the first action by the nurse should be to listen for a pericardial friction rub. Checking the temperature and notifying the health care provider are also appropriate actions but would not be done before listening for a rub. It is not stated for what symptom (e.g., headache) or finding (e.g., increased temperature) the PRN acetaminophen (Tylenol) is ordered.

A patient who was admitted the previous day with pneumonia complains of a sharp pain "whenever I take a deep breath." Which action will the nurse take next? a. Listen to the patient's lungs. b. Administer the PRN morphine. c. Have the patient cough forcefully. d. Notify the patient's health care provider.

ANS: A The patient's statement indicates that pleurisy or a pleural effusion may have developed and the nurse will need to listen for a pleural friction rub and/or decreased breath sounds. Assessment should occur before administration of pain medications. The patient is unlikely to be able to cough forcefully until pain medication has been administered. The nurse will want to obtain more assessment data before calling the health care provider. DIF: Cognitive Level: Application REF: 576 OBJ: Special Questions: Prioritization TOP: Nursing Process: Assessment MSC: NCLEX: Physiological Integrity

Following a thyroidectomy, a patient complains of "a tingling feeling around my mouth." Which assessment should the nurse complete immediately? a. Presence of the Chvostek's sign b. Abnormal serum potassium level c. Decreased thyroid hormone level d. Bleeding on the patient's dressing

ANS: A The patient's symptoms indicate possible hypocalcemia, which can occur secondary to parathyroid injury/removal during thyroidectomy. There is no indication of a need to check the potassium level, the thyroid hormone level, or for bleeding

11. The nurse completes discharge instructions for a patient with a total laryngectomy. Which statement by the patient indicates that additional instruction is needed? a. "I must keep the stoma covered with an occlusive dressing at all times." b. "I can participate in most of my prior fitness activities except swimming." c. "I should wear a Medic-Alert bracelet that identifies me as a neck breather." d. "I need to be sure that I have smoke and carbon monoxide detectors installed."

ANS: A The stoma may be covered with clothing or a loose dressing, but this is not essential. An occlusive dressing will completely block the patient's airway. The other patient comments are all accurate and indicate that the teaching has been effective.

After completing discharge instructions for a patient with a total laryngectomy, the nurse determines that additional instruction is needed when the patient says, a. "I must keep the stoma covered with a loose sterile dressing at all times." b. "I can participate in most of my prior fitness activities except swimming." c. "I should wear a Medic Alert bracelet that identifies me as a neck breather." d. "I need to be sure that I have smoke and carbon monoxide detectors installed."

ANS: A The stoma may be covered with clothing or a loose dressing, but this is not essential. The other patient comments are all accurate and indicate that the teaching has been effective. DIF: Cognitive Level: Application REF: 542 TOP: Nursing Process: Evaluation MSC: NCLEX: Physiological Integrity

Following assessment of a patient with pneumonia, the nurse identifies a nursing diagnosis of ineffective airway clearance. Which information best supports this diagnosis? a. Weak, nonproductive cough effort b. Large amounts of greenish sputum c. Respiratory rate of 28 breaths/minute d. Resting pulse oximetry (SpO2) of 85%

ANS: A The weak, nonproductive cough indicates that the patient is unable to clear the airway effectively. The other data would be used to support diagnoses such as impaired gas exchange and ineffective breathing pattern. DIF: Cognitive Level: Application REF: 551-552 TOP: Nursing Process: Diagnosis MSC: NCLEX: Physiological Integrity

The nurse notes a serum calcium level of 7.9 mg/dL for a patient who has chronic malnutrition. Which action should the nurse take next? a. Monitor ionized calcium level. b. Give oral calcium citrate tablets. c. Check parathyroid hormone level. d. Administer vitamin D supplements.

ANS: A This patient with chronic malnutrition is likely to have a low serum albumin level, which will affect the total serum calcium. A more accurate reflection of calcium balance is the ionized calcium level. Most of the calcium in the blood is bound to protein (primarily albumin). Alterations in serum albumin levels affect the interpretation of total calcium levels. Low albumin levels result in a drop in the total calcium level, although the level of ionized calcium is not affected. The other actions may be needed if the ionized calcium is also decreased

A few days after experiencing a myocardial infarction (MI) and successful percutaneous coronary intervention, the patient states, "It was just a little chest pain. As soon as I get out of here, I'm going for my vacation as planned." Which reply would be most appropriate for the nurse to make? a. "What do you think caused your chest pain?" b. "Where are you planning to go for your vacation?" c. "Sometimes plans need to change after a heart attack." d. "Recovery from a heart attack takes at least a few weeks."

ANS: A When the patient is experiencing denial, the nurse should assist the patient in testing reality until the patient has progressed beyond this step of the emotional adjustment to MI. Asking the patient about vacation plans reinforces the patient's plan, which is not appropriate in the immediate post-MI period. Reminding the patient in denial about the MI is likely to make the patient angry and lead to distrust of the nursing staff. DIF: Cognitive Level: Apply (application)

Which finding is most important for the nurse to communicate to the health care provider when caring for a patient who is receiving negative-pressure wound therapy? a. Low serum albumin level b. Serosanguineous drainage c. Deep red and moist wound bed d. Cobblestone appearance of wound

ANS: A With negative pressure therapy, serum protein levels may decrease, which will adversely affect wound healing. The other findings are expected with wound healing. DIF: Cognitive Level: Analyze (analysis)

A patient with laryngeal cancer has received teaching about radiation therapy. Which statement by the patient indicates that the teaching has been effective? a. "I will need to buy a water bottle to carry with me." b. "I should not use any lotions on my neck and throat." c. "Until the radiation is complete, I may have diarrhea." d. "Alcohol-based mouthwashes will help clean oral ulcers."

ANS: A Xerostomia can be partially alleviated by drinking fluids at frequent intervals. Radiation will damage tissues at the site being radiated but should not affect the abdominal organs, so loose stools are not a usual complication of head and neck radiation therapy. Frequent oral rinsing with non-alcohol-based rinses is recommended. Prescribed lotions and sunscreen may be used on irradiated skin, although they should not be used just before the radiation therapy. DIF: Cognitive Level: Application REF: 538 TOP: Nursing Process: Evaluation MSC: NCLEX: Physiological Integrity

7. Which statement by the patient indicates that the teaching has been effective for a patient scheduled for radiation therapy of the larynx? a. "I will need to buy a water bottle to carry with me." b. "I should not use any lotions on my neck and throat." c. "Until the radiation is complete, I may have diarrhea." d. "Alcohol-based mouthwashes will help clean oral ulcers."

ANS: A Xerostomia can be partially alleviated by drinking fluids at frequent intervals. Radiation will damage tissues at the site being radiated but should not affect the abdominal organs, so loose stools are not a usual complication of head and neck radiation therapy. Frequent oral rinsing with non-alcohol-based rinses is recommended. Prescribed lotions and sunscreen may be used on radiated skin, although they should not be used just before the radiation therapy.

A postoperative patient who has been receiving nasogastric suction for 3 days has a serum sodium level of 125 mEq/L (125 mmol/L). Which of these prescribed therapies that the patient has been receiving should the nurse question? a. Infuse 5% dextrose in water at 125 ml/hr. b. Administer IV morphine sulfate 4 mg every 2 hours PRN. c. Give IV metoclopramide (Reglan) 10 mg every 6 hours PRN for nausea. d. Administer 3% saline if serum sodium drops to less than 128 mEq/L.

ANS: A Infuse 5% dextrose in water at 125 ml/hr. Because the patient's gastric suction has been depleting electrolytes, the IV solution should include electrolyte replacement. Solutions such as lactated Ringer's solution would usually be ordered for this patient. The other orders are appropriate for a postoperative patient with gastric suction.

The nurse notes that a patient who was admitted with diabetic ketoacidosis has rapid, deep respirations. Which action should the nurse take? a. Notify the patient's health care provider. b. Give the prescribed PRN lorazepam (Ativan). c. Start the prescribed PRN oxygen at 2 to 4 L/min. d. Encourage the patient to take deep, slow breaths.

ANS: A Notify the patient's health care provider. The rapid, deep (Kussmaul) respirations indicate a metabolic acidosis and the need for actions such as administration of sodium bicarbonate, which will require a prescription by the health care provider. Oxygen therapy is not indicated because there is no indication that the increased respiratory rate is related to hypoxemia. The respiratory pattern is compensatory, and the patient will not be able to slow the respiratory rate. Ativan administration will slow the respiratory rate and increase the level of acidosis.

When caring for a patient admitted with hyponatremia, which actions will the nurse anticipate taking? a. Restrict patient's oral free water intake. b. Avoid use of electrolyte-containing drinks. c. Infuse a solution of 5% dextrose in 0.45% saline. d. Administer vasopressin (antidiuretic hormone, [ADH]).

ANS: A Restrict patient's oral free water intake. To help improve serum sodium levels, water intake is restricted. Electrolyte-containing beverages will improve the patient's sodium level. Administration of vasopressin or hypotonic IV solutions will decrease the serum sodium level further.

The nurse obtains all of the following assessment data about a patient with deficient fluid volume caused by a massive burn injury. Which of the following assessment data will be of greatest concern? a. The blood pressure is 90/40 mm Hg. b. Urine output is 30 ml over the last hour. c. Oral fluid intake is 100 ml for the last 8 hours. d. There is prolonged skin tenting over the sternum.

ANS: A The blood pressure is 90/40 mm Hg. The blood pressure indicates that the patient may be developing hypovolemic shock as a result of fluid loss. This will require immediate intervention to prevent the complications associated with systemic hypoperfusion. The poor oral intake, decreased urine output, and skin tenting all indicate the need for increasing the patient's fluid intake but not as urgently as the hypotension.

Which assessment finding about a patient who has a serum calcium level of 7.0 mEq/L is most important for the nurse to report to the health care provider? a. The patient is experiencing laryngeal stridor. b. The patient complains of generalized fatigue. c. The patient's bowels have not moved for 4 days. d. The patient has numbness and tingling of the lips.

ANS: A The patient is experiencing laryngeal stridor. Laryngeal stridor may lead to respiratory arrest and requires rapid action to correct the patient's calcium level. The other data also are consistent with hypocalcemia, but do not indicate a need for immediate action.

The nurse in the outpatient clinic who notes that a patient has a decreased magnesium level should ask the patient about a. daily alcohol intake. b. intake of dietary protein. c. multivitamin/mineral use. d. use of over-the-counter (OTC) laxatives.

ANS: A daily alcohol intake. Hypomagnesemia is associated with alcoholism. Protein intake would not have a significant effect on magnesium level. OTC laxatives (such as milk of magnesia) and use of multivitamin/mineral supplements would tend to increase magnesium level.

When teaching a patient with renal failure about a low phosphate diet, the nurse will include information to restrict a. ingestion of dairy products. b. the amount of high-fat foods. c. the quantity of fruits and juices. d. intake of green, leafy vegetables.

ANS: A ingestion of dairy products. Foods high in phosphate include milk and other dairy products, so these are restricted on low-phosphate diets. Green, leafy vegetables; high-fat foods; and fruits/juices are not high in phosphate and are not restricted.

A patient is receiving 3% NaCl solution for correction of hyponatremia. During administration of the solution, the most important assessment for the nurse to monitor is a. lung sounds. b. urinary output. c. peripheral pulses. d. peripheral edema.

ANS: A lung sounds. Hypertonic solutions cause water retention, so the patient should be monitored for symptoms of fluid excess. Crackles in the lungs may indicate the onset of pulmonary edema and are the most serious of the symptoms of fluid excess listed. Bounding peripheral pulses, peripheral edema, or changes in urine output also are important to monitor when administering hypertonic solutions, but they do not indicate acute respiratory or cardiac decompensation.

A patient has the following arterial blood gas (ABG) results: pH 7.32, PaO2 88 mm Hg, PaCO2 37 mm Hg, and HCO3 16 mEq/L. The nurse interprets these results as a. metabolic acidosis. b. metabolic alkalosis. c. respiratory acidosis. d. respiratory alkalosis.

ANS: A metabolic acidosis. The pH and HCO3 indicate that the patient has a metabolic acidosis. The ABGs are inconsistent with the other responses.

A patient who has been receiving diuretic therapy is admitted to the emergency department with a serum potassium level of 3.1 mEq/L. Of the following medications that the patient has been taking at home, the nurse will be most concerned about a. oral digoxin (Lanoxin) 0.25 mg daily. b. ibuprofen (Motrin) 400 mg every 6 hours. c. metoprolol (Lopressor) 12.5 mg orally daily. d. lantus insulin 24 U subcutaneously every evening.

ANS: A oral digoxin (Lanoxin) 0.25 mg daily Hypokalemia increases the risk for digoxin toxicity, which can cause serious dysrhythmias. The nurse also will need to do more assessment regarding the other medications, but there is not as much concern with the potassium level.

The teaching plan for a patient with acute sinusitis will need to include which of the following interventions (select all that apply)? a. Taking a hot shower will increase sinus drainage and decrease pain. b. Over-the-counter (OTC) antihistamines can be used to relieve congestion and inflammation. c. Saline nasal spray can be made at home and used to wash out secretions. d. Blowing the nose forcefully should be avoided to decrease nosebleed risk. e. You will be more comfortable if you keep your head in an upright position.

ANS: A, B, C, E The steam and heat from a shower will help thin secretions and improve drainage. Antihistamines can be used. Patients can use either OTC sterile saline solutions or home-prepared saline solutions to thin and remove secretions. Maintaining an upright posture decreases sinus pressure and the resulting pain. Blowing the nose after a hot shower or using the saline spray is recommended to expel secretions. DIF: Cognitive Level: Analysis REF: 526-527 OBJ: Special Questions: Alternate Item Format TOP: Nursing Process: Implementation MSC: NCLEX: Physiological Integrity

2. The nurse is reviewing the medical records for five patients who are scheduled for their yearly physical examinations in September. Which patients should receive the inactivated influenza vaccination (select all that apply)? a. A 76-year-old nursing home resident b. A 36-year-old female patient who is pregnant c. A 42-year-old patient who has a 15 pack-year smoking history d. A 30-year-old patient who takes corticosteroids for rheumatoid arthritis e. A 24-year-old patient who has allergies to penicillin and cephalosporins

ANS: A, B, D Current guidelines suggest that healthy individuals between 6 months and age 49 receive intranasal immunization with live, attenuated influenza vaccine. Individuals who are pregnant, residents of nursing homes, or are immunocompromised or who have chronic medical conditions should receive inactivated vaccine by injection. The corticosteroid use by the 30-year-old increases the risk for infection.

1. The clinic nurse is teaching a patient with acute sinusitis. Which interventions should the nurse plan to include in the teaching session (select all that apply)? a. Decongestants can be used to relieve swelling. b. Blowing the nose should be avoided to decrease the nosebleed risk. c. Taking a hot shower will increase sinus drainage and decrease pain. d. Saline nasal spray can be made at home and used to wash out secretions. e. You will be more comfortable if you keep your head in an upright position.

ANS: A, C, D, E The steam and heat from a shower will help thin secretions and improve drainage. Decongestants can be used to relieve swelling. Patients can use either over-the-counter (OTC) sterile saline solutions or home-prepared saline solutions to thin and remove secretions. Maintaining an upright posture decreases sinus pressure and the resulting pain. Blowing the nose after a hot shower or using the saline spray is recommended to expel secretions.

After an acute myocardial infarction (AMI), a patient ambulates in the hospital hallway. When the nurse evaluates the patient's response to the activity, which data would indicate that the exercise level should be decreased? a. O2 saturation drops from 99% to 95%. b. Heart rate increases from 66 to 98 beats/min. c. Respiratory rate goes from 14 to 20 breaths/min. d. Blood pressure (BP) changes from 118/60 to 126/68 mm Hg.

ANS: B A change in heart rate of more than 20 beats over the resting heart rate indicates that the patient should stop and rest. The increases in BP and respiratory rate, and the slight decrease in O2 saturation, are normal responses to exercise. DIF: Cognitive Level: Apply (application)

When a patient is admitted to the PACU, what are the priority interventions the nurse performs? a. Assess the surgical site, no tine presence and character of drainage b. Assess the amount of urine output and the presence of bladder distention c. Assess for airway potency and quality of expirations, and obtain vital signs. d. Review results of intraoperative laboratory values and medications received.

c. Assess for airway potency and quality of expirations, and obtain vital signs. Rationale: Assessment in the postanesthesia care unit (PACU) begins with evaluation of the airway, breathing, and circulation (ABC) status of the patient. Identification of inadequate oxygenation and ventilation or respiratory compromise necessitates prompt intervention.

When assessing a patient's surgical dressing on the first postoperative day, the nurse notes new, bright-red drainage about 5 cm in diameter. In response to this finding, what should the nurse do first? a. Recheck in 1 hour for increased drainage. b. Notify the surgeon of a potential hemorrhage. c. Assess the patient's blood pressure and heart rate. d. Remove the dressing and assess the surgical incision.

c. Assess the patient's blood pressure and heart rate. The first action by the nurse is to gather additional assessment data to form a more complete clinical picture. The nurse can then report all of the findings. Continued reassessment will be done. Agency policy determines whether the nurse may change the dressing for the first time or simply reinforce it.

Which of these findings is the best indicator that the fluid resuscitation for a patient with hypovolemic shock has been successful? a. Hemoglobin is within normal limits. b. Urine output is 60 mL over the last hour. c. Pulmonary artery wedge pressure (PAWP) is normal. d. Mean arterial pressure (MAP) is 65 mm Hg.

ANS: B Assessment of end organ perfusion, such as an adequate urine output, is the best indicator that fluid resuscitation has been successful. The hemoglobin level, PAWP, and MAP are useful in determining the effects of fluid administration, but they are not as useful as data indicating good organ perfusion.

When developing a teaching plan for a 61-yr-old patient with multiple risk factors for coronary artery disease (CAD), the nurse should focus primarily on the a. family history of coronary artery disease. b. elevated low-density lipoprotein (LDL) level. c. increased risk associated with the patient's gender. d. increased risk of cardiovascular disease as people age.

ANS: B Because family history, gender, and age are nonmodifiable risk factors, the nurse should focus on the patient's LDL level. Decreases in LDL will help reduce the patient's risk for developing CAD. DIF: Cognitive Level: Apply (application)

In caring for the postoperative patient on the clinical unit after transfer from the PACU, which care can be delegated to the unlicensed assistive personnel (UAP)? a. Monitor the patient's pain. b. Do the admission vital signs. c. Assist the patient to take deep breaths and cough. d. Change the dressing when there is excess drainage.

c. Assist the patient to take deep breaths and cough. The UAP can encourage and assist the patient to do deep breathing and coughing exercises and report complaints of pain to the nurse caring for the patient. The RN should do the admission vital signs for the patient transferring to the clinical unit from the PACU. The LPN or RN will monitor and treat the patient's pain and change the dressings.

3. Which words are most likely to be used to describe neuropathic pain (select all that apply)? a. Dull b. Mild c. Burning d. Shooting e. Shock-like

c. Burning

To improve the physical activity level for a mildly obese 71-yr-old patient, which action should the nurse plan to take? a. Stress that weight loss is a major benefit of increased exercise. b. Determine what kind of physical activities the patient usually enjoys. c. Tell the patient that older adults should exercise for no more than 20 minutes at a time. d. Teach the patient to include a short warm-up period at the beginning of physical activity.

ANS: B Because patients are more likely to continue physical activities that they already enjoy, the nurse will plan to ask the patient about preferred activities. The goal for older adults is 30 minutes of moderate activity on most days. Older adults should plan for a longer warm-up period. Benefits of exercises, such as improved activity tolerance, should be emphasized rather than aiming for significant weight loss in older mildly obese adults. DIF: Cognitive Level: Apply (application)

When developing a teaching plan for a patient with a 42 pack-year history of cigarette smoking, it will be most important for the nurse to include information about a. computed tomography (CT) screening for lung cancer. b. options for smoking cessation. c. reasons for annual sputum cytology testing. d. erlotinib (Tarceva) therapy to prevent tumor risk.

ANS: B Because smoking is the major cause of lung cancer, the most important role for the nurse is educating patients about the benefits of and means of smoking cessation. Early screening of at-risk patients using sputum cytology, chest x-ray, or CT scanning has not been effective in reducing mortality. Erlotinib may be used in patients who have lung cancer but not to reduce risk for developing tumors. DIF: Cognitive Level: Application REF: 563 | 565 TOP: Nursing Process: Planning MSC: NCLEX: Health Promotion and Maintenance

A patient comes to the clinic complaining of frequent, watery stools for the last 2 days. Which action should the nurse take first? a. Obtain the baseline weight. b. Check the patient's blood pressure. c. Draw blood for serum electrolyte levels. d. Ask about any extremity numbness or tingling.

ANS: B Because the patient's history suggests that fluid volume deficit may be a problem, assessment for adequate circulation is the highest priority. The other actions are also appropriate, but are not as essential as determining the patient's perfusion status

After receiving a change-of-shift report, which patient should the nurse assess first? a. The patient who has multiple leg wounds with eschar to be debrided b. The patient receiving chemotherapy who has a temperature of 102° F c. The patient who requires analgesics before a scheduled dressing change d. The newly admitted patient with a stage IV pressure ulcer on the coccyx

ANS: B Chemotherapy is an immunosuppressant. Even a low fever in an immunosuppressed patient is a sign of serious infection and should be treated immediately with cultures and rapid initiation of antibiotic therapy. The nurse should assess the other patients as soon as possible after assessing and implementing appropriate care for the immunosuppressed patient. DIF: Cognitive Level: Analyze (analysis)

The nurse is admitting a patient who has chest pain. Which assessment data suggest that the pain is caused by an acute myocardial infarction (AMI)? a. The pain increases with deep breathing. b. The pain has lasted longer than 30 minutes. c. The pain is relieved after the patient takes nitroglycerin. d. The pain is reproducible when the patient raises the arms.

ANS: B Chest pain that lasts for 20 minutes or more is characteristic of AMI. Changes in pain that occur with raising the arms or with deep breathing are more typical of musculoskeletal pain or pericarditis. Stable angina is usually relieved when the patient takes nitroglycerin. DIF: Cognitive Level: Apply (application)

1. Which statement by a nurse to a patient newly diagnosed with type 2 diabetes is correct? a. Insulin is not used to control blood glucose in patients with type 2 diabetes. b. Complications of type 2 diabetes are less serious than those of type 1 diabetes. c. Changes in diet and exercise may control blood glucose levels in type 2 diabetes. d. Type 2 diabetes is usually diagnosed when the patient is admitted with a hyperglycemic coma.

c. Changes in diet and exercise may control blood glucose levels in type 2 diabetes.

When caring for a patient who is recovering from a sudden cardiac death (SCD) event and has no evidence of an acute myocardial infarction (AMI), the nurse will anticipate teaching the patient that a. sudden cardiac death events rarely reoccur. b. additional diagnostic testing will be required. c. long-term anticoagulation therapy will be needed. d. limiting physical activity will prevent future SCD events.

ANS: B Diagnostic testing (e.g., stress test, Holter monitor, electrophysiologic studies, cardiac catheterization) is used to determine the possible cause of the SCD and treatment options. SCD is likely to recur. Anticoagulation therapy will not have any effect on the incidence of SCD, and SCD can occur even when the patient is resting. DIF: Cognitive Level: Apply (application)

When caring for a patient with renal failure on a low phosphate diet, the nurse will inform unlicensed assistive personnel (UAP) to remove which food from the patient's food tray? a. Grape juice b. Milk carton c. Mixed green salad d. Fried chicken breast

ANS: B Foods high in phosphate include milk and other dairy products, so these are restricted on low-phosphate diets. Green, leafy vegetables; high-fat foods; and fruits/juices are not high in phosphate and are not restricted

After receiving report on the following patients, which patient should the nurse assess first? a. Patient with rheumatic fever who has sharp chest pain with a deep breath b. Patient with acute aortic regurgitation whose blood pressure is 86/54 mm Hg c. Patient with infective endocarditis who has a murmur and splinter hemorrhages d. Patient with dilated cardiomyopathy who has bilateral crackles at the lung bases

ANS: B Hypotension in patients with acute aortic regurgitation may indicate cardiogenic shock. The nurse should immediately assess this patient for other findings such as dyspnea or chest pain. The findings in the other patients are typical of their diagnoses and do not indicate a need for urgent assessment and intervention.

IV potassium chloride (KCl) 60 mEq is prescribed for treatment of a patient with severe hypokalemia. Which action should the nurse take? a. Administer the KCl as a rapid IV bolus. b. Infuse the KCl at a rate of 10 mEq/hour. c. Only give the KCl through a central venous line. d. Discontinue cardiac monitoring during the infusion.

ANS: B IV KCl is administered at a maximal rate of 10 mEq/hr. Rapid IV infusion of KCl can cause cardiac arrest. Although the preferred concentration for KCl is no more than 40 mEq/L, concentrations up to 80 mEq/L may be used for some patients. KCl can cause inflammation of peripheral veins, but it can be administered by this route. Cardiac monitoring should be continued while patient is receiving potassium because of the risk for dysrhythmias

During assessment of the chest in a patient with pneumococcal pneumonia, the nurse would expect to find a. vesicular breath sounds. b. increased tactile fremitus. c. dry, nonproductive cough. d. hyperresonance to percussion.

ANS: B Increased tactile fremitus over the area of pulmonary consolidation is expected with bacterial pneumonias. Dullness to percussion would be expected. Pneumococcal pneumonia typically presents with a loose, productive cough. Adventitious breath sounds such as crackles and wheezes are typical. DIF: Cognitive Level: Application REF: 549 TOP: Nursing Process: Assessment MSC: NCLEX: Physiological Integrity

Which nursing intervention is likely to be most effective when assisting the patient with coronary artery disease to make appropriate dietary changes? a. Inform the patient about a diet containing no saturated fat and minimal salt. b. Help the patient modify favorite high-fat recipes by using monounsaturated oils. c. Emphasize the increased risk for heart problems unless the patient makes the dietary changes. d. Give the patient a list of low-sodium, low-cholesterol foods that should be included in the diet.

ANS: B Lifestyle changes are more likely to be successful when consideration is given to the patient's values and preferences. The highest percentage of calories from fat should come from monounsaturated or polyunsaturated fats. Although low-sodium and low-cholesterol foods are appropriate, providing the patient with a list alone is not likely to be successful in making dietary changes. Completely removing saturated fat from the diet is not a realistic expectation. Up to 7% of calories in the therapeutic lifestyle changes diet can come from saturated fat. Telling the patient about the increased risk without assisting further with strategies for dietary change is unlikely to be successful. DIF: Cognitive Level: Analyze (analysis)

When inflating the cuff on a tracheostomy tube to the appropriate level, the best action by the nurse will be to a. check the pilot balloon after inflation to ensure that it is firm. b. use a manometer to ensure cuff pressure is at an appropriate level. c. check the amount of cuff pressure ordered by the health care provider. d. fill the balloon until minimal air leakage around the cuff is auscultated.

ANS: B Measurement of cuff pressure using a manometer to ensure that cuff pressure is 20 mm Hg or lower will avoid compression of the tracheal capillaries. A firm pilot balloon indicates that the cuff is inflated but does not assess for overinflation. A health care provider's order is not required to determine safe cuff pressure. A minimal leak technique is an alternate means for cuff inflation, but this technique does allow a small air leak around the cuff and increases the risk for aspiration. DIF: Cognitive Level: Application REF: 530 TOP: Nursing Process: Implementation MSC: NCLEX: Physiological Integrity

23. The health care provider suspects the Somogyi effect in a 50-yr-old patient whose 6:00 AM blood glucose is 230 mg/dL. Which action will the nurse teach the patient to take? a. Avoid snacking at bedtime. b. Increase the rapid-acting insulin dose. c. Check the blood glucose during the night d. Administer a larger dose of long-acting insulin.

c. Check the blood glucose during the night - If the Somogyi effect is causing the patient's increased morning glucose level, the patient will experience hypoglycemia between 2:00 and 4:00 AM. The dose of insulin will be reduced, rather than increased. A bedtime snack is used to prevent hypoglycemic episodes during the night.

Which assessment information obtained by the nurse when caring for a patient who has just had a thoracentesis is most important to communicate to the health care provider? a. BP is 150/90 mm Hg. b. Oxygen saturation is 89%. c. Pain level is 5/10 with a deep breath. d. Respiratory rate is 24 when lying flat.

ANS: B Oxygen saturation would be expected to improve after a thoracentesis. A saturation of 89% indicates that a complication such as pneumothorax may be occurring. The other assessment data also indicate a need for ongoing assessment or intervention, but the low oxygen saturation is the priority. DIF: Cognitive Level: Application REF: 576 OBJ: Special Questions: Prioritization TOP: Nursing Process: Assessment MSC: NCLEX: Physiological Integrity

Which of the following is the most reliable way to monitor a patient's fluid status? a. I&O b. Skin turgor c. Daily weights d. Lung sounds

c. Daily weights are the best way to monitor fluid imbalances. They are easier to monitor accurately than intake and output.

The nurse suspects that the patient with stable angina is experiencing a side effect of the prescribed drug metoprolol (Lopressor) if the a. patient is restless and agitated. b. blood pressure is 90/54 mm Hg. c. patient complains about feeling anxious. d. heart monitor shows normal sinus rhythm.

ANS: B Patients taking -adrenergic blockers should be monitored for hypotension and bradycardia. Because this class of medication inhibits the sympathetic nervous system, restlessness, agitation, hypertension, and anxiety will not be side effects. Normal sinus rhythm is a normal and expected heart rhythm. DIF: Cognitive Level: Apply (application)

After the nurse teaches the patient about the use of carvedilol (Coreg) in preventing anginal episodes, which statement by a patient indicates that the teaching has been effective? a. "Carvedilol will help my heart muscle work harder." b. "It is important not to suddenly stop taking the carvedilol." c. "I can expect to feel short of breath when taking carvedilol." d. "Carvedilol will increase the blood flow to my heart muscle."

ANS: B Patients who have been taking -adrenergic blockers can develop intense and frequent angina if the medication is suddenly discontinued. Carvedilol (Coreg) decreases myocardial contractility. Shortness of breath that occurs when taking -adrenergic blockers for angina may be due to bronchospasm and should be reported to the health care provider. Carvedilol works by decreasing myocardial O2 demand, not by increasing blood flow to the coronary arteries. DIF: Cognitive Level: Apply (application)

The nurse is performing tuberculosis (TB) screening in a clinic that has many patients who have immigrated to the United States. Before doing a TB skin test on a patient, which question is most important for the nurse to ask? a. "Is there any family history of TB?" b. "Have you received the bacille Calmette-Guérin (BCG) vaccine for TB?" c. "How long have you lived in the United States?" d. "Do you take any over-the-counter (OTC) medications?"

ANS: B Patients who have received the BCG vaccine will have a positive Mantoux test. Another method for screening (such as a chest x-ray) will need to be used in determining whether the patient has a TB infection. The other information also may be valuable but is not as pertinent to the decision about doing TB skin testing. DIF: Cognitive Level: Application REF: 557 OBJ: Special Questions: Prioritization TOP: Nursing Process: Assessment MSC: NCLEX: Physiological Integrity

After reviewing a patient's history, vital signs, physical assessment, and laboratory data, which information shown in the accompanying figure is most important for the nurse to communicate to the health care provider? a. Hyperglycemia c. Q waves on ECG b. Bilateral crackles d. Elevated troponin

ANS: B Pulmonary congestion suggests that the patient may be developing heart failure, a complication of myocardial infarction (MI). Hyperglycemia is common after MI because of the inflammatory process that occurs with tissue necrosis. Troponin levels will be elevated for several days after MI. Q waves often develop with ST-segment-elevation MI. DIF: Cognitive Level: Analyze (analysis)

A patient who is recovering from an acute myocardial infarction (AMI) asks the nurse when sexual intercourse can be resumed. Which response by the nurse is best? a. "Most patients are able to enjoy intercourse without any complications." b. "Sexual activity uses about as much energy as climbing two flights of stairs." c. "The doctor will provide sexual guidelines when your heart is strong enough." d. "Holding and cuddling are good ways to maintain intimacy after a heart attack."

ANS: B Sexual activity places about as much physical stress on the cardiovascular system as most moderate-energy activities such as climbing two flights of stairs. The other responses do not directly address the patient's question or may not be accurate for this patient. DIF: Cognitive Level: Apply (application)

A patient with primary pulmonary hypertension (PPH) is receiving nifedipine (Procardia). The nurse will evaluate that the treatment is effective if a. the BP is less than 140/90 mm Hg. b. the patient reports decreased exertional dyspnea. c. the heart rate is between 60 and 100 beats/minute. d. the patient's chest x-ray indicates clear lung fields.

ANS: B Since a major symptom of PPH is exertional dyspnea, an improvement in this symptom would indicate that the medication was effective. Nifedipine will affect BP and heart rate, but these parameters would not be used to monitor effectiveness of therapy for a patient with PPH. The chest x-ray will show clear lung fields even if the therapy is not effective. DIF: Cognitive Level: Application REF: 582 TOP: Nursing Process: Evaluation MSC: NCLEX: Physiological Integrity

An RN is observing a nursing student who is suctioning a hospitalized patient with a tracheostomy in place. Which action by the student requires the RN to intervene? a. The student preoxygenates the patient for 1 minute before suctioning. b. The student puts on clean gloves and uses a sterile catheter to suction. c. The student inserts the catheter about 5 inches into the tracheostomy tube. d. The student applies suction for 10 seconds while withdrawing the catheter.

ANS: B Sterile gloves and a sterile catheter are used when suctioning a tracheostomy. The other student actions do not require intervention by the RN. Although the patient may not need 1 minute of preoxygenation, this would not be unsafe. Suctioning for 10 seconds is appropriate. The length of catheter that should be inserted depends on the length of the tracheostomy tube, but 5 inches would be appropriate for most adult patients. DIF: Cognitive Level: Comprehension REF: 530 TOP: Nursing Process: Implementation MSC: NCLEX: Safe and Effective Care Environment

Which of these nursing actions can the RN working in a long-term care facility delegate to an experienced LPN/LVN who is caring for a patient with a permanent tracheostomy? a. Assessing the patient's risk for aspiration b. Suctioning the tracheostomy when needed c. Educating the patient about self-care of the tracheostomy d. Determining the need for replacement of the tracheostomy tube

ANS: B Suctioning of a stable patient can be delegated to LPN/LVNs. Assessments and patient teaching should be done by the RN. DIF: Cognitive Level: Application REF: 532-534 | 542 OBJ: Special Questions: Delegation TOP: Nursing Process: Planning

17. Which nursing action could the registered nurse (RN) working in a skilled care hospital unit delegate to an experienced licensed practical/vocational nurse (LPN/LVN) caring for a patient with a permanent tracheostomy? a. Assess the patient's risk for aspiration. b. Suction the tracheostomy when needed. c. Teach the patient about self-care of the tracheostomy. d. Determine the need for replacement of the tracheostomy tube.

ANS: B Suctioning of a stable patient can be delegated to LPNs/LVNs. Patient assessment and patient teaching should be done by the RN.

Which assessment finding by the nurse caring for a patient who has had coronary artery bypass grafting using a right radial artery graft is most important to communicate to the health care provider? a. Complaints of incisional chest pain b. Pallor and weakness of the right hand c. Fine crackles heard at both lung bases d. Redness on both sides of the sternal incision

ANS: B The changes in the right hand indicate compromised blood flow, which requires immediate evaluation and actions such as prescribed calcium channel blockers or surgery. The other changes are expected or require nursing interventions. DIF: Cognitive Level: Analyze (analysis)

16. The nurse obtains the following assessment data on an older patient who has influenza. Which information will be most important for the nurse to communicate to the health care provider? a. Fever of 100.4° F (38° C) b. Diffuse crackles in the lungs c. Sore throat and frequent cough d. Myalgia and persistent headache

ANS: B The crackles indicate that the patient may be developing pneumonia, a common complication of influenza, which would require aggressive treatment. Myalgia, headache, mild temperature elevation, and sore throat with cough are typical manifestations of influenza and are treated with supportive care measures such as over-the-counter (OTC) pain relievers and increased fluid intake.

The nurse obtains the following assessment data in a 76-year-old patient who has influenza. Which information will be most important to communicate to the health care provider? a. Fever of 100.4° F (38° C) b. Diffuse crackles in the lungs c. Sore throat and frequent cough d. Myalgia and persistent headache

ANS: B The crackles indicate that the patient may be developing pneumonia, a common complication of influenza, which would require aggressive treatment. Myalgia, headache, mild temperature elevation, and sore throat with cough are typical symptoms of influenza and are treated with supportive care measures such as over-the-counter (OTC) pain relievers and increased fluid intake.

14. Following a laryngectomy a patient coughs violently during suctioning and dislodges the tracheostomy tube. Which action should the nurse take first? a. Cover stoma with sterile gauze and ventilate through stoma. b. Attempt to reinsert the tracheostomy tube with the obturator in place. c. Assess the patient's oxygen saturation and notify the health care provider. d. Ventilate the patient with a manual bag and face mask until the health care provider arrives.

ANS: B The first action should be to attempt to reinsert the tracheostomy tube to maintain the patient's airway. Assessing the patient's oxygenation is an important action, but it is not the most appropriate first action in this situation. Covering the stoma with a dressing and manually ventilating the patient may be an appropriate action if the nurse is unable to reinsert the tracheostomy tube. Ventilating with a facemask is not appropriate for a patient with a total laryngectomy because there is a complete separation between the upper airway and the trachea.

Which action should the nurse take first when a patient develops a nosebleed? a. Pack both nares tightly with 1/2-inch ribbon gauze. b. Pinch the lower portion of the nose for 10 minutes. c. Prepare supplies that will be needed for cauterization. d. Apply ice compresses over the patient's nose and cheeks.

ANS: B The first nursing action for epistaxis is to apply direct pressure by pinching the nostrils. Application of cold packs may decrease blood flow to the area somewhat, but will not be sufficient to stop bleeding. Cauterization or nasal packing may be needed if pressure to the nares does not stop bleeding, but these are not the first actions to take for nosebleed. DIF: Cognitive Level: Application REF: 520-521 OBJ: Special Questions: Prioritization TOP: Nursing Process: Implementation MSC: NCLEX: Physiological Integrity

When titrating IV nitroglycerin for a patient with a myocardial infarction (MI), which action will the nurse take to evaluate the effectiveness of the drug? a. Monitor heart rate. c. Check blood pressure. b. Ask about chest pain. d. Observe for dysrhythmias.

ANS: B The goal of IV nitroglycerin administration in MI is relief of chest pain by improving the balance between myocardial oxygen supply and demand. The nurse will also monitor heart rate and blood pressure and observe for dysrhythmias, but these parameters will not indicate whether the medication is effective. DIF: Cognitive Level: Apply (application)

Which patient at the cardiovascular clinic requires the most immediate action by the nurse? a. Patient with type 2 diabetes whose current blood glucose level is 145 mg/dL b. Patient with stable angina whose chest pain has recently increased in frequency c. Patient with familial hypercholesterolemia and a total cholesterol of 465 mg/dL d. Patient with chronic hypertension whose blood pressure today is 172/98 mm Hg

ANS: B The history of more frequent chest pain suggests that the patient may have unstable angina, which is part of the acute coronary syndrome spectrum. This will require rapid implementation of actions such as cardiac catheterization and possible percutaneous coronary intervention. The data about the other patients suggest that their conditions are stable. DIF: Cognitive Level: Analyze (analysis)

The nurse assesses a patient's surgical wound on the first postoperative day and notes redness and warmth around the incision. Which action by the nurse is appropriate? a. Obtain wound cultures. c. Notify the health care provider. b. Document the assessment. d. Assess the wound every 2 hours.

ANS: B The incisional redness and warmth are indicators of the normal initial (inflammatory) stage of wound healing by primary intention. The nurse should document the wound appearance and continue to monitor the wound. Notification of the health care provider, assessment every 2 hours, and obtaining wound cultures are not indicated because the healing is progressing normally. DIF: Cognitive Level: Apply (application)

The health care provider inserts a chest tube in a patient with a hemopneumothorax. When monitoring the patient after the chest tube placement, the nurse will be most concerned about a. a large air leak in the water-seal chamber. b. 400 mL of blood in the collection chamber. c. complaint of pain with each deep inspiration. d. subcutaneous emphysema at the insertion site.

ANS: B The large amount of blood may indicate that the patient is in danger of developing hypovolemic shock. A large air leak would be expected immediately after chest tube placement for pneumothorax. The pain should be treated but is not as urgent a concern as the possibility of continued hemorrhage. Subcutaneous emphysema should be monitored but is not unusual in a patient with pneumothorax. DIF: Cognitive Level: Application REF: 572 TOP: Nursing Process: Assessment MSC: NCLEX: Physiological Integrity

When the nurse is caring for a patient who has had a total laryngectomy and radical neck dissection during the first 24 hours after surgery, what is the priority nursing action? a. Monitor for bleeding. b. Assess breath sounds. c. Clean the inner cannula every 8 hours. d. Avoid changing the tracheostomy ties.

ANS: B The most important goals posttracheotomy are to maintain the airway and ensure adequate oxygenation. Assessment of the breath sounds is the priority action. Maintenance of the tracheostomy ties, cleaning the inner cannula, and checking for bleeding also are appropriate nursing actions but are not of as high a priority. DIF: Cognitive Level: Application REF: 538-541 OBJ: Special Questions: Prioritization TOP: Nursing Process: Implementation MSC: NCLEX: Physiological Integrity

An older adult patient who is malnourished presents to the emergency department with a serum protein level of 5.2 g/dL. The nurse would expect which clinical manifestation? a. Pallor b. Edema c. Confusion d. Restlessness

ANS: B The normal range for total protein is 6.4 to 8.3 g/dL. Low serum protein levels cause a decrease in plasma oncotic pressure and allow fluid to remain in interstitial tissues, causing edema. Confusion, restlessness, and pallor are not associated with low serum protein levels

A patient who is being admitted to the emergency department with intermittent chest pain gives the following list of daily medications to the nurse. Which medication has the most immediate implications for the patient's care? a. captopril c. furosemide (Lasix) b. sildenafil (Viagra) d. warfarin (Coumadin)

ANS: B The nurse will need to avoid giving nitrates to the patient because nitrate administration is contraindicated in patients who are using sildenafil because of the risk of severe hypotension caused by vasodilation. The other home medications should also be documented and reported to the health care provider but do not have as immediate an impact on decisions about the patient's treatment. DIF: Cognitive Level: Analyze (analysis)

A patient with newly diagnosed lung cancer tells the nurse, "I think I am going to die pretty soon." Which response by the nurse is best? a. "Would you like to talk to the hospital chaplain about your feelings?" b. "Can you tell me what it is that makes you think you will die so soon?" c. "Are you afraid that the treatment for your cancer will not be effective?" d. "Do you think that taking an antidepressant medication would be helpful?"

ANS: B The nurse's initial response should be to collect more assessment data about the patient's statement. The answer beginning "Can you tell me what it is" is the most open-ended question and will offer the best opportunity for obtaining more data. The answer beginning, "Are you afraid" implies that the patient thinks that the cancer will be immediately fatal, although the patient's statement may not be related to the cancer diagnosis. The remaining two answers offer interventions that may be helpful to the patient, but more assessment is needed to determine whether these interventions are appropriate. DIF: Cognitive Level: Application REF: 565 TOP: Nursing Process: Implementation MSC: NCLEX: Psychosocial Integrity

Three days after experiencing a myocardial infarction (MI), a patient who is scheduled for discharge asks for assistance with hygiene activities, saying, "I am too nervous about my heart to be alone while I get washed up." Based on this information, which nursing diagnosis is appropriate? a. Activity intolerance related to weakness b. Anxiety related to change in health status c. Denial related to lack of acceptance of the MI d. Altered body image related to cardiac disease

ANS: B The patient data indicate anxiety about the impact of the MI is a concern. The other nursing diagnoses may be appropriate for some patients after an MI, but the data for this patient do not support denial, activity intolerance, or altered body image. DIF: Cognitive Level: Apply (application)

After receiving the following information about four patients during change-of-shift report, which patient should the nurse assess first? a. Patient with acute pericarditis who has a pericardial friction rub b. Patient who has just returned to the unit after balloon valvuloplasty c. Patient who has hypertrophic cardiomyopathy and a heart rate of 116 d. Patient with a mitral valve replacement who has an anticoagulant scheduled

ANS: B The patient who has just arrived after balloon valvuloplasty will need assessment for complications such as bleeding and hypotension. The information about the other patients is consistent with their diagnoses and does not indicate any complications or need for urgent assessment or intervention.

22. When assessing a patient with a sore throat, the nurse notes anterior cervical lymph node swelling, a temperature of 101.6° F (38.7° C), and yellow patches on the tonsils. Which action will the nurse anticipate taking? a. Teach the patient about the use of expectorants. b. Use a swab to obtain a sample for a rapid strep antigen test. c. Discuss the need to rinse the mouth out after using any inhalers. d. Teach the patient to avoid use of nonsteroidal antiinflammatory drugs (NSAIDs).

ANS: B The patient's clinical manifestations are consistent with streptococcal pharyngitis and the nurse will anticipate the need for a rapid strep antigen test and/or cultures. Because patients with streptococcal pharyngitis usually do not have a cough, use of expectorants will not be anticipated. Rinsing the mouth out after inhaler use may prevent fungal oral infections, but the patient's assessment data are not consistent with a fungal infection. NSAIDs are frequently prescribed for pain and fever relief with pharyngitis.

A patient who had a transverse colectomy for diverticulosis 18 hours ago has nasogastric suction and is complaining of anxiety and incisional pain. The patient's respiratory rate is 32 breaths/minute and the arterial blood gases (ABGs) indicate respiratory alkalosis. Which action should the nurse take first? a. Discontinue the nasogastric suction. b. Give the patient the PRN IV morphine sulfate 4 mg. c. Notify the health care provider about the ABG results. d. Teach the patient how to take slow, deep breaths when anxious.

ANS: B The patient's respiratory alkalosis is caused by the increased respiratory rate associated with pain and anxiety. The nurse's first action should be to medicate the patient for pain. Although the nasogastric suction may contribute to the alkalosis, it is not appropriate to discontinue the tube when the patient needs gastric suction. The health care provider may be notified about the ABGs but is likely to instruct the nurse to medicate for pain. The patient will not be able to take slow, deep breaths when experiencing pain.

43. A few weeks after an 82-yr-old patient with a new diagnosis of type 2 diabetes has been placed on metformin (Glucophage) therapy and taught about appropriate diet and exercise, the home health nurse makes a visit. Which finding should the nurse promptly discuss with the health care provider? a. Hemoglobin A1C level is 7.9%. b. Last eye examination was 18 months ago. c. Glomerular filtration rate is decreased. d. Patient has questions about the prescribed diet.

c. Glomerular filtration rate is decreased. - The decrease in renal function may indicate a need to adjust the dose of metformin or change to a different medication. In older patients, the goal for A1C may be higher in order to avoid complications associated with hypoglycemia. The nurse will plan on scheduling the patient for an eye examination and addressing the questions about diet, but the area for prompt intervention is the patient's decreased renal function.

Which nursing action will be most effective in preventing aspiration pneumonia in patients who are at risk? a. Turn and reposition immobile patients at least every 2 hours. b. Place patients with altered consciousness in side-lying positions. c. Monitor for respiratory symptoms in patients who are immunosuppressed. d. Provide for continuous subglottic aspiration in patients receiving enteral feedings.

ANS: B The risk for aspiration is decreased when patients with a decreased level of consciousness are placed in a side-lying or upright position. Frequent turning prevents pooling of secretions in immobilized patients but will not decrease the risk for aspiration in patients at risk. Monitoring of parameters such as breath sounds and oxygen saturation will help detect pneumonia in immunocompromised patients, but it will not decrease the risk for aspiration. Continuous subglottic suction is recommended for intubated patients but not for all patients receiving enteral feedings. DIF: Cognitive Level: Application REF: 551 TOP: Nursing Process: Implementation MSC: NCLEX: Physiological Integrity

5. A patient with type 2 diabetes is scheduled for a follow-up visit in the clinic several months from now. Which test will the nurse schedule to evaluate the effectiveness of treatment for the patient? a. Fasting blood glucose b. Oral glucose tolerance c. Glycosylated hemoglobin d. Urine dipstick for glucose

c. Glycosylated hemoglobin - The glycosylated hemoglobin (A1C) test shows the overall control of glucose over 90 to 120 days. A fasting blood level indicates only the glucose level at one time. Urine glucose testing is not an accurate reflection of blood glucose level and does not reflect the glucose over a prolonged time. Oral glucose tolerance testing is done to diagnose diabetes but is not used for monitoring glucose control after diabetes has been diagnosed.

When admitting a patient with stage III pressure ulcers on both heels, which information obtained by the nurse will have the most impact on wound healing? a. The patient has had the heel ulcers for 6 months. b. The patient takes oral hypoglycemic agents daily. c. The patient states that the ulcers are very painful. d. The patient has several incisions that formed keloids.

ANS: B The use of oral hypoglycemics indicates diabetes, which can interfere with wound healing. The persistence of the ulcers over the past 6 months is a concern, but changes in care may be effective in promoting healing. Keloids are not disabling or painful, although the cosmetic effects may be distressing for some patients. Actions to reduce the patient's pain will be implemented, but pain does not directly affect wound healing. DIF: Cognitive Level: Analyze (analysis) Apply

When evaluating the effectiveness of preoperative teaching with a patient scheduled for coronary artery bypass graft (CABG) surgery using the internal mammary artery, the nurse determines that additional teaching is needed when the patient says which of the following? a. "They will circulate my blood with a machine during surgery." b. "I will have incisions in my leg where they will remove the vein." c. "They will use an artery near my heart to go around the area that is blocked." d. "I will need to take an aspirin every day after the surgery to keep the graft open."

ANS: B When the internal mammary artery is used there is no need to have a saphenous vein removed from the leg. The other statements by the patient are accurate and indicate that the teaching has been effective. DIF: Cognitive Level: Apply (application)

A patient with a pleural effusion is scheduled for a thoracentesis. Before the procedure, the nurse will plan to a. start a peripheral intravenous line to administer the necessary sedative drugs. b. position the patient sitting upright on the edge of the bed and leaning forward. c. remove the water pitcher and remind the patient not to eat or drink anything for 6 hours. d. instruct the patient about the importance of incentive spirometer use after the procedure.

ANS: B When the patient is sitting up, fluid accumulates in the pleural space at the lung bases and can more easily be located and removed. The lung will expand after the effusion is removed; incentive spirometry is not needed to assure alveolar expansion. The patient does not usually require sedation for the procedure, and there are no restrictions on oral intake because the patient is not sedated or unconscious. DIF: Cognitive Level: Application REF: 576 TOP: Nursing Process: Planning MSC: NCLEX: Physiological Integrity

A patient who has been hospitalized for 2 days has been receiving normal saline IV at 100 ml/hr, has a nasogastric tube to low suction, and is NPO. Which assessment finding by the nurse is the priority to report to the health care provider? a. Serum sodium level of 138 mEq/L (138 mmol/L) b. Gradually decreasing level of consciousness (LOC) c. Oral temperature of 100.1° F with bibasilar lung crackles d. Weight gain of 2 pounds (1 kg) above the admission weight

ANS: B Gradually decreasing level of consciousness (LOC) The patient's history and change in LOC could be indicative of several fluid and electrolyte disturbances: extracellular fluid (ECF) excess, ECF deficit, hyponatremia, hypernatremia, hypokalemia, or metabolic alkalosis. Further diagnostic information will be ordered by the health care provider to determine the cause of the change in LOC and the appropriate interventions. The weight gain, elevated temperature, crackles, and serum sodium level also will be reported, but do not indicate a need for rapid action to avoid complications.

Which of these actions can the nurse who is caring for a critically ill patient with multiple intravenous (IV) lines delegate to an experienced LPN? a. Administer IV antibiotics through the implantable port. b. Monitor the IV sites for redness, swelling, or tenderness. c. Remove the patient's nontunneled subclavian central venous catheter. d. Adjust the flow rate of the 0.9% normal saline in the peripheral IV line.

ANS: B Monitor the IV sites for redness, swelling, or tenderness. An experienced LPN has the education, experience, and scope of practice to monitor IV sites for signs of infection. Administration of medications, adjustment of infusion rates, and removal of central catheters in critically ill patients require RN level education and scope of practice.

A patient with advanced lung cancer is admitted to the emergency department with urinary retention caused by renal calculi. Which of these laboratory values will require the most immediate action by the nurse? a. Arterial blood pH is 7.32. b. Serum calcium is 18 mEq/L. c. Serum potassium is 5.1 mEq/L. d. Arterial oxygen saturation is 91%.

ANS: B Serum calcium is 18 mEq/L. The serum calcium is well above the normal level and puts the patient at risk for cardiac dysrhythmias. The nurse should initiate cardiac monitoring and notify the health care provider. The potassium, oxygen saturation, and pH also are abnormal, and the nurse should notify the health care provider about these values as well, but they are not immediately life-threatening.

The following data are obtained by the nurse when assessing a pregnant patient with eclampsia who is receiving IV magnesium sulfate. Which finding is most important to report to the health care provider immediately? a. The bibasilar breath sounds are decreased. b. The patellar and triceps reflexes are absent. c. The patient has been sleeping most of the day. d. The patient reports feeling "sick to my stomach."

ANS: B The patellar and triceps reflexes are absent. The loss of the deep tendon reflexes indicates that the patient's magnesium level may be reaching toxic levels. Nausea and lethargy also are side effects associated with magnesium elevation and should be reported, but they are not as significant as the loss of deep tendon reflexes. The decreased breath sounds suggest that the patient needs to cough and deep breathe to prevent atelectasis.

Which action will the nurse include in the plan of care for a patient who has a central venous access device (CVAD)? a. Avoid using friction when cleaning around the CVAD insertion site. b. Use the push-pause method to flush the CVAD after giving medications. c. Obtain an order from the health care provider to change CVAD dressing. d. Have the patient turn the head toward the CAVD during injection cap changes.

ANS: B Use the push-pause method to flush the CVAD after giving medications. The push-pause enhances the removal of debris from the CVAD lumen and decreases the risk for clotting. To decrease infection risk, friction should be used when cleaning the CVAD insertion site. The dressing should be changed whenever it becomes damp, loose, or visibly soiled and the patient should turn away from the CVAD during cap changes.

When the nurse is evaluating the fluid balance for a patient admitted for hypovolemia associated with multiple draining wounds, the most accurate assessment to include is a. skin turgor. b. daily weight. c. presence of edema. d. hourly urine output.

ANS: B daily weight. Daily weight is the most easily obtained and accurate means of assessing volume status. Skin turgor varies considerably with age. Considerable excess fluid volume may be present before fluid moves into the interstitial space and causes edema. Hourly urine outputs do not take account of fluid intake or of fluid loss through insensible loss, sweating, or loss from the gastrointestinal tract or wounds.

The home health nurse notes that an elderly patient has a low serum protein level. The nurse will plan to assess for a. pallor. b. edema. c. confusion. d. restlessness.

ANS: B edema. Low serum protein levels cause a decrease in plasma oncotic pressure and allow fluid to remain in interstitial tissues, causing edema. Confusion, restlessness, and pallor are not associated with low serum protein levels.

When caring for an alert and oriented elderly patient with a history of dehydration, the home health nurse will teach the patient to increase fluid intake a. in the late evening hours. b. if the oral mucosa feels dry. c. when the patient feels thirsty. d. as soon as changes in level of consciousness (LOC) occur.

ANS: B if the oral mucosa feels dry. An alert, elderly patient will be able to self-assess for signs of oral dryness such as thick oral secretions or dry-appearing mucosa. The thirst mechanism decreases with age and is not an accurate indicator of volume depletion. Many older patients prefer to restrict fluids slightly in the evening to improve sleep quality. The patient will not be likely to notice and act appropriately when changes in LOC occur.

The nurse is reviewing the charts for five patients who are scheduled for their yearly physical examinations in October. Which of the following patients will require the inactivated influenza vaccination (select all that apply)? a. A 56-year-old patient who is allergic to eggs b. A 36-year-old female patient who is pregnant c. A 42-year-old patient who has a 15 pack-year smoking history d. A 30-year-old patient who takes corticosteroids for rheumatoid arthritis e. A 24-year-old patient who has allergies to penicillin and the cephalosporins

ANS: B, D Current guidelines suggest that healthy individuals between 6 months and age 49 receive intranasal immunization with live, attenuated influenza vaccine. Individuals who are pregnant, have chronic medical conditions, or are immunocompromised should receive inactivated vaccine. The corticosteroid use by the 30-year-old increases the risk for infection. Individuals with egg allergies should not receive inactivated flu vaccine because it is made using eggs. DIF: Cognitive Level: Application REF: 524 | 525 OBJ: Special Questions: Alternate Item Format TOP: Nursing Process: Planning MSC: NCLEX: Health Promotion and Maintenance

When caring for a patient who is hospitalized with active tuberculosis (TB), the nurse observes a family member who is visiting the patient. The nurse will need to intervene if the family member a. washes the hands before entering the patient's room. b. hands the patient a tissue from the box at the bedside. c. puts on a surgical face mask before visiting the patient. d. brings food from a "fast-food" restaurant to the patient.

ANS: C A high-efficiency particulate-absorbing (HEPA) mask, rather than a standard surgical mask, should be used when entering the patient's room because the HEPA mask can filter out 100% of small airborne particles. Hand washing before visiting the patient is not necessary, but there is no reason for the nurse to stop the family member from doing this. Because anorexia and weight loss are frequent problems in patients with TB, bringing food from outside the hospital is appropriate. The family member should wash the hands after handling a tissue that the patient has used, but no precautions are necessary when giving the patient an unused tissue. DIF: Cognitive Level: Application REF: 557 TOP: Nursing Process: Implementation MSC: NCLEX: Physiological Integrity

36. A patient who was admitted with diabetic ketoacidosis secondary to a urinary tract infection has been weaned off an insulin drip 30 minutes ago. The patient reports feeling lightheaded and sweaty. Which action should the nurse take first? a. Infuse dextrose 50% by slow IV push. b. Administer 1 mg glucagon subcutaneously. c. Obtain a glucose reading using a finger stick. d. Have the patient drink 4 ounces of orange juice.

c. Obtain a glucose reading using a finger stick. - The patient's clinical manifestations are consistent with hypoglycemia, and the initial action should be to check the patient's glucose with a finger stick or order a stat blood glucose. If the glucose is low, the patient should ingest a rapid-acting carbohydrate, such as orange juice. Glucagon or dextrose 50% might be given if the patient's symptoms become worse or if the patient is unconscious.

The nurse notes that a patient has incisional pain, a poor cough effort, and scattered rhonchi after a thoracotomy. Which action should the nurse take first? a. Assist the patient to sit up at the bedside. b. Splint the patient's chest during coughing. c. Medicate the patient with the prescribed morphine. d. Have the patient use the prescribed incentive spirometer.

ANS: C A major reason for atelectasis and poor airway clearance in patients after chest surgery is incisional pain (which increases with deep breathing and coughing). The first action by the nurse should be to medicate the patient to minimize incisional pain. The other actions are all appropriate ways to improve airway clearance but should be done after the morphine is given. DIF: Cognitive Level: Application REF: 574 OBJ: Special Questions: Prioritization TOP: Nursing Process: Implementation MSC: NCLEX: Physiological Integrity

5. A patient with a tracheostomy has a new order for a fenestrated tracheostomy tube. Which action should the nurse include in the plan of care in collaboration with the speech therapist? a. Leave the tracheostomy inner cannula inserted at all times. b. Place the decannulation cap in the tube before cuff deflation. c. Assess the ability to swallow before using the fenestrated tube. d. Inflate the tracheostomy cuff during use of the fenestrated tube.

ANS: C Because the cuff is deflated when using a fenestrated tube, the patient's risk for aspiration should be assessed before changing to a fenestrated tracheostomy tube. The decannulation cap is never inserted before cuff deflation because to do so would obstruct the patient's airway. The cuff is deflated and the inner cannula removed to allow air to flow across the patient's vocal cords when using a fenestrated tube.

The nurse is caring for a spontaneously breathing patient who has a tracheostomy. To determine that the patient can protect the airway when eating without having the tracheostomy cuff inflated, the nurse will deflate the cuff and a. ask the patient to say a few sentences. b. monitor for signs of respiratory distress. c. have the patient drink a small amount of grape juice and observe for coughing. d. auscultate the lungs for crackles after having the patient take a few sips of water.

ANS: C Because the cuff is deflated when using a fenestrated tube, the patient's risk for aspiration should be assessed before changing to a fenestrated tracheostomy tube. The decannulation cap is never inserted before cuff deflation because to do so would obstruct the patient's airway. The cuff is deflated and the inner cannula removed to allow air to flow across the patient's vocal cords when using a fenestrated tube. DIF: Cognitive Level: Application REF: 529 | 535 TOP: Nursing Process: Planning MSC: NCLEX: Physiological Integrity

Which information about a patient who has been receiving thrombolytic therapy for an acute myocardial infarction is most important for the nurse to communicate to the health care provider? a. An increase in troponin levels from baseline b. A large bruise at the patient's IV insertion site c. No change in the patient's reported level of chest pain d. A decrease in ST-segment elevation on the electrocardiogram

ANS: C Continued chest pain suggests that the thrombolytic therapy is not effective and that other interventions such as percutaneous coronary intervention may be needed. Bruising is a possible side effect of thrombolytic therapy, but it is not an indication that therapy should be discontinued. The decrease of the ST-segment elevation indicates that thrombolysis is occurring and perfusion is returning to the injured myocardium. An increase in troponin levels is expected with reperfusion and is related to the washout of cardiac biomarkers into the circulation as the blocked vessel is opened. DIF: Cognitive Level: Analyze (analysis)

The nurse recognizes that the goals of teaching regarding the transmission of pulmonary tuberculosis (TB) have been met when the patient with TB a. demonstrates correct use of a nebulizer. b. washes dishes and personal items after use. c. covers the mouth and nose when coughing. d. reports daily to the public health department.

ANS: C Covering the mouth and nose will help decrease airborne transmission of TB. The other actions will not be effective in decreasing the spread of TB. DIF: Cognitive Level: Application REF: 557 TOP: Nursing Process: Evaluation MSC: NCLEX: Health Promotion and Maintenance

A patient who has just been admitted with pneumococcal pneumonia has a temperature of 101.6° F with a frequent cough and is complaining of severe pleuritic chest pain. Which of these prescribed medications should the nurse give first? a. guaifenesin (Robitussin) b. acetaminophen (Tylenol) c. azithromycin (Zithromax) d. codeine phosphate (Codeine)

ANS: C Early initiation of antibiotic therapy has been demonstrated to reduce mortality. The other medications also are appropriate and should be given as soon as possible, but the priority is to start antibiotic therapy. DIF: Cognitive Level: Application REF: 549 OBJ: Special Questions: Prioritization TOP: Nursing Process: Implementation MSC: NCLEX: Physiological Integrity

The long-term care nurse is evaluating the effectiveness of protein supplements for an older resident who has a low serum total protein level. Which assessment finding indicates that the patient's condition has improved? a. Hematocrit 28% b. Absence of skin tenting c. Decreased peripheral edema d. Blood pressure 110/72 mm Hg

ANS: C Edema is caused by low oncotic pressure in individuals with low serum protein levels. The decrease in edema indicates an improvement in the patient's protein status. Good skin turgor is an indicator of fluid balance, not protein status. A low hematocrit could be caused by poor protein intake. Blood pressure does not provide a useful clinical tool for monitoring protein status

When providing preoperative instruction for a patient scheduled for a left pneumonectomy for cancer of the lung, the nurse informs the patient that the postoperative care includes a. positioning on the right side. b. bed rest for the first 24 hours. c. frequent use of an incentive spirometer. d. chest tubes to water-seal chest drainage.

ANS: C Frequent deep breathing and coughing are needed after chest surgery to prevent atelectasis. To promote gas exchange, patients after pneumonectomy are positioned on the surgical side. Chest tubes are not usually used after pneumonectomy because the affected side is allowed to fill with fluid. Early mobilization decreases the risk for postoperative complications such as pneumonia and deep vein thrombosis. DIF: Cognitive Level: Application REF: 573 TOP: Nursing Process: Planning MSC: NCLEX: Physiological Integrity

Heparin is ordered for a patient with a non-ST-segment-elevation myocardial infarction (NSTEMI). What is the purpose of the heparin? a. Heparin enhances platelet aggregation at the plaque site. b. Heparin decreases the size of the coronary artery plaque. c. Heparin prevents the development of new clots in the coronary arteries. d. Heparin dissolves clots that are blocking blood flow in the coronary arteries.

ANS: C Heparin helps prevent the conversion of fibrinogen to fibrin and decreases coronary artery thrombosis. It does not change coronary artery plaque, dissolve already formed clots, or enhance platelet aggregation. DIF: Cognitive Level: Understand (comprehension)

A patient who has a small cell carcinoma of the lung develops syndrome of inappropriate antidiuretic hormone (SIADH). The nurse should notify the health care provider about which assessment finding? a. Reported weight gain b. Serum hematocrit of 42% c. Serum sodium level of 120 mg/dL d. Total urinary output of 280 mL during past 8 hours

ANS: C Hyponatremia is the most important finding to report. SIADH causes water retention and a decrease in serum sodium level. Hyponatremia can cause confusion and other central nervous system effects. A critically low value likely needs to be treated. At least 30 mL/hr of urine output indicates adequate kidney function. The hematocrit level is normal. Weight gain is expected with SIADH because of water retention

A nurse is assessing a newly admitted patient with chronic heart failure who forgot to take prescribed medications and seems confused. The patient complains of "just blowing up" and has peripheral edema and shortness of breath. Which assessment should the nurse complete first? a. Skin turgor b. Heart sounds c. Mental status d. Capillary refill

ANS: C Increases in extracellular fluid (ECF) can lead to swelling of cells in the central nervous system, initially causing confusion, which may progress to coma or seizures. Although skin turgor, capillary refill, and heart sounds also may be affected by increases in ECF, these are signs that do not have as immediate impact on patient outcomes as cerebral edema

10. A patient who had a total laryngectomy has a nursing diagnosis of hopelessness related to loss of control of personal care. Which information obtained by the nurse is the best indicator that this identified problem is resolving? a. The patient lets the spouse provide tracheostomy care. b. The patient allows the nurse to suction the tracheostomy. c. The patient asks how to clean the tracheostomy stoma and tube. d. The patient uses a communication board to request "No Visitors."

ANS: C Independently caring for the laryngectomy tube indicates that the patient has regained control of personal care and hopelessness is at least partially resolved. Letting the nurse and spouse provide care and requesting no visitors may indicate that the patient is still experiencing hopelessness.

A patient who had a total laryngectomy has a nursing diagnosis of hopelessness related to loss of control of personal care. Which information obtained by the nurse is the best indicator that the problem identified in this nursing diagnosis is resolving? a. The patient lets the spouse provide tracheostomy care. b. The patient allows the nurse to suction the tracheostomy. c. The patient asks how to clean the tracheostomy stoma and tube. d. The patient uses a communication board to request "No Visitors."

ANS: C Independently caring for the laryngectomy tube indicates that the patient has regained control of personal care and hopelessness is at least partially resolved. Letting the nurse and spouse provide care and requesting no visitors may indicate that the patient is still experiencing hopelessness. DIF: Cognitive Level: Application REF: 539-540 | 542 TOP: Nursing Process: Evaluation MSC: NCLEX: Psychosocial Integrity

After a patient has undergone a rhinoplasty, which nursing intervention will be included in the plan of care? a. Educate the patient about how to safely remove and reapply nasal packing. b. Reassure the patient that the nose will look normal when the swelling subsides. c. Instruct the patient to keep the head elevated for 48 hours to minimize swelling and pain. d. Teach the patient to use nonsteroidal anti-inflammatory drugs (NSAIDs) for pain control.

ANS: C Maintaining the head in an elevated position will decrease the amount of nasal swelling. NSAIDs increase the risk for postoperative bleeding and should not be used postoperatively. The patient would not be taught to remove or reapply nasal packing, which is usually removed by the surgeon on the day after surgery. Although return to a preinjury appearance is the goal of the surgery, it is not always possible to achieve this result. DIF: Cognitive Level: Application REF: 520 TOP: Nursing Process: Implementation MSC: NCLEX: Physiological Integrity

A young adult patient who is receiving antibiotics for an infected leg wound has a temperature of 101.8° F (38.7° C) The patient reports having no discomfort. Which action by the nurse is appropriate? a. Apply a cooling blanket. b. Notify the health care provider. c. Check the patient's temperature again in 4 hours. d. Give acetaminophen (Tylenol) prescribed PRN for pain.

ANS: C Mild to moderate temperature elevations (<103° F) do not harm young adult patients and may benefit host defense mechanisms. The nurse should continue to monitor the temperature. Antipyretics are not indicated unless the patient is complaining of fever-related symptoms, and the patient does not require analgesics if not reporting discomfort. There is no need to notify the patient's health care provider or to use a cooling blanket for a moderate temperature elevation. DIF: Cognitive Level: Apply (application)

Which statement by a patient who has been hospitalized for pneumonia indicates a good understanding of the discharge instructions given by the nurse? a. "I will call the doctor if I still feel tired after a week." b. "I will need to use home oxygen therapy for 3 months." c. "I will continue to do the deep breathing and coughing exercises at home." d. "I will schedule two appointments for the pneumonia and influenza vaccines."

ANS: C Patients should continue to cough and deep breathe after discharge. Fatigue for several weeks is expected. Home oxygen therapy is not needed with successful treatment of pneumonia. The pneumovax and influenza vaccines can be given at the same time. DIF: Cognitive Level: Application REF: 552 TOP: Nursing Process: Evaluation MSC: NCLEX: Physiological Integrity

Which statement by a patient with restrictive cardiomyopathy indicates that the nurse's discharge teaching about self-management has been most effective? a. "I will avoid taking aspirin or other antiinflammatory drugs." b. "I will need to limit my intake of salt and fluids even in hot weather." c. "I will take antibiotics when my teeth are cleaned at the dental office." d. "I should begin an exercise program that includes things like biking or swimming."

ANS: C Patients with restrictive cardiomyopathy are at risk for infective endocarditis and should use prophylactic antibiotics for any procedure that may cause bacteremia. The other statements indicate a need for more teaching by the nurse. Dehydration and vigorous exercise impair ventricular filling in patients with restrictive cardiomyopathy. There is no need to avoid salt (unless ordered), aspirin, or NSAIDs.

After the home health nurse teaches a patient's family member about how to care for a sacral pressure ulcer, which finding indicates that additional teaching is needed? a. The family member uses a lift sheet to reposition the patient. b. The family member uses clean tap water to clean the wound. c. The family member dries the wound using a hair dryer on a low setting. d. The family member places contaminated dressings in a plastic grocery bag.

ANS: C Pressure ulcers need to be kept moist to facilitate wound healing. The other actions indicate a good understanding of pressure ulcer care. DIF: Cognitive Level: Apply (application)

Diltiazem (Cardizem) is ordered for a patient with newly diagnosed Prinzmetal's (variant) angina. When teaching the patient, the nurse will include the information that diltiazem will a. reduce heart palpitations. b. prevent coronary artery plaque. c. decrease coronary artery spasms. d. increase contractile force of the heart.

ANS: C Prinzmetal's angina is caused by coronary artery spasm. Calcium channel blockers (e.g., diltiazem, amlodipine [Norvasc]) are a first-line therapy for this type of angina. Lipid-lowering drugs help reduce atherosclerosis (i.e., plaque formation), and -adrenergic blockers decrease sympathetic stimulation of the heart (i.e., palpitations). Medications or activities that increase myocardial contractility will increase the incidence of angina by increasing O2 demand. DIF: Cognitive Level: Apply (application)

After the nurse has received change-of-shift report about the following four patients, which patient should be assessed first? a. A 77-year-old patient with tuberculosis (TB) who has four antitubercular medications due in 15 minutes b. A 23-year-old patient with cystic fibrosis who has pulmonary function testing scheduled c. A 46-year-old patient who has a deep vein thrombosis and is complaining of sudden onset shortness of breath. d. A 35-year-old patient who was admitted the previous day with pneumonia and has a temperature of 100.2° F (37.8° C)

ANS: C Sudden onset shortness of breath in a patient with a deep vein thrombosis suggests a pulmonary embolism and requires immediate assessment and actions such as oxygen administration. The other patients also should be assessed as soon as possible, but there is no indication that they may need immediate action to prevent clinical deterioration. DIF: Cognitive Level: Application REF: 577-578 OBJ: Special Questions: Multiple Patients TOP: Nursing Process: Planning MSC: NCLEX: Physiological Integrity

A patient with hyperlipidemia has a new order for colesevelam (Welchol). Which nursing action is appropriate when scheduling this medication? a. Administer the medication at the patient's usual bedtime. b. Have the patient take the colesevelam 1 hour before breakfast. c. Give the patient's other medications 2 hours after colesevelam. d. Have the patient take the dose at the same time as the prescribed aspirin.

ANS: C The bile acid sequestrants interfere with the absorption of many other drugs and giving other medications at the same time should be avoided. Taking an aspirin concurrently with the colesevelam may increase the incidence of gastrointestinal side effects such as heartburn. For maximum effect, colesevelam should be administered with meals. DIF: Cognitive Level: Apply (application)

During the administration of the thrombolytic agent to a patient with an acute myocardial infarction, the nurse should stop the drug infusion if the patient experiences a. bleeding from the gums. b. increase in blood pressure. c. a decrease in level of consciousness. d. a nonsustained episode of ventricular tachycardia.

ANS: C The change in level of consciousness indicates that the patient may be experiencing intracranial bleeding, a possible complication of thrombolytic therapy. Some bleeding of the gums is an expected side effect of the therapy but not an indication to stop infusion of the thrombolytic medication. A decrease in blood pressure could indicate internal bleeding. A nonsustained episode of ventricular tachycardia is a common reperfusion dysrhythmia and may indicate that the therapy is effective. DIF: Cognitive Level: Apply (application)

The nurse obtains the following data when assessing a patient who experienced an ST-segment-elevation myocardial infarction (STEMI) 2 days previously. Which information is most important to report to the health care provider? a. The troponin level is elevated. b. The patient denies having a heart attack. c. Bilateral crackles in the mid-lower lobes. d. Occasional premature atrial contractions (PACs).

ANS: C The crackles indicate that the patient may be developing heart failure, a possible complication of myocardial infarction (MI). The health care provider may need to order medications such as diuretics or angiotensin-converting enzyme inhibitors for the patient. Elevation in troponin level at this time is expected. PACs are not life-threatening dysrhythmias. Denial is a common response in the immediate period after the MI. DIF: Cognitive Level: Analyze (analysis)

A patient is admitted to the emergency department with an open stab wound to the right chest. What is the first action that the nurse should take? a. Position the patient so that the right chest is dependent. b. Keep the head of the patient's bed at no more than 30 degrees elevation. c. Tape a nonporous dressing on three sides over the chest wound. d. Cover the sucking chest wound firmly with an occlusive dressing.

ANS: C The dressing taped on three sides will allow air to escape when intrapleural pressure increases during expiration, but it will prevent air from moving into the pleural space during inspiration. Placing the patient on the right side or covering the chest wound with an occlusive dressing will allow trapped air in the pleural space and cause tension pneumothorax. The head of the bed should be elevated to 30 to 45 degrees to facilitate breathing. DIF: Cognitive Level: Application REF: 567 OBJ: Special Questions: Prioritization TOP: Nursing Process: Implementation MSC: NCLEX: Physiological Integrity

The nurse is caring for a patient with primary pulmonary hypertension who is receiving epoprostenol (Flolan). Which assessment information requires the most immediate action? a. The BP is 98/56 mm Hg. b. The oxygen saturation is 94%. c. The patient's central intravenous line is disconnected. d. The international normalized ratio (INR) is prolonged.

ANS: C The half-life of this drug is 6 minutes, so the nurse will need to restart the infusion as soon as possible to prevent rapid clinical deterioration. The other data also indicate a need for ongoing monitoring or intervention, but the priority action is to reconnect the infusion. DIF: Cognitive Level: Application REF: 581 OBJ: Special Questions: Prioritization TOP: Nursing Process: Assessment MSC: NCLEX: Physiological Integrity

Which action should the nurse take first when a patient complains of acute chest pain and dyspnea soon after insertion of a centrally inserted IV catheter? a. Notify the health care provider. b. Offer reassurance to the patient. c. Auscultate the patient's breath sounds. d. Give the prescribed PRN morphine sulfate IV.

ANS: C The initial action should be to assess the patient further because the history and symptoms are consistent with several possible complications of central line insertion, including embolism and pneumothorax. The other actions may be appropriate, but further assessment of the patient is needed before notifying the health care provider, offering reassurance, or administration of morphine

After receiving change-of-shift report, which patient should the nurse assess first? a. Patient with serum potassium level of 5.0 mEq/L who is complaining of abdominal cramping b. Patient with serum sodium level of 145 mEq/L who has a dry mouth and is asking for a glass of water c. Patient with serum magnesium level of 1.1 mEq/L who has tremors and hyperactive deep tendon reflexes d. Patient with serum phosphorus level of 4.5 mg/dL who has multiple soft tissue calcium-phosphate precipitates

ANS: C The low magnesium level and neuromuscular irritability suggest that the patient may be at risk for seizures. The other patients have mild electrolyte disturbances and/or symptoms that require action, but they are not at risk for life-threatening complications

A young male patient with paraplegia has a stage II sacral pressure ulcer and is being cared for at home by his family. To prevent further tissue damage, what instructions are most important for the nurse to teach the patient and family? a. Change the patient's bedding frequently. b. Apply a hydrocolloid dressing over the ulcer. c. Change the patient's position every 1 to 2 hours. d. Record the size and appearance of the ulcer weekly.

ANS: C The most important intervention is to avoid prolonged pressure on bony prominences by frequent repositioning. The other interventions may also be included in family teaching. DIF: Cognitive Level: Analyze (analysis)

After a patient with right lower-lobe pneumonia has been treated with intravenous (IV) antibiotics for 2 days, which assessment data obtained by the nurse indicates that the treatment has been effective? a. Bronchial breath sounds are heard at the right base. b. The patient coughs up small amounts of green mucus. c. The patient's white blood cell (WBC) count is 9000/µl. d. Increased tactile fremitus is palpable over the right chest.

ANS: C The normal WBC count indicates that the antibiotics have been effective. All the other data suggest that a change in treatment is needed. DIF: Cognitive Level: Application REF: 549 TOP: Nursing Process: Evaluation MSC: NCLEX: Physiological Integrity

3. The nurse discusses management of upper respiratory infections (URI) with a patient who has acute sinusitis. Which statement by the patient indicates that additional teaching is needed? a. "I can take acetaminophen (Tylenol) to treat my discomfort." b. "I will drink lots of juices and other fluids to stay well hydrated." c. "I can use my nasal decongestant spray until the congestion is all gone." d. "I will watch for changes in nasal secretions or the sputum that I cough up."

ANS: C The nurse should clarify that nasal decongestant sprays should be used for no more than 3 days to prevent rebound vasodilation and congestion. The other responses indicate that the teaching has been effective.

After discussing management of upper respiratory infections (URI) with a patient who has acute viral rhinitis, the nurse determines that additional teaching is needed when the patient says a. "I can take acetaminophen (Tylenol) to treat discomfort." b. "I will drink lots of juices and other fluids to stay hydrated." c. "I can use my nasal decongestant spray until the congestion is all gone." d. "I will watch for changes in nasal secretions or the sputum that I cough up."

ANS: C The nurse should clarify that nasal decongestant sprays should be used for no more than 3 days to prevent rebound vasodilation and congestion. The other responses indicate that the teaching has been effective. DIF: Cognitive Level: Application REF: 524 TOP: Nursing Process: Evaluation MSC: NCLEX: Physiological Integrity

After reviewing information shown in the accompanying figure from the medical records of a 43-yr-old patient, which risk factor modification for coronary artery disease should the nurse include in patient teaching? a. Importance of daily physical activity b. Effect of weight loss on blood pressure c. Dietary changes to improve lipid levels d. Cardiac risk associated with previous tobacco use

ANS: C The patient has an elevated low-density lipoprotein cholesterol and low high-density lipoprotein cholesterol, which will increase the risk of coronary artery disease. Although the blood pressure is in the prehypertensive range, the patient's waist circumference and body mass index indicate an appropriate body weight. The risk for coronary artery disease a year after quitting smoking is the same as a nonsmoker. The patient's occupation indicates that daily activity is at the levels suggested by national guidelines. DIF: Cognitive Level: Analyze (analysis)

Which electrocardiographic (ECG) change is most important for the nurse to report to the health care provider when caring for a patient with chest pain? a. Inverted P wave c. ST-segment elevation b. Sinus tachycardia d. First-degree atrioventricular block

ANS: C The patient is likely to be experiencing an ST-segment-elevation myocardial infarction. Immediate therapy with percutaneous coronary intervention or thrombolytic medication is indicated to minimize myocardial damage. The other ECG changes may also suggest a need for therapy but not as rapidly. DIF: Cognitive Level: Analyze (analysis)

In preparation for discharge, the nurse teaches a patient with chronic stable angina how to use the prescribed short-acting and long-acting nitrates. Which patient statement indicates that the teaching has been effective? a. "I will check my pulse rate before I take any nitroglycerin tablets." b. "I will put the nitroglycerin patch on as soon as I get any chest pain." c. "I will stop what I am doing and sit down before I put the nitroglycerin under my tongue." d. "I will be sure to remove the nitroglycerin patch before taking any sublingual nitroglycerin."

ANS: C The patient should sit down before taking the nitroglycerin to decrease cardiac workload and prevent orthostatic hypotension. Transdermal nitrates are used prophylactically rather than to treat acute pain and can be used concurrently with sublingual nitroglycerin. Although the nurse should check blood pressure before giving nitroglycerin, patients do not need to check the pulse rate before taking nitrates. DIF: Cognitive Level: Apply (application)

When caring for a patient with acute coronary syndrome who has returned to the coronary care unit after having angioplasty with stent placement, the nurse obtains the following assessment data. Which data indicate the need for immediate action by the nurse? a. Heart rate 102 beats/min c. Report of severe chest pain b. Pedal pulses 1+ bilaterally d. Blood pressure 103/54 mm Hg

ANS: C The patient's chest pain indicates that restenosis of the coronary artery may be occurring and requires immediate actions, such as administration of oxygen and nitroglycerin, by the nurse. The other information indicates a need for ongoing assessments by the nurse. DIF: Cognitive Level: Analyze (analysis)

A patient with a systemic bacterial infection feels cold and has a shaking chill. Which assessment finding will the nurse expect next? a. Skin flushing c. Rising body temperature b. Muscle cramps d. Decreasing blood pressure

ANS: C The patient's complaints of feeling cold and shivering indicate that the hypothalamic set point for temperature has been increased and the temperature is increasing. Because associated peripheral vasoconstriction and sympathetic nervous system stimulation will occur, skin flushing and hypotension are not expected. Muscle cramps are not expected with chills and shivering or with a rising temperature. DIF: Cognitive Level: Apply (application)

When the nurse is deflating the cuff of a tracheostomy tube to evaluate the patient's ability to swallow, it is important to a. clean the inner cannula of the tracheostomy tube before deflation. b. deflate the cuff during the inhalation phase of the respiratory cycle. c. suction the patient's mouth and trachea before deflation of the cuff. d. insert exactly the same volume of air into the cuff during reinflation.

ANS: C The patient's mouth and trachea should be suctioned before the cuff is deflated to prevent aspiration of oral secretions. The amount of air needed to inflate the cuff varies and is adjusted by measuring cuff pressure or using the minimal leak technique, not by measuring the volume of air removed from the cuff. The cuff is deflated during patient exhalation so that secretions will be forced into the mouth rather than aspirated. There is no need to clean the inner cannula before cuff deflation. DIF: Cognitive Level: Application REF: 534 TOP: Nursing Process: Implementation MSC: NCLEX: Physiological Integrity

A patient recovering from a myocardial infarction (MI) develops chest pain on day 3 that increases when taking a deep breath and is relieved by leaning forward. Which action should the nurse take as focused follow-up on this symptom? a. Assess the feet for pedal edema. b. Palpate the radial pulses bilaterally. c. Auscultate for a pericardial friction rub. d. Check the heart monitor for dysrhythmias.

ANS: C The patient's symptoms are consistent with the development of pericarditis, a possible complication of MI. The other assessments listed are not consistent with the description of the patient's symptoms. DIF: Cognitive Level: Apply (application)

A patient who has chest pain is admitted to the emergency department (ED), and all of the following are ordered. Which one should the nurse arrange to be completed first? a. Chest x-ray c. Electrocardiogram (ECG) b. Troponin level d. Insertion of a peripheral IV

ANS: C The priority for the patient is to determine whether an acute myocardial infarction (AMI) is occurring so that the appropriate therapy can begin as quickly as possible. ECG changes occur very rapidly after coronary artery occlusion, and an ECG should be obtained as soon as possible. Troponin levels will increase after about 3 hours. Data from the chest x-ray may impact the patient's care but are not helpful in determining whether the patient is experiencing a myocardial infarction. Peripheral access will be needed but not before the ECG. DIF: Cognitive Level: Analyze (analysis)

A patient with diabetes mellitus and chronic stable angina has a new order for captopril . The nurse should teach the patient that the primary purpose of captopril is to a. decrease the heart rate. c. prevent changes in heart muscle. b. control blood glucose levels. d. reduce the frequency of chest pain.

ANS: C The purpose for angiotensin-converting enzyme (ACE) inhibitors in patients with chronic stable angina who are at high risk for a cardiac event is to decrease ventricular remodeling. ACE inhibitors do not directly impact angina frequency, blood glucose, or heart rate. DIF: Cognitive Level: Apply (application)

The nurse notes that a patient who was admitted with diabetic ketoacidosis has rapid, deep respirations. Which action should the nurse take? a. Give the prescribed PRN lorazepam (Ativan). b. Start the prescribed PRN oxygen at 2 to 4 L/min. c. Administer the prescribed normal saline bolus and insulin. d. Encourage the patient to take deep, slow breaths with guided imagery.

ANS: C The rapid, deep (Kussmaul) respirations indicate a metabolic acidosis and the need for correction of the acidosis with a saline bolus to prevent hypovolemia followed by insulin administration to allow glucose to reenter the cells. Oxygen therapy is not indicated because there is no indication that the increased respiratory rate is related to hypoxemia. The respiratory pattern is compensatory, and the patient will not be able to slow the respiratory rate. Lorazepam administration will slow the respiratory rate and increase the level of acidosis

A patient's 4 x 3-cm leg wound has a 0.4-cm black area in the center of the wound surrounded by yellow-green semiliquid material. Which dressing should the nurse apply to the wound? a. Dry gauze dressing c. Hydrocolloid dressing b. Nonadherent dressing d. Transparent film dressing

ANS: C The wound requires debridement of the necrotic areas and absorption of the yellow-green slough. A hydrocolloid dressing such as DuoDerm would accomplish these goals. Transparent film dressings are used for clean wounds or approximated surgical incisions. Dry dressings will not debride the necrotic areas. Nonadherent dressings will not absorb wound drainage or debride the wound. DIF: Cognitive Level: Apply (application)

4. A nurse who is caring for patient with a tracheostomy tube in place has just auscultated rhonchi bilaterally. If the patient is unsuccessful in coughing up secretions, what action should the nurse take? a. Encourage increased incentive spirometer use. b. Encourage the patient to increase oral fluid intake. c. Put on sterile gloves and use a sterile catheter to suction. d. Preoxygenate the patient for 3 minutes before suctioning.

ANS: C This patient needs suctioning now to secure a patent airway. Sterile gloves and a sterile catheter are used when suctioning a tracheostomy. Preoxygenation for 3 minutes is not necessary. Incentive spirometer (IS) use opens alveoli and can induce coughing, which can mobilize secretions. However, the patient with a tracheostomy may not be able to use an incentive spirometer. Increasing oral fluid intake would not moisten and help mobilize secretions in a timely manner.

The health care provider writes an order for bacteriologic testing for a patient who has a positive tuberculosis skin test. Which action will the nurse take? a. Repeat the tuberculin skin testing. b. Teach about the reason for the blood tests. c. Obtain consecutive sputum specimens from the patient for 3 days. d. Instruct the patient to expectorate three specimens as soon as possible.

ANS: C Three consecutive sputum specimens are obtained on different days for bacteriologic testing for M. tuberculosis. The patient should not provide all the specimens at once. Blood cultures are not used for tuberculosis testing. Once skin testing is positive, it is not repeated. DIF: Cognitive Level: Application REF: 555 TOP: Nursing Process: Implementation MSC: NCLEX: Physiological Integrity

A patient with ST-segment elevation in three contiguous electrocardiographic leads is admitted to the emergency department and diagnosed as having an ST-segment-elevation myocardial infarction. Which question should the nurse ask to determine whether the patient is a candidate for thrombolytic therapy? a. "Do you have any allergies?" b. "Do you take aspirin on a daily basis?" c. "What time did your chest pain begin?" d. "Can you rate your chest pain using a 0 to 10 scale?"

ANS: C Thrombolytic therapy should be started within 6 hours of the onset of the myocardial infarction, so the time at which the chest pain started is a major determinant of the appropriateness of this treatment. The other information is not a factor in the decision about thrombolytic therapy. DIF: Cognitive Level: Apply (application)

The nurse is caring for a patient with diabetes who had abdominal surgery 3 days ago. Which finding is most important for the nurse to report to the health care provider? a. Blood glucose of 136 mg/dL b. Oral temperature of 101° F (38.3° C) c. Separation of the proximal wound edges d. Patient complaint of increased incisional pain

ANS: C Wound separation 3 days postoperatively indicates possible wound dehiscence and should be immediately reported to the health care provider. The other findings will also be reported but do not require intervention as rapidly. DIF: Cognitive Level: Analyze (analysis)

Which information obtained by the nurse about a patient who has been diagnosed with both human immunodeficiency virus (HIV) and active tuberculosis (TB) disease is most important to communicate to the health care provider? a. The Mantoux test had an induration of only 8 mm. b. The chest-x-ray showed infiltrates in the upper lobes. c. The patient is being treated with antiretrovirals for HIV infection. d. The patient has a cough that is productive of blood-tinged mucus.

ANS: C Drug interactions can occur between the antiretrovirals used to treat HIV infection and the medications used to treat tuberculosis. The other data are expected in a patient with HIV and TB disease. DIF: Cognitive Level: Application REF: 556 OBJ: Special Questions: Prioritization TOP: Nursing Process: Assessment

A patient receiving isoosmolar continuous tube feedings develops restlessness, agitation, and weakness. Which laboratory result is most important to report to the health care provider? a. K+ 3.4 mEq/L (3.4 mmol/L) b. Ca+2 7.8 mg/dl (1.95 mmol/L) c. Na+ 154 mEq/L (154 mmol/L) d. PO4-3 4.8 mg/dl (1.55 mmol/L)

ANS: C Na+ 154 mEq/L (154 mmol/L) The elevated serum sodium level is consistent with the patient's neurologic symptoms and indicates a need for immediate action to prevent further serious complications such as seizures. The potassium and calcium levels vary slightly from the normal but do not require any immediate action by the nurse. The phosphate level is within the normal parameters.

A patient with renal failure who has been taking aluminum hydroxide/magnesium hydroxide suspension (Maalox) at home for indigestion is somnolent and has decreased deep tendon reflexes. Which action should the nurse take first? a. Notify the patient's health care provider. b. Withhold the next scheduled dose of Maalox. c. Review the magnesium level on the patient's chart. d. Check the chart for the most recent potassium level.

ANS: C Review the magnesium level on the patient's chart. The patient has a history and symptoms consistent with hypermagnesemia; the nurse should check the chart for a recent serum magnesium level. Notification of the health care provider will be done after the nurse knows the magnesium level. The Maalox should be held, but more immediate action is needed to correct the patient's decreased deep tendon reflexes (DTRs) and somnolence. Monitoring of potassium levels also is important for patients with renal failure, but the patient's current symptoms are not consistent with hyperkalemia.

A recently admitted patient has a small cell carcinoma of the lung, which is causing the syndrome of inappropriate antidiuretic hormone (SIADH). The nurse will monitor carefully for a. increased total urinary output. b. elevation of serum hematocrit. c. decreased serum sodium level. d. rapid and unexpected weight loss.

ANS: C decreased serum sodium level. SIADH causes water retention and a decrease in serum sodium level. Weight loss, increased urine output, and elevated serum hematocrit may be associated with excessive loss of water, but not with SIADH and water retention.

When assessing a patient with increased extracellular fluid (ECF) osmolality, the priority assessment for the nurse to obtain is a. skin turgor. b. heart sounds. c. mental status. d. capillary refill.

ANS: C mental status. Changes in ECF osmolality lead to swelling or shrinking of cells in the central nervous system, initially causing confusion, which may progress to coma or seizures. Although skin turgor, capillary refill, and heart sounds also may be affected by ECF osmolality changes and resultant fluid shifts, these are signs that occur later and do not have as immediate an impact on patient outcomes.

A patient who has an infusion of 50% dextrose prescribed asks the nurse why a peripherally inserted central catheter must be inserted. Which explanation by the nurse is correct? a. The prescribed infusion can be given much more rapidly when the patient has a central line. b. There is a decreased risk for infection when 50% dextrose is infused through a central line. c. The 50% dextrose is hypertonic and will be more rapidly diluted when given through a central line. d. The required blood glucose monitoring is more accurate when samples are obtained from a central line.

ANS: C. The 50% dextrose is hypertonic and will be more rapidly diluted when given through a central line. Shrinkage of red blood cells can occur when solutions with dextrose concentrations greater than 10% are administered intravenously. Blood glucose testing is not more accurate when samples are obtained from a central line. The infection risk is higher with a central catheter than with peripheral IV lines. Hypertonic or concentrated IV solutions are not given rapidly.

The nurse is obtaining a health history from a 24-year-old patient with hypertrophic cardiomyopathy (HC). Which information obtained by the nurse is most important? a. The patient has a history of a recent upper respiratory infection. b. The patient has a family history of coronary artery disease (CAD). c. The patient reports using cocaine a "couple of times" as a teenager. d. The patient's 29-year-old brother died from a sudden cardiac arrest.

ANS: D About half of all cases of HC have a genetic basis, and it is the most common cause of sudden cardiac death in otherwise healthy young people. The information about the patient's brother will be helpful in planning care (such as an automatic implantable cardioverter-defibrillator [AICD

After receiving change-of-shift report about the following four patients on the cardiac care unit, which patient should the nurse assess first? a. A 39-yr-old patient with pericarditis who is complaining of sharp, stabbing chest pain b. A 56-yr-old patient with variant angina who is scheduled to receive nifedipine (Procardia) c. A 65-yr-old patient who had a myocardial infarction (MI) 4 days ago and is anxious about today's planned discharge d. A 59-yr-old patient with unstable angina who has just returned after a percutaneous coronary intervention (PCI)

ANS: D After PCI, the patient is at risk for hemorrhage from the arterial access site. The nurse should assess the patient's blood pressure, pulses, and the access site immediately. The other patients should also be assessed as quickly as possible, but assessment of this patient has the highest priority. DIF: Cognitive Level: Analyze (analysis)

A patient admitted to the coronary care unit (CCU) with an ST-segment-elevation myocardial infarction (STEMI) is restless and anxious. The blood pressure is 86/40 mm Hg, and heart rate is 132 beats/min. Based on this information, which nursing diagnosis is a priority for the patient? a. Acute pain related to myocardial infarction b. Anxiety related to perceived threat of death c. Stress overload related to acute change in health d. Decreased cardiac output related to cardiogenic shock

ANS: D All the nursing diagnoses may be appropriate for this patient, but the hypotension and tachycardia indicate decreased cardiac output and shock from the damaged myocardium. This will result in decreased perfusion to all vital organs (e.g., brain, kidney, heart) and is a priority. DIF: Cognitive Level: Analyze (analysis)

A patient with pneumonia has a fever of 101.2° F (38.5° C), a nonproductive cough, and an oxygen saturation of 89%. The patient is very weak and needs assistance to get out of bed. The priority nursing diagnosis for the patient is a. hyperthermia related to infectious illness. b. impaired transfer ability related to weakness. c. ineffective airway clearance related to thick secretions. d. impaired gas exchange related to respiratory congestion.

ANS: D All these nursing diagnoses are appropriate for the patient, but the patient's oxygen saturation indicates that all body tissues are at risk for hypoxia unless the gas exchange is improved. DIF: Cognitive Level: Application REF: 552-553 OBJ: Special Questions: Prioritization TOP: Nursing Process: Diagnosis MSC: NCLEX: Physiological Integrity

Which statement made by a patient with coronary artery disease after the nurse has completed teaching about the therapeutic lifestyle changes (TLC) diet indicates that further teaching is needed? a. "I will switch from whole milk to 1% milk." b. "I like salmon and I will plan to eat it more often." c. "I can have a glass of wine with dinner if I want one." d. "I will miss being able to eat peanut butter sandwiches."

ANS: D Although only 30% of the daily calories should come from fats, most of the fat in the TLC diet should come from monounsaturated fats such as are found in nuts, olive oil, and canola oil. The patient can include peanut butter sandwiches as part of the TLC diet. The other patient comments indicate a good understanding of the TLC diet. DIF: Cognitive Level: Apply (application)

Spironolactone (Aldactone), an aldosterone antagonist, is prescribed for a patient. Which statement by the patient indicates that the teaching about this medication has been effective? a. "I will try to drink at least 8 glasses of water every day." b. "I will use a salt substitute to decrease my sodium intake." c. "I will increase my intake of potassium-containing foods." d. "I will drink apple juice instead of orange juice for breakfast."

ANS: D Because spironolactone is a potassium-sparing diuretic, patients should be taught to choose low-potassium foods (e.g., apple juice) rather than foods that have higher levels of potassium (e.g., citrus fruits). Because the patient is using spironolactone as a diuretic, the nurse would not encourage the patient to increase fluid intake. Teach patients to avoid salt substitutes, which are high in potassium

Nadolol (Corgard) is prescribed for a patient with chronic stable angina and left ventricular dysfunction. To determine whether the drug is effective, the nurse will monitor for a. decreased blood pressure and heart rate. b. fewer complaints of having cold hands and feet. c. improvement in the strength of the distal pulses. d. participation in daily activities without chest pain.

ANS: D Because the drug is ordered to improve the patient's angina, effectiveness is indicated if the patient is able to accomplish daily activities without chest pain. Blood pressure and heart rate may decrease, but these data do not indicate that the goal of decreased angina has been met. The noncardioselective -adrenergic blockers can cause peripheral vasoconstriction, so the nurse would not expect an improvement in distal pulse quality or skin temperature. DIF: Cognitive Level: Apply (application)

20. A patient arrives in the ear, nose, and throat clinic complaining of a piece of tissue being "stuck up my nose" and with foul-smelling nasal drainage from the right nare. Which action should the nurse take first? a. Notify the clinic health care provider. b. Obtain aerobic culture specimens of the drainage. c. Ask the patient about how the cotton got into the nose. d. Have the patient occlude the left nare and blow the nose.

ANS: D Because the highest priority action is to remove the foreign object from the nare, the nurse's first action should be to assist the patient to remove the object. The other actions are also appropriate but should be done after attempting to clear the nose.

Which information from a patient helps the nurse confirm the previous diagnosis of chronic stable angina? a. "The pain wakes me up at night." b. "The pain is level 3 to 5 (0 to 10 scale)." c. "The pain has gotten worse over the last week." d. "The pain goes away after a nitroglycerin tablet."

ANS: D Chronic stable angina is typically relieved by rest or nitroglycerin administration. The level of pain is not a consistent indicator of the type of angina. Pain occurring at rest or with increased frequency is typical of unstable angina. DIF: Cognitive Level: Understand (comprehension)

A patient who has a right-sided chest tube following a thoracotomy has continuous bubbling in the suction-control chamber of the collection device. The most appropriate action by the nurse is to a. document the presence of a large air leak. b. obtain and attach a new collection device. c. notify the surgeon of a possible pneumothorax. d. take no further action with the collection device.

ANS: D Continuous bubbling is expected in the suction-control chamber and indicates that the suction-control chamber is connected to suction. An air leak would be detected in the water-seal chamber. There is no evidence of pneumothorax. A new collection device is needed when the collection chamber is filled. DIF: Cognitive Level: Application REF: 572 TOP: Nursing Process: Implementation MSC: NCLEX: Physiological Integrity

To determine the effectiveness of prescribed therapies for a patient with cor pulmonale and right-sided heart failure, which assessment will the nurse make? a. Lung sounds b. Heart sounds c. Blood pressure d. Peripheral edema

ANS: D Cor pulmonale is right ventricular failure caused by pulmonary hypertension, so clinical manifestations of right ventricular failure such as peripheral edema, jugular vein distention, and right upper-quadrant abdominal tenderness would be expected. Abnormalities in lung sounds, blood pressure, or heart sounds are not caused by cor pulmonale. DIF: Cognitive Level: Application REF: 582 TOP: Nursing Process: Evaluation MSC: NCLEX: Physiological Integrity

The home health nurse cares for an alert and oriented older adult patient with a history of dehydration. Which instructions should the nurse give to this patient related to fluid intake? a. "Increase fluids if your mouth feels dry. b. "More fluids are needed if you feel thirsty." c. "Drink more fluids in the late evening hours." d. "If you feel lethargic or confused, you need more to drink."

ANS: D Crackles throughout both lungs suggest that the patient may be experiencing pulmonary edema, a life-threatening adverse effect of hypertonic solutions. The increased pulse rate and blood pressure and the appearance of the urine also should be reported, but they are not as dangerous as the presence of fluid in the alveoli

An alcoholic and homeless patient is diagnosed with active tuberculosis (TB). Which intervention by the nurse will be most effective in ensuring adherence with the treatment regimen? a. Educating the patient about the long-term impact of TB on health b. Giving the patient written instructions about how to take the medications c. Teaching the patient about the high risk for infecting others unless treatment is followed d. Arranging for a daily noontime meal at a community center and giving the medication then

ANS: D Directly observed therapy is the most effective means for ensuring compliance with the treatment regimen, and arranging a daily meal will help to ensure that the patient is available to receive the medication. The other nursing interventions may be appropriate for some patients, but are not likely to be as helpful with this patient. DIF: Cognitive Level: Application REF: 556 TOP: Nursing Process: Implementation MSC: NCLEX: Physiological Integrity

A patient who is taking a potassium-wasting diuretic for treatment of hypertension complains of generalized weakness. It is most appropriate for the nurse to take which action? a. Assess for facial muscle spasms. b. Ask the patient about loose stools. c. Suggest that the patient avoid orange juice with meals. d. Ask the health care provider to order a basic metabolic panel.

ANS: D Generalized weakness is a manifestation of hypokalemia. After the health care provider orders the metabolic panel, the nurse should check the potassium level. Facial muscle spasms might occur with hypocalcemia. Orange juice is high in potassium and would be advisable to drink if the patient was hypokalemic. Loose stools are associated with hyperkalemia

Which of these orders will the nurse act on first for a patient who has just been admitted with probable bacterial pneumonia and sepsis? a. Administer aspirin suppository. b. Send to radiology for chest x-ray. c. Give ciprofloxacin (Cipro) 400 mg IV. d. Obtain blood cultures from two sites.

ANS: D Initiating antibiotic therapy rapidly is essential, but it is important that the cultures be obtained before antibiotic administration. The chest radiograph and aspirin administration can be done last. DIF: Cognitive Level: Application REF: 549 OBJ: Special Questions: Prioritization TOP: Nursing Process: Implementation MSC: NCLEX: Physiological Integrity

A patient had a non-ST-segment-elevation myocardial infarction (NSTEMI) 3 days ago. Which nursing intervention included in the plan of care is appropriate for the registered nurse (RN) to delegate to an experienced licensed practical/vocational nurse (LPN/LVN)? a. Evaluation of the patient's response to walking in the hallway b. Completion of the referral form for a home health nurse follow-up c. Education of the patient about the pathophysiology of heart disease d. Reinforcement of teaching about the purpose of prescribed medications

ANS: D LPN/LVN education and scope of practice include reinforcing education that has previously been done by the RN. Evaluating the patient's response to exercise after a NSTEMI requires more education and should be done by the RN. Teaching and discharge planning and referral are skills that require RN education and scope of practice. DIF: Cognitive Level: Apply (application)

The nurse could delegate care of which patient to a licensed practical/vocational nurse (LPN/LVN)? a. The patient who reports increased tenderness and swelling around a leg wound b. The patient who was just admitted after suturing of a full-thickness arm wound c. The patient who needs teaching about home care for a draining abdominal wound d. The patient who requires a hydrocolloid dressing change for a stage III sacral ulcer

ANS: D LPN/LVN education and scope of practice include sterile dressing changes for stable patients. Initial wound assessments, patient teaching, and evaluation for possible poor wound healing or infection should be done by the registered nurse (RN). DIF: Cognitive Level: Apply (application)

After discharge teaching has been completed for a patient who has had a lung transplant, the nurse will evaluate that the teaching has been effective if the patient states a. "I will make an appointment to see the doctor every year." b. "I will not turn the home oxygen up higher than 2 L/minute." c. "I will not worry if I feel a little short of breath with exercise." d. "I will call the health care provider right away if I develop a fever."

ANS: D Low-grade fever may indicate infection or acute rejection, so the patient should notify the health care provider immediately if the temperature is elevated. Patients require frequent follow-up visits with the transplant team; annual health care provider visits would not be sufficient. Home oxygen use is not an expectation after lung transplant. Shortness of breath should be reported. DIF: Cognitive Level: Application REF: 583 TOP: Nursing Process: Evaluation MSC: NCLEX: Physiological Integrity

1. The nurse teaches a patient about discharge instructions after a rhinoplasty. Which statement, if made by the patient, indicates that the teaching was successful? a. "I can take 800 mg ibuprofen for pain control." b. "I will safely remove and reapply nasal packing daily." c. "My nose will look normal after 24 hours when the swelling goes away." d. "I will keep my head elevated for 48 hours to minimize swelling and pain."

ANS: D Maintaining the head in an elevated position will decrease the amount of nasal swelling. NSAIDs, such as ibuprofen, increase the risk for postoperative bleeding and should not be used postoperatively. The patient would not be taught to remove or reapply nasal packing, which is usually removed by the surgeon on the day after surgery. Although return to a preinjury appearance is the goal of the surgery, it is not always possible to achieve this result, especially in the first few weeks after surgery.

A lobectomy is scheduled for a patient with stage I non-small cell lung cancer. The patient tells the nurse, "I would rather have radiation than surgery." Which response by the nurse is most appropriate? a. "Are you afraid that the surgery will be very painful?" b. "Did you have bad experiences with previous surgeries?" c. "Surgery is the treatment of choice for stage I lung cancer." d. "Tell me what you know about the various treatments available."

ANS: D More assessment of the patient's concerns about surgery is indicated. An open-ended response will elicit the most information from the patient. The answer beginning, "Surgery is the treatment of choice" is accurate, but it discourages the patient from sharing concerns about surgery. The remaining two answers indicate that the nurse has jumped to conclusions about the patient's reasons for not wanting surgery. DIF: Cognitive Level: Application REF: 565 TOP: Nursing Process: Implementation MSC: NCLEX: Psychosocial Integrity

Which information about a patient who has a recent history of tuberculosis (TB) indicates that the nurse can discontinue airborne isolation precautions? a. Chest x-ray shows no upper lobe infiltrates. b. TB medications have been taken for 6 months. c. Mantoux testing shows an induration of 10 mm. d. Three sputum smears for acid-fast bacilli are negative.

ANS: D Negative sputum smears indicate that M. tuberculosis is not present in the sputum, and the patient cannot transmit the bacteria by the airborne route. Chest x-rays are not used to determine whether treatment has been successful. Taking medications for 6 months is necessary, but the multidrug-resistant forms of the disease might not be eradicated after 6 months of therapy. Repeat Mantoux testing would not be done since it will not change even with effective treatment. DIF: Cognitive Level: Application REF: 557 TOP: Nursing Process: Implementation MSC: NCLEX: Physiological Integrity

21. The nurse is caring for a patient who has acute pharyngitis caused by Candida albicans. Which action is appropriate for the nurse to include in the plan of care? a. Avoid giving patient warm liquids to drink. b. Assess patient for allergies to penicillin antibiotics. c. Teach the patient about the need to sleep in a warm, dry environment. d. Teach patient to "swish and swallow" prescribed oral nystatin (Mycostatin).

ANS: D Oral or pharyngeal fungal infections are treated with nystatin solution. The goal of the "swish and swallow" technique is to expose all of the oral mucosa to the antifungal agent. Warm liquids may be soothing to a sore throat. The patient should be taught to use a cool mist humidifier. There is no need to assess for penicillin/cephalosporin allergies because Candida albicans infection is treated with antifungals.

Which action by the occupational health nurse at a manufacturing plant where there is potential exposure to inhaled dust will be most helpful in reducing incidence of lung disease? a. Teach about symptoms of lung disease. b. Treat workers who inhale dust particles. c. Monitor workers for shortness of breath. d. Require the use of protective equipment.

ANS: D Prevention of lung disease requires the use of appropriate protective equipment such as masks. The other actions will help in recognition or early treatment of lung disease, but will not be effective in prevention of lung damage. DIF: Cognitive Level: Application REF: 560-561 TOP: Nursing Process: Assessment MSC: NCLEX: Health Promotion and Maintenance

A patient with rheumatoid arthritis has been taking oral corticosteroids for 2 years. Which nursing action is most likely to detect early signs of infection in this patient? a. Monitor white blood cell counts. b. Check the skin for areas of redness. c. Measure the temperature every 2 hours. d. Ask about feelings of fatigue or malaise.

ANS: D The earliest manifestation of an infection may be "just not feeling well." Common clinical manifestations of inflammation and infection are frequently not present when patients receive immunosuppressive medications. DIF: Cognitive Level: Analyze (analysis)

After the nurse has finished teaching a patient about the use of sublingual nitroglycerin (Nitrostat), which patient statement indicates that the teaching has been effective? a. "I can expect nausea as a side effect of nitroglycerin." b. "I should only take nitroglycerin when I have chest pain." c. "Nitroglycerin helps prevent a clot from forming and blocking blood flow to my heart." d. "I will call an ambulance if I still have pain after taking three nitroglycerin 5 minutes apart."

ANS: D The emergency response system (ERS) should be activated when chest pain or other symptoms are not completely relieved after three sublingual nitroglycerin tablets taken 5 minutes apart. Nitroglycerin can be taken to prevent chest pain or other symptoms from developing (e.g., before intercourse). Gastric upset (e.g., nausea) is not an expected side effect of nitroglycerin. Nitroglycerin does not impact the underlying pathophysiology of coronary artery atherosclerosis. DIF: Cognitive Level: Apply (application)

13. A nurse is caring for a patient who has had a total laryngectomy and radical neck dissection. During the first 24 hours after surgery what is the priority nursing action? a. Monitor for bleeding. b. Maintain adequate IV fluid intake. c. Suction tracheostomy every eight hours. d. Keep the patient in semi-Fowler's position.

ANS: D The most important goals after a laryngectomy and radical neck dissection are to maintain the airway and ensure adequate oxygenation. Keeping the patient in a semi-Fowler's position will decrease edema and limit tension on the suture lines to help ensure an open airway. Maintenance of IV fluids and monitoring for bleeding are important, but maintaining an open airway is the priority. Tracheostomy care and suctioning should be provided as needed. During the immediate postoperative period, the patient with a laryngectomy requires frequent suctioning of the tracheostomy tube.

2. The nurse plans to teach a patient how to manage allergic rhinitis. Which information should the nurse include in the teaching plan? a. Hand washing is the primary way to prevent spreading the condition to others. b. Use of oral antihistamines for 2 weeks before the allergy season may prevent reactions. c. Corticosteroid nasal sprays will reduce inflammation, but systemic effects limit their use. d. Identification and avoidance of environmental triggers are the best way to avoid symptoms.

ANS: D The most important intervention is to assist the patient in identifying and avoiding potential allergens. Intranasal corticosteroids (not oral antihistamines) should be started several weeks before the allergy season. Corticosteroid nasal sprays have minimal systemic absorption. Acute viral rhinitis (the common cold) can be prevented by washing hands.

When teaching the patient with allergic rhinitis about management of the condition, the nurse explains that a. over-the-counter (OTC) antihistamines cause sedation, so prescription antihistamines are usually ordered. b. corticosteroid nasal sprays will reduce inflammation, but systemic effects limit their use. c. use of oral antihistamines for a few weeks before the allergy season may prevent reactions. d. identification and avoidance of environmental triggers are the best way to avoid symptoms.

ANS: D The most important intervention is to assist the patient in identifying and avoiding potential allergens. Intranasal corticosteroids (not oral antihistamines) should be started several weeks before the allergy season. Corticosteroid nasal sprays have minimal systemic absorption. Nonsedating antihistamines are available OTC. DIF: Cognitive Level: Application REF: 521-523 TOP: Nursing Process: Implementation MSC: NCLEX: Physiological Integrity

A patient who was involved in a motor vehicle crash has had a tracheostomy placed to allow for continued mechanical ventilation. How should the nurse interpret the following arterial blood gas results: pH 7.48, PaO2 85 mm Hg, PaCO2 32 mm Hg, and HCO3 25 mEq/L? a. Metabolic acidosis b. Metabolic alkalosis c. Respiratory acidosis d. Respiratory alkalosis

ANS: D The pH indicates that the patient has alkalosis and the low PaCO2 indicates a respiratory cause. The other responses are incorrect based on the pH and the normal HCO3

A patient admitted with acute dyspnea is newly diagnosed with dilated cardiomyopathy. Which information will the nurse plan to teach the patient about managing this disorder? a. A heart transplant should be scheduled as soon as possible. b. Elevating the legs above the heart will help relieve dyspnea. c. Careful compliance with diet and medications will prevent heart failure. d. Notify the doctor about any symptoms of heart failure such as shortness of breath.

ANS: D The patient should be instructed to notify the health care provider about any worsening of heart failure symptoms. Because dilated cardiomyopathy does not respond well to therapy, even patients with good compliance with therapy may have recurrent episodes of heart failure. Elevation of the legs above the heart will worsen symptoms (although this approach is appropriate for a patient with hypertrophic cardiomyopathy). The patient with terminal or end-stage cardiomyopathy may consider heart transplantation.

When assessing a 24-year-old patient who has just arrived after an automobile accident, the emergency department nurse notes that the breath sounds are absent on the right side. The nurse will anticipate the need for a. emergency pericardiocentesis. b. stabilization of the chest wall with tape. c. administration of an inhaled bronchodilator. d. insertion of a chest tube with a chest drainage system.

ANS: D The patient's history and absent breath sounds suggest a right-sided pneumothorax or hemothorax, which will require treatment with a chest tube and drainage. The other therapies would be appropriate for an acute asthma attack, flail chest, or cardiac tamponade, but the patient's clinical manifestations are not consistent with these problems. DIF: Cognitive Level: Application REF: 567 TOP: Nursing Process: Planning MSC: NCLEX: Physiological Integrity

A patient who has had chest pain for several hours is admitted with a diagnosis of rule out acute myocardial infarction (AMI). Which laboratory test should the nurse monitor to best determine whether the patient has had an AMI? a. Myoglobin c. C-reactive protein b. Homocysteine d. Cardiac-specific troponin

ANS: D Troponin levels increase about 4 to 6 hours after the onset of myocardial infarction (MI) and are highly specific indicators for MI. Myoglobin is released within 2 hours of MI, but it lacks specificity and its use is limited. The other laboratory data are useful in determining the patient's risk for developing coronary artery disease but are not helpful in determining whether an acute MI is in progress. DIF: Cognitive Level: Understand (comprehension)

Which action could the nurse delegate to unlicensed assistive personnel (UAP) trained as electrocardiogram (ECG) technicians working on the cardiac unit? a. Select the best lead for monitoring a patient with an admission diagnosis of Dressler syndrome. b. Obtain a list of herbal medications used at home while admitting a new patient with pericarditis. c. Teach about the need to monitor the weight daily for a patient who has hypertrophic cardiomyopathy. d. Check the heart monitor for changes in rhythm while a patient who had a valve replacement ambulates.

ANS: D Under the supervision of registered nurses (RNs), UAP check the patient's cardiac monitor and obtain information about changes in heart rate and rhythm with exercise. Teaching and obtaining information about home medications (prescribed or complementary) and selecting the best leads for monitoring patients require more critical thinking and should be done by the RN.

9. A patient scheduled for a total laryngectomy and radical neck dissection for cancer of the larynx asks the nurse, "Will I be able to talk normally after surgery?" What is the best response by the nurse? a. "You will breathe through a permanent opening in your neck, but you will not be able to communicate orally." b. "You won't be able to talk right after surgery, but you will be able to speak again after the tracheostomy tube is removed." c. "You won't be able to speak as you used to, but there are artificial voice devices that will give you the ability to speak normally." d. "You will have a permanent opening into your neck, and you will need to have rehabilitation for some type of voice restoration."

ANS: D Voice rehabilitation is planned after a total laryngectomy, and a variety of assistive devices are available to restore communication. Although the ability to communicate orally is changed, it would not be appropriate to tell a patient that this ability would be lost. Artificial voice devices do not permit normal-sounding speech. In a total laryngectomy, the vocal cords are removed, so normal speech is impossible.

A patient scheduled for a total laryngectomy and radical neck dissection for cancer of the larynx asks the nurse, "How will I talk after the surgery?" The best response by the nurse is, a. "You will breathe through a permanent opening in your neck, but you will not be able to communicate orally." b. "You won't be able to talk right after surgery, but you will be able to speak again after the tracheostomy tube is removed." c. "You won't be able to speak as you used to, but there are artificial voice devices that will give you the ability to speak normally." d. "You will have a permanent opening into your neck, and you will need to have rehabilitation for some type of voice restoration."

ANS: D Voice rehabilitation is planned after a total laryngectomy, and a variety of assistive devices are available to restore communication. Although the ability to communicate orally is changed, it would not be appropriate to tell a patient that this ability would be lost. Artificial voice devices do not permit normal-sounding speech. In a total laryngectomy, the vocal cords are removed, so normal speech is impossible. DIF: Cognitive Level: Application REF: 541 TOP: Nursing Process: Implementation MSC: NCLEX: Physiological Integrity

The nurse should plan to use a wet-to-dry dressing for which patient? a. A patient who has a pressure ulcer with pink granulation tissue b. A patient who has a surgical incision with pink, approximated edges c. A patient who has a full-thickness burn filled with dry, black material d. A patient who has a wound with purulent drainage and dry brown areas

ANS: D Wet-to-dry dressings are used when there is minimal eschar to be removed. A full-thickness wound filled with eschar will require interventions such as surgical debridement to remove the necrotic tissue. Wet-to-dry dressings are not needed on approximated surgical incisions. Wet-to-dry dressings are not used on uninfected granulating wounds because of the damage to the granulation tissue. DIF: Cognitive Level: Apply (application)

A postoperative patient who is receiving nasogastric suction is complaining of anxiety and incisional pain. The patient's respiratory rate is 32 breaths/minute and the arterial blood gases (ABGs) indicate respiratory alkalosis. Which action should the nurse take first? a. Discontinue the nasogastric suctions for a few hours. b. Notify the health care provider about the ABG results. c. Teach the patient about the need to take slow, deep breaths. d. Give the patient the PRN morphine sulfate 4 mg intravenously.

ANS: D Give the patient the PRN morphine sulfate 4 mg intravenously. The patient's respiratory alkalosis is caused by the increased respiratory rate associated with pain and anxiety. The nurse's first action should be to medicate the patient for pain. Although the nasogastric suction may contribute to the alkalosis, it is not appropriate to discontinue the tube when the patient needs gastric suction. The health care provider may be notified about the ABGs but is likely to instruct the nurse to medicate for pain. The patient will not be able to take slow, deep breaths when experiencing pain.

The nurse has administered 3% saline to a patient with hyponatremia. Which one of these assessment data will require the most rapid response by the nurse? a. The patient's radial pulse is 105 beats/minute. b. There is sediment and blood in the patient's urine. c. The blood pressure increases from 120/80 to 142/94. d. There are crackles audible throughout both lung fields.

ANS: D There are crackles audible throughout both lung fields. Crackles throughout both lungs suggest that the patient may be experiencing pulmonary edema, a life-threatening adverse effect of hypertonic solutions. The increased pulse rate and blood pressure and the appearance of the urine also should be reported, but they are not as dangerous as the presence of fluid in the alveoli.

A patient with hypercalcemia is being cared for on the medical unit. Nursing actions included on the care plan will include a. maintaining the patient on bed rest. b. auscultating lung sounds every 4 hours. c. monitoring for Trousseau's and Chvostek's signs. d. encouraging fluid intake up to 4000 ml every day.

ANS: D encouraging fluid intake up to 4000 ml every day. To decrease the risk for renal calculi, the patient should have an intake of 3000 to 4000 ml daily. Ambulation helps decrease the loss of calcium from bone and is encouraged in patients with hypercalcemia. Trousseau's and Chvostek's signs are monitored when there is a possibility of hypocalcemia. There is no indication that the patient needs frequent assessment of lung sounds, although these would be assessed every shift.

A patient is taking a potassium-wasting diuretic for treatment of hypertension. The nurse will teach the patient to report symptoms of adverse effects such as a. personality changes. b. frequent loose stools. c. facial muscle spasms. d.generalized weakness.

ANS: D generalized weakness. Generalized weakness progressing to flaccidity is a manifestation of hypokalemia. Facial muscle spasms might occur with hypocalcemia. Loose stools are associated with hyperkalemia. Personality changes are not associated with electrolyte disturbances, although changes in mental status are common manifestations with sodium excess or deficit.

A patient who has required prolonged mechanical ventilation has the following arterial blood gas results: pH 7.48, PaO2 85 mm Hg, PaCO2 32 mm Hg, and HCO3 25 mEq/L. The nurse interprets these results as a. metabolic acidosis. b. metabolic alkalosis. c. respiratory acidosis. d. respiratory alkalosis.

ANS: D respiratory alkalosis. The pH indicates that the patient has alkalosis and the low PaCO2 indicates a respiratory cause. The other responses are incorrect based on the pH and the normal HCO3.

What is the nurse's primary concern regarding fluid & electrolytes when caring for an elderly pt who is intermittently confused? 1. risk of dehydration 2. risk of kidney damage 3. risk of stroke 4. risk of bleeding

Answer: 1 Rationale 1: As an adult ages, the thirst mechanism declines. Adding this in a pt with an altered level of consciousness, there is an increased risk of dehydration & high serum osmolality. Rationale 2: The risks for kidney damage are not specifically related to aging or fluid & electrolyte issues. Rationale 3: The risk of stroke is not specifically related to aging or fluid & electrolyte issues. Rationale 4: The risk of bleeding is not specifically related to aging or fluid & electrolyte issues.

The nurse is planning care for a pt with severe burns. Which of the following is this pt at risk for developing? 1. intracellular fluid deficit 2. intracellular fluid overload 3. extracellular fluid deficit 4. interstitial fluid deficit

Answer: 1 Rationale 1: Because this pt was severely burned, the fluid within the cells is diminished, leading to an intracellular fluid deficit. Rationale 2: The intracellular fluid is all fluids that exist within the cell cytoplasm & nucleus. Because this pt was severely burned, the fluid within the cells is diminished, leading to an intracellular fluid deficit. Rationale 3: The extracellular fluid is all fluids that exist outside the cell, including the interstitial fluid between the cells. Because this pt was severely burned, the fluid within the cells is diminished, leading to an intracellular fluid deficit. Rationale 4: The extracellular fluid is all fluids that exist outside the cell, including the interstitial fluid between the cells. Because this pt was severely burned, the fluid within the cells is diminished, leading to an intracellular fluid deficit.

A pt, experiencing multisystem fluid volume deficit, has the symptoms of tachycardia, pale, cool skin, & decreased urine output. The nurse realizes these findings are most likely a direct result of which of the following? 1. the body's natural compensatory mechanisms 2. pharmacological effects of a diuretic 3. effects of rapidly infused intravenous fluids 4. cardiac failure

Answer: 1 Rationale 1: The internal vasoconstrictive compensatory reactions within the body are responsible for the symptoms exhibited. The body naturally attempts to conserve fluid internally specifically for the brain & heart. Rationale 2: A diuretic would cause further fluid loss, & is contraindicated. Rationale 3: Rapidly infused intravenous fluids would not cause a decrease in urine output. Rationale 4: The manifestations reported are not indicative of cardiac failure in this pt.

A pregnant pt is admitted with excessive thirst, increased urination, & has a medical diagnosis of diabetes insipidus. The nurse chooses which of the following nursing diagnoses as most appropriate? 1. Risk for Imbalanced Fluid Volume 2. Excess Fluid Volume 3. Imbalanced Nutrition 4. Ineffective Tissue Perfusion

Answer: 1 Rationale 1: The pt with excessive thirst, increased urination & a medical diagnosis of diabetes insipidus is at risk for Imbalanced Fluid Volume due to the pt &'s excess volume loss that can increase the serum levels of sodium. Rationale 2: Excess Fluid Volume is not an issue for pts with diabetes insipidus, especially during the early stages of treatment. Rationale 3: Imbalanced Nutrition does not apply. Rationale 4: Ineffective Tissue Perfusion does not apply

A pt recovering from surgery has an indwelling urinary catheter. The nurse would contact the pt's primary healthcare provider with which of the following 24-hour urine output volumes? 1. 600 mL 2. 750 mL 3. 1000 mL 4. 1200 mL

Answer: 1 Rationale 1: A urine output of less than 30 mL per hour must be reported to the primary healthcare provider. This indicates inadequate renal perfusion, placing the pt at increased risk for acute renal failure & inadequate tissue perfusion. A minimum of 720 mL over a 24-hour period is desired (30 mL multiplied by 24 hours equals 720 mL per 24 hours).

An elderly postoperative pt is demonstrating lethargy, confusion, & a resp rate of 8 per minute. The nurse sees that the last dose of pain medication administered via a pt controlled anesthesia (PCA) pump was within 30 minutes. Which of the following acid-base disorders might this pt be experiencing? 1. respiratory acidosis 2. metabolic acidosis 3. respiratory alkalosis 4. metabolic alkalosis

Answer: 1 Rationale 1: Acute respiratory acidosis occurs due to a sudden failure of ventilation. Overdoses of narcotic or sedative medications can lead to this condition. Rationale 2: The pt condition being described is respiratory not metabolic in nature. Rationale 3: Acute respiratory acidosis occurs due to a sudden failure of ventilation. Overdoses of narcotic or sedative medications can lead to this condition. Rationale 4: Acute respiratory acidosis occurs due to a sudden failure of ventilation. Overdoses of narcotic or sedative medications can lead to this condition. The pt condition being described is respiratory not metabolic in nature.

A pt is receiving intravenous fluids postoperatively following cardiac surgery. Nursing assessments should focus on which postoperative complication? 1. fluid volume excess 2. fluid volume deficit 3. seizure activity 4. liver failure

Answer: 1 Rationale 1: Antidiuretic hormone & aldosterone levels are commonly increased following the stress response before, during, & immediately after surgery. This increase leads to sodium & water retention. Adding more fluids intravenously can cause a fluid volume excess & stress upon the heart & circulatory system. Rationale 2: Adding more fluids intravenously can cause a fluid volume excess, not fluid volume deficit, & stress upon the heart & circulatory system. Rationale 3: Seizure activity would more commonly be associated with electrolyte imbalances. Rationale 4: Liver failure is not anticipated related to postoperative intravenous fluid administration.

An elderly pt with peripheral neuropathy has been taking magnesium supplements. The nurse realizes that which of the following symptoms can indicate hypermagnesaemia? 1. hypotension, warmth, & sweating 2. nausea & vomiting 3. hyperreflexia 4. excessive urination

Answer: 1 Rationale 1: Elevations in magnesium levels are accompanied by hypotension, warmth, & sweating. Rationale 2: Lower levels of magnesium are associated with nausea & vomiting. Rationale 3: Lower levels of magnesium are associated & hyperreflexia. Rationale 4: Urinary changes are not noted.

A pt is diagnosed with hypokalemia. After reviewing the pt's current medications, which of the following might have contributed to the pt's health problem? 1. corticosteroid 2. thiazide diuretic 3. narcotic 4. muscle relaxer

Answer: 1 Rationale 1: Excess potassium loss through the kidneys is often caused by such meds as corticosteroids, potassium-wasting diuretics, amphotericin B, & large doses of some antibiotics. Rationale 2: Excessive sodium is lost with the use of thiazide diuretics. Rationale 3: Narcotics do not typically affect electrolyte balance. Rationale 4: Muscle relaxants do not typically affect electrolyte balance.

A pt is diagnosed with hyperphosphatemia. The nurse realizes that this pt might also have an imbalance of which of the following electrolytes? 1. calcium 2. sodium 3. potassium 4. chloride

Answer: 1 Rationale 1: Excessive serum phosphate levels cause few specific symptoms. The effects of high serum phosphate levels on nerves & muscles are more likely the result of hypocalcemia that develops secondary to an elevated serum phosphorus level. The phosphate in the serum combines with ionized calcium, & the ionized serum calcium level falls.

A pt prescribed spironolactone is demonstrating ECG changes & complaining of muscle weakness. The nurse realizes this pt is exhibiting signs of which of the following? 1. hyperkalemia 2. hypokalemia 3. hypercalcemia 4. hypocalcemia

Answer: 1 Rationale 1: Hyperkalemia is serum potassium level greater than 5.0 mEq/L. Decreased potassium excretion is seen in potassium-sparing diuretics such as spironolactone. Common manifestations of hyperkalemia are muscle weakness & ECG changes. Rationale 2: Hypokalemia is seen in non-potassium diuretics such as furosemide. Rationale 3: Hypercalcemia has been associated with thiazide diuretics. Rationale 4: Hypocalcemia is seen in pts who have received many units of citrated blood & is not associated with diuretic use.

A pt who is taking digoxin (Lanoxin) is admitted with possible hypokalemia. Which of the following does the nurse realize might occur with this pt? 1. Digoxin toxicity may occur. 2. A higher dose of digoxin (Lanoxin) may be needed. 3. A diuretic may be needed. 4. Fluid volume deficit may occur.

Answer: 1 Rationale 1: Hypokalemia increases the risk of digitalis toxicity in pts who receive this drug for heart failure. Rationale 2: More digoxin is not needed. Rationale 3: A diuretic may cause further fluid loss. Rationale 4: There is inadequate information to assess for concerns related to fluid volume deficits.

The nurse is caring for a pt diagnosed with renal failure. Which of the following does the nurse recognize as compensation for the acid-base disturbance found in pts with renal failure? 1. The pt breathes rapidly to eliminate carbon dioxide. 2. The pt will retain bicarbonate in excess of normal. 3. The pH will decrease from the present value. 4. The pt's oxygen saturation level will improve.

Answer: 1 Rationale 1: In metabolic acidosis compensation is accomplished through increased ventilation or "blowing off" C02. This raises the pH by eliminating the volatile respiratory acid & compensates for the acidosis. Rationale 2: Because compensation must be performed by the system other than the affected system, the pt cannot retain bicarbonate; the manifestation of metabolic acidosis of renal failure is a lower than normal bicarbonate value. Rationale 3: Metabolic acidosis of renal failure causes a low pH; this is the manifestation of the disease process, not the compensation. Rationale 4: Oxygenation disturbance is not part of the acid-base status of the pt with renal failure.

A pt is diagnosed with severe hyponatremia. The nurse realizes this pt will mostly likely need which of the following precautions implemented? 1. seizure 2. infection 3. neutropenic 4. high-risk fall

Answer: 1 Rationale 1: Severe hyponatremia can lead to seizures. Seizure precautions such as a quiet environment, raised side rails, & having an oral airway at the bedside would be included. Rationale 2: Infection precautions not specifically indicated for a pt with hyponatremia. Rationale 3: Neutropenic precautions not specifically indicated for a pt with hyponatremia. Rationale 4: High-risk fall precautions not specifically indicated for a pt with hyponatremia.

When analyzing an arterial blood gas report of a pt with COPD & respiratory acidosis, the nurse anticipates that compensation will develop through which of the following mechanisms? 1. The kidneys retain bicarbonate. 2. The kidneys excrete bicarbonate. 3. The lungs will retain carbon dioxide. 4. The lungs will excrete carbon dioxide.

Answer: 1 Rationale 1: The kidneys will compensate for a respiratory disorder by retaining bicarbonate. Rationale 2: Excreting bicarbonate causes acidosis to develop. Rationale 3: Retaining carbon dioxide causes respiratory acidosis. Rationale 4: Excreting carbon dioxide causes respiratory alkalosis

The nurse is planning care for a pt with fluid volume overload & hyponatremia. Which of the following should be included in this pt's plan of care? 1. Restrict fluids. 2. Administer intravenous fluids. 3. Provide Kayexalate. 4. Administer intravenous normal saline with furosemide.

Answer: 1 Rationale 1: The nursing care for a pt with hyponatremia is dependent on the cause. Restriction of fluids to 1,000 mL/day is usually implemented to assist sodium increase & to prevent the sodium level from dropping further due to dilution. Rationale 2: The administration of intravenous fluids would be indicated in fluid volume deficit & hypernatremia. Rationale 3: Kayexalate is used in pts with hyperkalemia. Rationale 4: The administration of normal saline with furosemide is used to increase calcium secretion.

When caring for a pt diagnosed with hypocalcemia, which of the following should the nurse additionally assess in the pt? 1. other electrolyte disturbances 2. hypertension 3. visual disturbances 4. drug toxicity

Answer: 1 Rationale 1: The pt diagnosed with hypocalcemia may also have high phosphorus or decreased magnesium levels. Rationale 2: The pt with hypocalcemia may exhibit hypotension, & not hypertension. Rationale 3: Visual disturbances do not occur with hypocalcemia. Rationale 4: Hypercalcemia is more commonly caused by drug toxicities.

The nurse is caring for a pt who is anxious & dizzy following a traumatic experience. The arterial blood gas findings include: pH 7.48, PaO2 110, PaCO2 25, & HCO3 24. The nurse would anticipate which initial intervention to correct this problem? 1. Encourage the pt to breathe in & out slowly into a paper bag. 2. Immediately administer oxygen via a mask & monitor oxygen saturation. 3. Prepare to start an intravenous fluid bolus using isotonic fluids. 4. Anticipate the administration of intravenous sodium bicarbonate.

Answer: 1 Rationale 1: This pt is exhibiting signs of hyperventilation that is confirmed with the blood gas results of respiratory alkalosis. Breathing into a paper bag will help the pt to retain carbon dioxide & lower oxygen levels to normal, correcting the cause of the problem. Rationale 2: The oxygen levels are high, so oxygen is not indicated, & would exacerbate the problem if given. Intravenous fluids would not be the initial intervention. Rationale 3: Not enough information is given to determine the need for intravenous fluids. Rationale 4: Bicarbonate would be contraindicated as the pH is already high.

A pt with a history of stomach ulcers is diagnosed with hypophosphatemia. Which of the following interventions should the nurse include in this pt's plan of care? 1. Request a dietitian consult for selecting foods high in phosphorous. 2. Provide aluminum hydroxide antacids as prescribed. 3. Instruct pt to avoid poultry, peanuts, & seeds. 4. Instruct to avoid the intake of sodium phosphate.

Answer: 1 Rationale 1: Treatment of hypophosphatemia includes treating the underlying cause & promoting a high phosphate diet, especially milk, if it is tolerated. Other foods high in phosphate are dried beans & peas, eggs, fish, organ meats, Brazil nuts & peanuts, poultry, seeds & whole grains. Rationale 2: Phosphate-binding antacids, such as aluminum hydroxide, should be avoided. Rationale 3: Poultry, peanuts, & seeds are part of a high phosphate diet. Rationale 4: Mild hypophosphatemia may be corrected by oral supplements, such as sodium phosphate.

The pt has a serum phosphate level of 4.7 mg/dL. Which interdisciplinary treatments would the nurse expect for this pt? Select all that apply. 1. IV normal saline 2. calcium containing antacids 3. IV potassium phosphate 4. encouraging milk intake 5. increasing vitamin D intake

Answer: 1,2 Rationale: Serum phosphate level of 4.7 mg/dL indicates hyperphosphatemia. IV normal saline promotes renal excretion of phosphate.

The nurse is admitting a new client, 80 years old, with congestive heart failure into your home health agency. The following assessment findings have been determined after meeting the client: overweight but no gain since the client left the hospital two days ago; VS: T 99.0, HR 100, R 22, BP 130/86. Foods eaten include canned soup at each meal, ham, and cheese. When completing the care plan for this client, the nurse should include which of the following nursing diagnosis: a. Improved Gas Exchange b. Risk for Fluid Volume Deficit c. Risk for Fluid Volume Imbalance d. Impaired Skin Integrity

c. Risk for Fluid Volume Imbalance Rationale: Sodium is found in high quantities in the foods noted that the client has consumed. When sodium levels increase in the body, water is retained, adding to the volume of fluid in circulation, making it harder for the body to move fluids through the circulation. Therefore, the excess fluid may in time impair gas exchange if levels eventually act on the lungs; fluid volume is increasing, not decreasing, in this situation, and this problem has no involvement with platelets.

An elderly pt comes into the clinic with the complaint of watery diarrhea for several days with abdominal & muscle cramping. The nurse realizes that this pt is demonstrating which of the following? 1. hypernatremia 2. hyponatremia 3. fluid volume excess 4. hyperkalemia

Answer: 2 Rationale 1: Hypernatremia is associated with fluid retention & overload. FVE is associated with hypernatremia. Rationale 2: This elderly pt has watery diarrhea, which contributes to the loss of sodium. The abdominal & muscle cramps are manifestations of a low serum sodium level. Rationale 3: This pt is more likely to develop clinical manifestations associated with fluid volume deficit. Rationale 4: Hyperkalemia is associated with cardiac dysrhythmias.

The nurse observes a pt's respirations & notes that the rate is 30 per minute & the respirations are very deep. The metabolic disorder this pt might be demonstrating is which of the following? 1. hypernatremia 2. increasing carbon dioxide in the blood 3. hypertension 4. pain

Answer: 2 Rationale 1: Hypernatremia is associated with profuse sweating & diarrhea. Rationale 2: Acute increases in either carbon dioxide or hydrogen ions in the blood stimulate the respiratory center in the brain. As a result, both the rate & depth of respiration increase. The increased rate & depth of lung ventilation eliminates carbon dioxide from the body, & carbonic acid levels fall, which brings the pH to a more normal range. Rationale 3: The respiratory rate in a pt exhibiting hypertension is not altered. Rationale 4: Pain may be manifested in rapid, shallow respirations.

A pt is prescribed 10 mEq of potassium chloride. The nurse realizes that the reason the pt is receiving this replacement is 1. to sustain respiratory function. 2. to help regulate acid-base balance. 3. to keep a vein open. 4. to encourage urine output.

Answer: 2 Rationale 1: Potassium does not sustain respiratory function. Rationale 2: Electrolytes have many functions. They assist in regulating water balance, help regulate & maintain acid-base balance, contribute to enzyme reactions, & are essential for neuromuscular activity. Rationale 3: Intravenous fluids are used to keep venous access not potassium. Rationale 4: Urinary output is impacted by fluid intake not potassium.

A pt with fluid retention related to renal problems is admitted to the hospital. The nurse realizes that this pt could possibly have which of the following electrolyte imbalances? 1. hypokalemia 2. hypernatremia 3. carbon dioxide 4. magnesium

Answer: 2 Rationale 1: The kidneys are the principal organs involved in the elimination of potassium. Renal failure is often associated with elevations potassium levels. Rationale 2: The kidney is the primary regulator of sodium in the body. Fluid retention is associated with hypernatremia. Rationale 3: Carbon dioxide abnormalities are not normally seen in this type of pt. Rationale 4: Magnesium abnormalities are not normally seen in this type of pt.

The pt, newly diagnosed with diabetes mellitus, is admitted to the emergency department with nausea, vomiting, & abdominal pain. ABG results reveal a pH of 7.2 & a bicarbonate level of 20 mEq/L. Which other assessment findings would the nurse anticipate in this pt? Select all that apply. 1. tachycardia 2. weakness 3. dysrhythmias 4. Kussmaul's respirations 5. cold, clammy skin

Answer: 2,3,4 Rationale: Further assessment findings of this condition are weakness, bradycardia, dysrhythmias, general malaise, decreased level of consciousness, warm flushed skin, & Kussmaul's respirations. Rationale: These ABG results, coupled with the pt's recent diagnosis of diabetes mellitus & history of vomiting would lead the nurse to suspect metabolic acidosis. Further assessment findings of this condition are weakness, bradycardia, dysrhythmias, general malaise, decreased level of consciousness, warm flushed skin, & Kussmaul's respirations.

The pt is receiving intravenous potassium (KCL). Which nursing actions are required? Select all that apply. 1. Administer the dose IV push over 3 minutes. 2. Monitor the injection site for redness. 3. Add the ordered dose to the IV hanging. 4. Use an infusion controller for the IV. 5. Monitor fluid intake & output.

Answer: 2,4,5

30. Which action by a patient indicates that the home health nurse's teaching about glargine and regular insulin has been successful? a. The patient administers the glargine 30 minutes before each meal. b. The patient's family prefills the syringes with the mix of insulins weekly. c. The patient discards the open vials of glargine and regular insulin after 4 weeks. d. The patient draws up the regular insulin and then the glargine in the same syringe.

c. The patient discards the open vials of glargine and regular insulin after 4 weeks. - Insulin can be stored at room temperature for 4 weeks. Glargine should not be mixed with other insulins or prefilled and stored. Short-acting regular insulin is administered before meals, and glargine is given once daily.

29. A patient who has type 2 diabetes is being prepared for an elective coronary angiogram. Which information would the nurse anticipate might lead to rescheduling the test? a. The patient's most recent A1C was 6.5%. b. The patient's blood glucose is 128 mg/dL. c. The patient took the prescribed metformin today. d. The patient took the prescribed captopril this morning.

c. The patient took the prescribed metformin today. - To avoid lactic acidosis, metformin should be discontinued a day or 2 before the coronary angiogram and should not be used for 48 hours after IV contrast media are administered. The other patient data will also be reported but do not indicate any need to reschedule the procedure.

The nurse should be alert for which manifestations n a patient receiving a loop diuretic? a. Restlessness and agitation b. Paresthesias and irritability c. Weak, irregular pulse and poor muscle tone d. Increased blood pressure and muscle spasms

c. Weak, irregular pulse and poor muscle tone Rationale: Loop diuretics may result in renal loss of potassium (i.e., hypokalemia). Clinical manifestations of hypokalemia include fatigue, muscle weakness, leg cramps, nausea, vomiting, paralytic ileus, soft, muscle flab, paresthesias, decreased reflexes, weak, irregular pulse, polyuria, hyperglycemia, and electrocardiographic changes.

31. A patient with diabetes rides a bicycle to and from work every day. Which site should the nurse teach the patient to use to administer the morning insulin? a. thigh. c. abdomen. b. buttock. d. upper arm.

c. abdomen.

8. An important nursing responsibility related to pain is to a. leave the patient alone to rest. b. help the patient appear to not be in pain. c. believe what the patient says about the pain. d. assume responsibility for eliminating the patient's pain.

c. believe what the patient says about the pain.

5. A cancer patient who reports ongoing, constant moderate pain with short periods of severe pain during dressing changes is a. probably exaggerating his pain. b. best treated by referral for surgical treatment of his pain. c. best treated by receiving a long-acting and a short-acting opioid. d. best treated by regularly scheduled short-acting opioids plus acetaminophen.

c. best treated by receiving a long-acting and a short-acting opioid.

2. A patient is receiving a PCA infusion after surgery to repair a hip fracture. She is sleeping soundly but awakens when the nurse speaks to her in a normal tone of voice. Her respirations are 8 breaths/minute. The most appropriate nursing action in this situation is to a. stop the PCA infusion. b. obtain an oxygen saturation level. c. continue to closely monitor the patient. d. administer naloxone and contact the physician.

c. continue to closely monitor the patient.

4. Unrelieved pain is a. expected after major surgery. b. expected in a person with cancer. c. dangerous and can lead to many physical and psychologic complications. d. an annoying sensation, but it is not as important as other physical care needs.

c. dangerous and can lead to many physical and psychologic complications.

9. Providing opioids to a dying patient who is experiencing moderate to severe pain a. may cause addiction. b. will probably be ineffective. c. is an appropriate nursing action. d. will likely hasten the person's death.

c. is an appropriate nursing action.

The typical fluid replacement for the patient with a fluid volume deficit is: a. dextran b. 0.45% saline c. lactated Ringer's d. 5% dextrose in 0.45% saline

c. lactated Ringer's Rationale: Administration of an isotonic solution expands only the extracellular fluid (ECF). There is no net loss or gain from the intracellular fluid (ICF). An isotonic solution is the ideal fluid replacement for a patient with an ECF volume deficit. Examples of isotonic solutions include lactated Ringer's solution and 0.9% NaCl.

2. A patient screened for diabetes at a clinic has a fasting plasma glucose level of 120 mg/dL (6.7 mmol/L). The nurse will plan to teach the patient about a. self-monitoring of blood glucose. b. using low doses of regular insulin. c. lifestyle changes to lower blood glucose. d. effects of oral hypoglycemic medications.

c. lifestyle changes to lower blood glucose. - The patient's impaired fasting glucose indicates prediabetes, and the patient should be counseled about lifestyle changes to prevent the development of type 2 diabetes. The patient with prediabetes does not require insulin or oral hypoglycemics for glucose control and does not need to self-monitor blood glucose.

An elderly pt does not complain of thirst. What should the nurse do to assess that this pt is not dehydrated? 1. Ask the physician for an order to begin intravenous fluid replacement. 2. Ask the physician to order a chest x-ray. 3. Assess the urine for osmolality. 4. Ask the physician for an order for a brain scan.

Answer: 3 Rationale 1: It is inappropriate to seek an IV at this stage. Rationale 2: There is no indication the pt is experiencing pulmonary complications thus a cheat x-ray is not indicated. Rationale 3: The thirst mechanism declines with aging, which makes older adults more vulnerable to dehydration & hyperosmolality. The nurse should check the pt's urine for osmolality as a 1st step in determining hydration status before other detailed & invasive testing is done. Rationale 4: There is no data to support the need for a brain scan.

A postoperative pt is diagnosed with fluid volume overload. Which of the following should the nurse assess in this pt? 1. poor skin turgor 2. decreased urine output 3. distended neck veins 4. concentrated hemoglobin & hematocrit levels

Answer: 3 Rationale 1: Poor skin turgor is associated with fluid volume deficit. Rationale 2: Decreased urine output is associated with fluid volume deficit. Rationale 3: Circulatory overload causes manifestations such as a full, bounding pulse; distended neck & peripheral veins; increased central venous pressure; cough; dyspnea; orthopnea; rales in the lungs; pulmonary edema; polyuria; ascites; peripheral edema, or if severe, anasarca, in which dilution of plasma by excess fluid causes a decreased hematocrit & blood urea nitrogen (BUN); & possible cerebral edema. Rationale 4: Increased hemoglobin & hematocrit values are associated with fluid volume deficit.

A pt is admitted for treatment of hypercalcemia. The nurse realizes that this pt's intravenous fluids will most likely be which of the following? 1. dextrose 5% in water 2. dextrose 5% in normal saline 3. dextrose 5% in 1/2 normal saline 4. normal saline

Answer: 4 Rationale 1: Isotonic saline is used because sodium excretion is accompanied by calcium excretion through the kidneys.

A pt's blood gases show a pH greater of 7.53 & bicarbonate level of 36 mEq/L. The nurse realizes that the acid-base disorder this pt is demonstrating is which of the following? 1. respiratory acidosis 2. metabolic acidosis 3. respiratory alkalosis 4. metabolic alkalosis

Answer: 4 Rationale 1& 2: Respiratory acidosis & metabolic acidosis are both consistent with pH less than 7.35. Rationale 3: Respiratory alkalosis is associated with a pH greater than 7.45 & a PaCO2 of less than 35 mmHG. It is caused by respiratory related conditions. Rationale 4: Arterial blood gases (ABGs) show a pH greater than 7.45 & bicarbonate level greater than 26 mEq/L when the pt is in metabolic alkalosis.

The nurse is admitting a pt who was diagnosed with acute renal failure. Which of the following electrolytes will be most affected with this disorder? 1. calcium 2. magnesium 3. phosphorous 4. potassium

Answer: 4 Rationale 1: This pt will be less likely to develop a calcium imbalance. Rationale 2: This pt will be less likely to develop a magnesium imbalance. Rationale 3: This pt will be less likely to develop a phosphorous imbalance. Rationale 4: Because the kidneys are the principal organs involved in the elimination of potassium, renal failure

A 28-year-old male pt is admitted with diabetic ketoacidosis. The nurse realizes that this pt will have a need for which of the following electrolytes? 1. sodium 2. potassium 3. calcium 4. magnesium

Answer: 4 Rationale 4: One risk factor for hypomagnesaemia is an endocrine disorder, including diabetic ketoacidosis.

The nurse is administering IV nitroglycerin to a client with a myocardial infarction (MI). Which of the following actions should the nurse take to evaluate the effectiveness of the medication? a. Check blood pressure. b. Monitor apical pulse rate. c. Monitor for dysrhythmias. d. Ask about chest discomfort

Ask about chest discomfort

The nurse is caring for a client with hypertension and has just administered the initial dose of labetalol. Which of the following actions should the nurse take? a. Encourage oral fluids to prevent dry mouth or dehydration. b. Instruct the client to ask for help if heart palpitations occur. c. Ask the client to request assistance when getting out of bed. d. Teach the client that headaches may occur with this medication.

Ask the client to request assistance when getting out of bed.

. During a physical examination of a client, the nurse palpates the point of maximal impulse (PMI) in the sixth intercostal space lateral to the left midclavicular line. Which of the following actions should the nurse implement next? a. Document that the PMI is in the normal anatomic location. b. Ask the client about risk factors for coronary artery disease. c. Auscultate both the carotid arteries for the presence of a bruit. d. Assess the client for symptoms of left ventricular hypertrophy.

Assess the client for symptoms of left ventricular hypertrophy.

To assist the client with coronary artery disease (CAD) in making appropriate dietary changes, which of the following nursing interventions will be most effective? a. Instruct the client that a diet containing no saturated fat and minimal sodium will be necessary. b. Emphasize the increased risk for cardiac problems unless the client makes the dietary changes. c. Assist the client to modify favourite high-fat recipes by using polyunsaturated oils when possible. d. Provide the client with a list of low-sodium, low-cholesterol foods that should be included in the diet.

Assist the client to modify favourite high-fat recipes by using polyunsaturated oils when possible.

The nurse is assessing a newly admitted client and notes a thrill along the left sternal border. To obtain more information about the cause of the thrill, which of the following actions should the nurse take next? a. Auscultate for any cardiac murmurs. b. Find the point of maximal impulse. c. Compare the apical and radial pulse rates. d. Palpate the quality of the peripheral pulses

Auscultate for any cardiac murmurs.

Which patient is at risk for respiratory acidosis? a. The patient with uncontrolled diabetes mellitus b. The patient with chronic pulmonary disease c. The patient who is very anxious d. The patient who overuses antacids

B Respiratory acidosis occurs when inadequate ventilation causes retention of CO2, which is acidic. Therefore, a patient with chronic pulmonary disease would be at risk. Other conditions causing respiratory acidosis include respiratory depression, barbiturate or opioid overdose, respiratory failure, and asthma.

A preoperative patient reveals that an uncle died during surgery because of a fever and cardiac arrest. The perioperative nurse alerts the surgical team, knowing that if the patient is at risk for malignant hyperthermia, a. the surgery will have to be cancelled b. specific precautions can be taken to safely anesthetize the patient c. dantrolene (Dantrium) must be given to prevent hyperthermia during surgery d. the patient should be placed on a cooling blanket during the surgical procedure

B (Although malignant hyperthermia can result in cardiac arrest and death, if the patient is known or suspected to be at risk for the disorder, appropriate precautions taken by the ACP can provide for safe anesthesia for the patient. Because preventive measures are possible if the risk is known, it is critical that the preoperative assessment include a careful family history of surgical events)

During epidural and spinal anesthesia, the nurse should monitor the patient for a. spinal headache b. hypotension and bradycardia c. loss of consciousness d. downward extension of nerve block

B (During epidural and spinal anesthesia, a sympathetic nervous system blockade may occur that results in hypotension, bradycardia, and nausea and vomiting. A spinal headache may occur after, not during, spinal anesthesia, and unconsciousness and seizures are indicative of IV absorption overdose. Upward extension of the effect of the anesthesia results in inadequate respiratory excursion and apnea)

Monitored anesthesia care (MAC) is being considered for a patient undergoing a cervical dilation and endometrial biopsy in health care provider's office. The patient asks the nurse, "What is the MAC?" The nurse's response is based on the knowledge that MAC a. can be administered only by anesthesiologists or nurse anesthetists b. enables the patient to respond to commands and accept painful procedures c. should never be used outside of the OR because of the risk of serious complications d. is so safe that it can be administered by nurses with direction from health care providers

B (MAC refers to sedation that allows the patient to manage his or her own airway and respond to commands, and yet the patient can emotionally and physically accept painful procedures. Drugs are used to provide analgesia, relieve anxiety, and/or provide amnesia. It can be administered by personnel other than anesthesiologists, but nurses should be specially trained in the techniques of MAC to carry out this procedure because of the high risk of complications resulting in clinical emergencies)

The physical environment of a surgery suite is designed primarily to promote a. electrical safety b. medical and surgical asepsis c. comfort and privacy of the patient d. communication among the surgical team

B (Medical and surgical asepsis Although all the factors are important to the safety and well being of the patient, the first consideration in the physical environment of the surgical suite is prevention of transmission of infection to the patient)

When transporting an inpatient to the surgical department, the nurse from another area of the hospital has access to a. the clean core b. the holding area c. corridors of the surgical suite d. an unprepared operating room

B (The holding area. Persons in street clothes or attire other than surgical scrub clothing can interact with personnel of the surgical suite in unrestricted areas, such as the holding area, nursing station, control desk, or lockers rooms. Only authorized personnel wearing surgical attire and hair covering are allowed in semirestricted areas, such as corridors, and masks must be worn in restricted areas, such as operating rooms, clean core, and scrub sink areas.)

Because of the rapid elimination of volatile liquids used for general anesthesia, the nurse should anticipate that early in the anesthesia recovery period, the patient will need a. warm blankets b. analgesic medication c. observation for respiratory depression d. airway protection in anticipation of vomiting

B (The volatile liquid inhalation agents have very little residual analgesia, and patients experience early onset of pain when the agents are discontinued. They are associated with a low incidence of nausea and vomiting. Prolonged respiratory depression is not common because of their rapid elimination. Hypothermia is not related to use of these agents, but they may precipitate malignant hyperthermia in conjunction with neuromuscular blocking agents.)

The nurse is admitting a patient who is scheduled to undergo a cardiac catheterization. Which of the following allergies is most important for the nurse to assess before this procedure? A) Iron B) Iodine C) Aspirin D) Penicillin

B) Iodine The physician usually will use an iodine-based contrast to perform this procedure. Therefore it is imperative to know whether or not the patient is allergic to iodine or shellfish.

A 59-year-old man has presented to the emergency department with chest pain. Which of the following components of his subsequent blood work is most clearly indicative of a myocardial infarction (MI)? A) CK-MB B) Troponin C) Myoglobin D) C-reactive protein

B) Troponin Troponin is the biomarker of choice in the diagnosis of MI, with sensitivity and specificity that exceed those of CK-MB and myoglobin. CRP levels are not used to diagnose acute MI.

Auscultation of a patient's heart reveals the presence of a murmur. This assessment finding is a result of A) Increased viscosity of the patient's blood. B) Turbulent blood flow across a heart valve. C) Friction between the heart and the myocardium. D) A deficit in heart conductivity that impairs normal contractility.

B) Turbulent blood flow across a heart valve. Turbulent blood flow across the affected valve results in a murmur. A murmur is not a direct result of variances in blood viscosity, conductivity, or friction between the heart and myocardium.

1. To monitor for complications in a patient with type 2 diabetes, which tests will the nurse in the diabetic clinic schedule at least annually (select all that apply)? a. Chest x-ray b. Blood pressure c. Serum creatinine d. Urine for microalbuminuria e. Complete blood count (CBC) f. Monofilament testing of the foot

B, C, D, F

The nurse provides discharge instructions for a 40-year-old woman who is newly diagnosed with cardiomyopathy. Which statement, if made by the patient, indicates that further teaching is necessary? A."I will avoid lifting heavy objects." B. "I can drink alcohol in moderation." C."My family will need to take a CPR course." D. "I will reduce stress by learning guided imagery."

B. "I can drink alcohol in moderation." Patients with cardiomyopathy should avoid alcohol consumption, especially in patients with alcohol-related dilated cardiomyopathy. Avoiding heavy lifting and stress, as well as family members learning CPR, are recommended teaching points.

Which of the following assessment findings for a client who is receiving furosemide to treat stage 2 hypertension is most important to report to the health care provider? a. Blood glucose level of 10 mmol/L b. Blood potassium level of 3.0 mmol/L c. Early morning BP reading of 164/96 mm Hg d. Orthostatic systolic BP decrease of 12 mm Hg

Blood potassium level of 3.0 mmol/L

The nurse is auscultating over the client's abdominal aorta and hears a humming sound. Which of the following terms should the nurse use to document this finding? a. Thrill b. Bruit c. Heave d. Murmur

Bruit

During surgery, a patient has a nursing diagnosis of risk for perioperative positioning injury. A common risk factor for this nursing diagnosis is a. skin lesions b. break in sterile technique c. musculoskeletal deformities d. electrical or mechanical equipment failure

C (Musculoskeletal deformities can be a risk factor for positioning injuries and require special padding and support on the operating table. Skin lesions and break in sterile technique are risk factors for infection, and electrical equipment failure may lead to other types of injuries.)

The primary goal of the circulating nurse during preparation of the operating room, transferring and positioning the patient, and assisting the anesthesia team is a. avoiding any type of injury to the patient b. maintaining a clean environment for the patient c. providing for patient comfort and sense of well being d. preventing breaks in aseptic technique by the sterile members of the team

C (Providing for patient comfort and sense of well being The protection of the patient from injury in the operating room environment is maintained by the circulating nurse by ensuring functioning equipment, preventing falls and injury during transport and transfer, monitoring asepsis, and being with the patient during anesthesia induction)

Goals for patient safety in the operating room (OR) include the Universal Protocol, in which a. all surgical centers of any type must submit reports on patient safety infractions to the accreditation agencies b. the members of the surgical team stop whatever they are doing to check that all sterile items have been properly prepared c. a surgical timeout is performed just before the procedure is started to verify patient identity, surgical procedure, and surgical site d. all members of the surgical team pause right before surgery to meditate for 1 minute to decrease stress and possible errors

C (The Universal Protocol supported by The Joint Commission is used to prevent wrong site, wrong procedure, and wrong surgery in view of a high rate of these problems nationally. It involves pausing just before the procedure starts to verify identity, site, and procedure.)

While assessing the cardiovascular status of a patient, the nurse performs auscultation. Which of the following practices should the nurse implement into the assessment during auscultation? A) Position the patient supine. B) Ask the patient to hold his or her breath. C) Palpate the radial pulse while auscultating the apical pulse. D) Use the bell of the stethoscope when auscultating S1 and S2.

C) Palpate the radial pulse while auscultating the apical pulse. In order to detect a pulse deficit, simultaneously palpate the radial pulse when auscultating the apical area. The diaphragm is more appropriate than the bell when auscultating S1 and S2. A sitting or side-lying position is most appropriate for cardiac auscultation. It is not necessary to ask the patient to hold his or her breath during cardiac auscultation.

A 72-year-old man with a history of aortic stenosis is admitted to the emergency department. He reports severe left-sided chest pressure radiating to the jaw. Which medication, if ordered by the health care provider, should the nurse question? A. Aspirin B. Oxygen C. Nitroglycerin D. Morphine sulfate

C. Nitroglycerin Aspirin, oxygen, nitroglycerin, and morphine sulfate are all commonly used to treat acute chest pain suspected to be caused by myocardial ischemia. However, nitroglycerin should be used cautiously or avoided in patients with aortic stenosis as a significant reduction in blood pressure may occur. Chest pain can worsen because of a drop in blood pressure.

Pink

Cap to one-way valve

Yellow #2

Cathetar

Gray #2

Catheter sheath

The nurse is caring for a client who has just diagnosed with hypertension and has a new prescription for captopril. Which of the following information is important to include when teaching the client? a. Check BP daily before taking the medication. b. Increase fluid intake if dryness of the mouth is a problem. c. Include high-potassium foods such as bananas in the diet. d. Change position slowly to help prevent dizziness and falls.

Change position slowly to help prevent dizziness and falls.

An elderly pt who is being medicated for pain had an episode of incontinence. The nurse realizes that this pt is at risk for developing 1. dehydration. 2. over-hydration. 3. fecal incontinence. 4. a stroke.

Correct Answer: 1 Rationale 1: Functional changes of aging also affect fluid balance. Older adults who have self-care deficits, or who are confused, depressed, tube-fed, on bed rest, or taking medications (such as sedatives, tranquilizers, diuretics, & laxatives), are at greatest risk for fluid volume imbalance. Rationale 2: There is inadequate evidence to support the risk of over-hydration. Rationale 3: There is inadequate evidence to support the risk of fecal incontinence. Rationale 4: There is inadequate evidence to support the risk of a stroke.

Which pts are at risk for the development of hypercalcemia? Select all that apply. 1. the pt with a malignancy 2. the pt taking lithium 3. the pt who uses sunscreen to excess 4. the pt with hyperparathyroidism 5. the pt who overuses antacids

Correct Answer: 1,2,4,5 Rationale 1: Pts with malignancy are at risk for development of hypercalcemia due to destruction of bone or the production of hormone-like substances by the malignancy. Rationale 2: Lithium & overuse of antacids can result in hypercalcemia. Hypercalcemia can result from hyperparathyroidism which causes release of calcium from the bones, increased calcium absorption in the intestines & retention of calcium by the kidneys. Rationale 3: The pt who uses sunscreen to excess is more likely to have a vitamin D deficiency which would result in hypocalcemia. Rationale 4: Hypercalcemia can result from hyperparathyroidism which causes release of calcium from the bones, increased calcium absorption in the intestines & retention of calcium by the kidneys. Rationale 5: Lithium & overuse of antacids can result in hypercalcemia.


Related study sets

Brunner Suddarth Chapter 06 Transcultural Nursing

View Set

Women's Health - Chapter 6 - Prep U

View Set

BUS205 CH. 3 Human Resource Management Strategy & Analysis

View Set

Law Quiz 1- Food Drug and Cosmetic Act

View Set